Вы находитесь на странице: 1из 150

Polio CSF=Lymphocytic Pleocytosis

MS oligonal bands on electophoresis


ALS (Lou Gherig's Disease) defect of superoxide dismutase 1
EBV china kissing monoclonal carcinoma of the nasopharynx
Internal Iliac branch ligation required to control heavy vaginal bleeding
Streptococcus Pneumoniae prophylaxis required if pt has a splenctomy
Locked-in Syndrome basilar artery
Inhaled glue "pt not themself" confusion, clumsiness, frequent falls ataxic gait
Xanthoma/ Achielles Tendon absence of functional LDL receptors in hepatocytes
pupils 4 mm in diameter and not reactive to light; due to trauma in
Middle Meningeal Artery
temporal area
Activation of Adenylyl Cyclase Toxin stool shows gram negative, comma-shaped bacteria
Alveolar Macrophase shipyard workers
NSAIDs use in hypertensive pt with
vasodilating prostaglandins at the afferent arteriole
bilateral renal artery stenosis
Calcitonin marker to monitor for thyroid neoplasm
Aortic Stenosis cardiac valve defect and concentric left ventricular hypertrophy
MS Charcot's Triad
Aldoesterone-secreting Adrenal
decreased plasma renin activity
Adenoma
Malignant Hyperthermia decreases release of Ca+ from the SR
pincer grasp finger feeding standing while holding onto a table
9mo. old Baby
playing peek-a-boo
Leptin suppresses appetite by its action in the CNS
female; facial hair chest hair clitoromegaly normal uterus
21-Hydroxylase increased concentrations of 17-hydroxyprogesterone and
androstenedione
Trypanosoma Cruzi brazil organism
Resolution of Pneumococcal
metaplasia of mesenchymal cells to pneumocytes
Pneumonia
Interstitial Pulmonary Fibrosis Increased radial traction on airways
Early Septic Shock Rx IV 1 liter of isotonic saline
Dissecting Aneurysm X-ray widened aortic arch
Ascites Rx in addition to loop diuretics spironlactone
Randomized clinical trial subjects assigned by coin toss to one of two groups
Tumor Necrosis Factor antibodies directed against in monocolonal antibody preparation
cholesterol decreasing drug inhibiting the transport of cholesterol
Ezetimibe MOA
through the intestinal wall
DVT Rx potentiates the action of antithrombin III
First Degree Burn severe erythema of back and extremities NO BLISTERS
Osteomyelitis predominance of neutrophils persistent foot pain
Alendronate MOA inhibition of osteoclast-mediation bone resorption
Vagus Nerve protude the tongue and say "Ah"
GI Blood Loss labs Hemoglobin: 9.5g/dL Leukocyte: 5400 Platelets: 350,000
Left Achilles Tendon Reflex strong with lower extremity is immobilized in a cast
Loperamide opioid antidiarrheal agent with little CNS effect
IgM antibody isotype most likely causing agglutination
Nephrolithiasis (Kidney Stones) can occur due to gout
low energy irritability crying spells difficulty falling asleep
Major Depressive Disorder
waking up frequently at night
Proliferative Endometrial Tissue seen
biopsy of ovarian cyst and/or peritoneal cyst
in
5a-reductase gene mutation labia majora would develop into scrotum
Parietal cell absense after gastrectomy
erythema over nose, cheeks, and scattered telangiectasia and a few
Rosacea
papules
Pelvic Splanchnic Nerve Dysfunction
constipation abdominal distension
Sx
gram positive rod fever, chills, and muscle aches can occur in
Listeria Monocytogenes
pregnant women
Ultrasound shows bilateral increased hydrostatic pressure in bowman space leading to renal
hydronephrosis and dilated ureters failure
Aflatoxin china peanut farmer
pt is confused by everything and is known to not behave in such a
Delirium
manner at home
Bronzing of Skin increased intestinal iron absorption
Inferior Rectal Artery 15-mm, blue-tinged, rounded mass at the anal margin
Newborn with absence of bowel gas in
incomplete formation of pleuroperitoneal membranein
abdomen
absent in specimen of epithelium in pts. with Kartagener
Dynein arms
Syndrome
Metastatic Tumor to Cerebellum ataxia of the left upper and lower extremities
spasticity in arm and legs impaired proprioception in feet
increased muscle stretch reflexes in arms and knees absence of
Vitamin B12 Deficiency
muscle stretch reflexes in ankles bilateral extensor plantar
responses
Farmer with 7-mm red scaly plaque on
actinic keratosis
the helical rim of left ear
Tick Bite in Farmer Rx doxycycline
hypervascular mass 12-cm solid mass on kidney Sx include flank
Renal Cell Adenocarcinoma
pain gross hemateuria
Physician response to pt.'s Rx "using something twice daily can be difficult. I assume you are
noncompliance like most patients who miss at least 10% of treatments"
lower back pain with stiffness stiffness when sat for prolonged
Sacroiliac Joint Inflammation sx
periods of time pain radiates down to buttocks
Ringing of Ears sx salicylate/ Asprin Poisioning
Ulcers Rx sulfasalazine
promotes cell growth and malignancy by causing cellular p53
Ubiquitin Ligase MOA
protein degradation
Trophoblastic tissue necrotic intrauterine mass and metastatic nodule in the lungs
Bilateral Bell's Palsy adverse effect of: Lyme Disease Guillan-Barre
congenitial urethral obstruction; marked dilation of ureters and
Nephroblastoma sx
renal pelvis minimal renal function
Thyroidectomy Consequences decreased PTH decreased Calcium parenthesis of hands and feet
Ischial spine injection with lidocaine alternative to epidural in
Pudendal Nerve Block
labor
Increased JVP and mild ankle edema increased capillary hydrostatic pressure
2/6 holosystolic murmur, left fifth intercoastal space ajacent to
Tricuspid Valve
sternum and increases with inspiration
Camping Trip results in itchy rash in
activation of T lymphocytes
arms and legs
CMV infevction sx petechial rash, microcephaly, and hepatosplenomegaly
Puberty begins when breast buds develop
Hyperthyroidism sx thyroid antibodies
HSV sx in Males broken blisters and open sores on penis
disruption of normal splicing by creation of a new 3' splice site
Beta-Thalassema
mutation from G --> A at position 355
Post Par-tum Depression Rx SSRI
Fibrinous Pericarditis sx pericardial friction rub after acute myocardial infarction
Cervical Biopsy in Microinvasive
neoplastic cells in sub-basement membrane connective tissue
Cervical Carcinoma
Increased amniotic fluid volume tracheoesophageal atresia
IL-8 responsibilty recruitment of neutrophils to inflammatory site
VII (proconvertin) clotting factor first to be decreased by 50%
Warfarn MOA
after initiation of theraphy
vaginal bleeding direct and rebound tenderness with guarding b-
Ectopic Pregnancy
hCG elevated closed cervical os
Overdose of Triiodothyronine in
decreased TSH, Free Throyxine Increased Free Triiodothyronine
Primary Hypothyroidism
Cocaine Users predisposition to myocardial ischemia
The binding site and action of hCG that
testicle; produces estradiol
causes gynecomastia
Strings of Beads sign fibromuscular dysplasia
Bacterial sx are a result of systemic
IL-1 and TNF-alpa
release of
Clostridium Difficle toxin intestinal tract will contain pseudomembranes of fibrin and
inflammatory debris
Lost in Menopause ovarian secretion of 17beta-estradiol
PTT prolongation defect in factor 8 (VIII) antihemophilic factor
Organs glucose produced in liver kidney
increased residual volume and alveolar-arterial Po2 difference
Elderly pulmonary function
decreased arterial Po2
Methylamphetamine MOA increased release of dopamine and norepinephrine
Asthmatic Pt. with Pollen Allergy Rx albuterol (acute resolution)
Ibutilide Adverse Effect Torsades de Pointes
Mallory Hyaline in biopsy seen in alcoholic hepatitis
The right eye does not adduct past the
midline on horizontal gaze when
abducens nerve
looking to the left indicates a lesion
involving
The persistence of leukocytosis in the
absence of infection indicates leukocyte adhesion and transmigration
impairment in
seen in elderly women with vulvar itching that has not resolved
Urethral Prolapse with treatment physical examination with show atrophy and
thinning of labia minora
Rx inhibiting 30s ribosome binding gentamicin
T-independent antigens to T-dependent forms to enhance
H. Influenzae Type B Vaccine
protection
Condition causing anemia and spleen
hereditary Spherocytosis
to increase 5x it's normal size
Antihypertensive causing descreased
hydrocholorthiazide
serum potassium concentration
1, 25-Dihydroxycholecaciferol active form of vitamin D
pts who smoke, are hypertensive and experience continued
Calcified 80% Stenois can occur in
extertional chest pain that is relieved by rest
Sepsis Rx 0.9% Saline
Indication for a Pacemaker third-degree atrioventricular block, hypotension and variable
intensity S1
Erectile dysfunction Rx MOA inhibition of phosphodiesterase (PDE5 inhibitor)
Autoantibody with affinity for
thymoma
acetylcholine receptor seen in
Physician answer to pt inquiry on "Yes, your weight gain can be caused by genes and enviromental
whether weight gain is hereditary factors"
Alcoholic Liver Disease adverse effect gynecomastia
Surgical intervention shows half of the
small intestine is found to have a dark Mesenteric venous thrombosis
purple-red hemorrhagic appearance
stimulates hormone-sensitive lipase in adipocytes that resulted in
Epinephrine
the accumulation of metabolites
Systemic Amyloid renal biopsy will show beta-pleated sheet structure
Defective Heme Synthesis porphyria; ALA rate limiting enzyme
Rx for Urinary Urgency inhibit of muscarinic receptors
Naloxone antidote for narcotic overdose in an emergency situation
Tardive Dyskinesia grimaces rigid jerking purposeless movements of fingers
Osteogenesis Imperfecta type 1 collagen
Cyclosporine MOA in transplant
suppresses early response of T lymphocytes to activation
patients
condition in which pt. avoid interacting with others and social
Schizoid Disorder
activities in addition to being cold and detached
First Generalized Tonic-Clonic Seizure
calcium
abnormality in
Loraradine Rx used to treat allergies; runny nose, itchy and watery eyes
Physiological changes when in water
increased ADH and ANP decreased central blood volume
of 60F for 20 mins
Sepsis after abortion causation decreased plasma fibrinogen concentration
Physician response when pt claims
"Yes, it is"
diagnosis is bad news
Diagnosing possible defect in fatty acid
measurement of serum amino acid concentrations
oxidation
Myocardinal infarction (CK-MB;
protease inactivation by cytoplasmic free calcium ions
troponin) markers result from
Neurofibromatosis Type 1 autosomal dominant
Chandelier Sign (cervical motion
gonorrhea dx
tenderness)
"It must be difficult for you to accept this diagnosis when you feel
Physician response to pt in denial
healthy"
Criteria for informed consent family agreement, competence, and cost
Renal stone composition when urine
struvite
analysis pH is 8
C5a; produced increased fibroblast migration and proliferation,
Cellular production in second-degree
increased synthesis of collagen and fibronectin, and decreased
burn precipitated by which component
degradation of extracellular matrix by metalloproteinases
Hemiballisum; uncontrollable irregular movements of the LEFT
Damage to right subthalamic
side of the body
Damage to posterior pituitary gland progressive thirst and urinary frequency
Abnormality in 6 week old baby
vomitting a small amount of milk 2 to immature lower esophageal spinchter
3 times a day
firm, smooth, umbilicated papules 2 to 4 mm in diameter in
Poxvirus sx
clusters
Bullous Pemphigoid results from development of autoantibodies against desmosomal proteins
Bulimic patients will have enlargement
parotid gland
of
HCO3- levels in advanced phase of
increased urinary excretion
COPD
Most common benign tumor of
myoxma
connective tissue
Dysphagia causing difficulty
swallowing solids, dilated
left atrium
cardiomyopathy would be as a result of
enlargement where
Agent used to slow the DNA
cyclophosphamide
replication process of neoplastic cells
in cancer patients
Weakness of grasp is indicative of rheumatoid arthritis
This casual virus replicates its genome
within the cell's nucleus
Caspofungin MOA beta-glucan carbohydrates in the cell wall
Stool analysis showing increased fat
concentration indicative of deficiency vitamin E
in
Physician response when pt. complains "I'm sorry I got delayed. I hope I haven't made you late somewhere
about tardiness else"
L3 to L4 herinations causes pain down the distal anterior thigh knee medial leg and foot
Africa trip + Wright Stain dx Malaria
Subclavian Central Catheter bacterial
Enterococcus Facalis
infection
E-Coli Virulence Factor Mannose-binding (type 1) fimbria
3 D's: dermatitis, dementia, and diarrhea (confusion, rash, and
Pellagra diarrhea) lack of nicotinic acid or its precursor, tryptophan in their
diet
Hypertensive patient already on
hydrocholorthiazide if not responding
Lisinopril
to Rx should be put on this ACE
inhibitor
Pt. experiencing proteinuria and
elevated hemoglobin is likely to have Creatine Kinase
which elevated marker?
Vertebra region in the lower right
quadrant about 5 cm superomedial to T10
the anterosuperior iliac spine?
Which area should be avoided during
ablatio to leave the sinoatrial the junction of the superior vena cava and the right atrium
(packmaker) node intact in a pt.?
Marked increase in dopamine
concentration when rising from a dopamine beta-hydroxylase
supine position to standing marks a
deficiency of?
Most common bacterial infection from
staphylcoccus aureus
breast feeding
Embryology: Syncytiotrophoblast
hCG
secretes which hormone
Embryology: Trophoblast acquisitions
mother
energy from
Embryology: Inner Cell mass will establish order
Embryology: Progesterone maintains
menstration
the endometrial lining to prevent
Embryology: Neural Crest derived
from ECTODERM, but which layers notocord + mesoderm
make this even happen
PNS, ear, eye adrenal gland, mouth, heart, digestive system,
Neural Crest derivatives
thyroid, and skin
Embryology: Germ layers serving as
endodermal yolk sac
secondary energy reservoire
Embryology: Week 10 sex of fetus
Teratogenic Rx: Aminoglycosides Ototoxicity
Teratogenic Rx: ACE inhibitors renal malformations
Teratogenic Rx: Fluroquinolones cartilage damage
Teratogenic Rx: Tetracyclines discolored Teeth
Teratogenic Rx: Cyclophosphamide facial anomalies, limb hypoplasia, absence of digits
Teratogenic Rx: Methotrexate abortion, NTD
Teratogenic Rx: Carbamazipine NTD
Teratogenic Rx: Valporic Acid NTD
Teratogenic Rx: Phenytoin fetal hydration syndrome
Teratogenic Rx: Lithium ebsteins anomaly
Teratogenic Rx: Statins cns and limb anomalies
Teratogenic Rx: Wafarin facial/ limb/ CNS anomalies, spontaneous abortion
Teratogenic Rx: Diethylstilbestrol clear cell vaginal adenocarcinoma
(DES)
phocomelia; rare congenital deformity in which the hands or feet
Teratogenic Rx: Thalidomide are attached close to the trunk, the limbs being grossly
underdeveloped or absent.
Teratogenic Rx: Isotretinoin spontaneous abortion
Holoprosencephaly Consequence from ethanol
Limb Hypoplasia Consequence from thalidomide; cyclophosphamide
blueprint for skeletal morphology (where things are supposed to
Homebox (HOX) Genes
go); code for transcription regulator
Mutation HOXD-13 genes will result
synpolydactyly (fused 3rd and 4th digit)
in
4 chamber heart begins to develop at
4
week
Most common cause of NTD folate deficency
Most common cause of congenital
alcohol use in pregnancy
malformations in the US
Most common cause of congenital
fetal alcohol syndrome
intellectual disability in the US
Vitamin A excess during pregnancy
cleft palate
will lead to a
COP-II functions in the cell cycle to
from ER to cis-Golgi; anterograde trafficking
help move products
This helps transport hydrolase enzymes
clathrin
from trans-Golgi to lysosome
Mannose-6-phosphate deficency
I-cell disease
results in
Mitchondria acts as buffer to calcium
Intermediate Filament Structures:
Structural component of: connective tissue
Vimenten
Progeria (Advanced Aging) nuclear lamins mutation
Cholesterol makes up 50% of the plasma membrane
Membrane Bound Receptors Tyrosine Kinase G-Protein-coupled Steroid receptors
PDGF and IGF-1 receptors 2alpha bound by disulfide bonds bidning extracellular ligand
Main Role of Plasma Membrane keeps sodium out of the cell; potassium inside of the cell
Corticosteriods inhibits phopholipase A2
Zileuton inhibits lipoxygenase
Zafirlukast and Montelukast inhibits leukotrienes
Cyclooxygenase inhibited by NSAIDS, Acetomenaphin, COX-2
Cell membrane lipid converted to
phosphatidylinositol
arachidonic acid by phospholipase A2
Which cells are constantly regenerating
skin, hair follicles, and bone marrow
(stays in G1 phase and no G0 phase)
In apoptosis, the plasma membrane is blebbing away via phagocytosis
Intrinsic pathway: bcl-2 is anti-apoptotic
Intrinsic pathway: BAX is pro-apoptotic; increase in mitochondrial permability
Extrinsic pathway: Death Receptor TNF + Fas
Extrinsic pathway: Killer T Cells use perforin
Result of Granzyme B entering cells activation of caspases
Intrinsic pathway: Increased
Cytochrome C
Mitochondrial permeability releases
nuclear pyknosis karyolysis karyorrhexis Ca2+ influx leading to
Irreversible Cell Injuries
caspase activation
Neutrophils found in acute inflammation
Collagen primarily needs vitamin C
Leukocyte Adhesion Syndrome delayed umbilicus seperate
Leukocyte Adhesion Syndrome abnormal integrin molecules
Granulomas are found in chronic inflammation
Collection of macrophages ultimately
granulomas
become
Receptor for Rolling (inflammation) E-Selection (endothelium)
Receptor(s) for tight binding
LFA-1; ICAM-1
(inflammation)
Metals known to facilitate production
iron and copper
of oxygen free radicals
Strong (I) Slippery (II) Bloody (III)
collagen
BM (IV)
Step 1 in the production of collagen precollagen sythesis alpha chains
Step 2 in the production of collagen hydroxylation of lysine and proline
glycosylation of hydroxylated lysine essentially making
Step 3 in the production of collagen
procollagen
Step 4 in the production of collagen exocytosis
Collagen construction taken place in fibroblasts
Step 5 in the production of collagen cleavage of terminal collagen tropocollagen
Step 6 in the production of collagen crosslink tropocollagen molecules to make collagen fibrils
Steroioids inhibit collagen synthesis
Osteogensis Imperfecta is an abnormal
Type I collagen
or defect in
Osteogensis Imperfecta Autosomal Dominant
Osteogensis Imperfecta Hallmark blue sclera
Ehlers Danlos Syndrome Hallmark hyperjointibility
Kidney disease + Deafness + Eye
Alport Syndrome
problems
Alport Syndrome defecent in type IV
Cant see, Cant pee, Cant hear high C Alport Syndrome
Marfan Syndrome hyperELASTICITY
Marfan Syndrome defect in fibrillin, which makes elastin
Marfan Syndrome Hallmark elasticity of skin
a-1 antitrypsin deficency defect elastin in alveoli
Hypothalamus: Nucleus Masterclock
suprachiasmatic nucleus
for Circadian Rhythm
Hypothalamus: Nucleus making
paraventricular nucleus
oxytocin
Hypothalamus: Nucleus secretes ADH supraoptic nucleus
Hypothalamus: Nucleus Dealing with
dorsomedial nucleus
Obesity + Savage behavior
Hypothalamus: Nucleus inhibited by
lateral nucleus
leptin
Hypothalamus: if Nucleus destructed
ventromedial nucleus
leads to obesity
Hypothalamus: Nucleus regulating
arcuate nucleus
dopamine and GHRH
Hypothalamus: Nucleus conserving
heat producing shivering in cold posterior nucleus
enviroments
Hypothalamus: destruction of this body
wernicke's encephalopathy
will lead to
Hypothalamus: Nucleus stimulating GI dorsomedial nucleus
Posterior Pituitary otherwise known as neurohyphysis
Posterior Pituitary is derived from
neuroectoderm
which germ layer
Hypothalamus: Nucleus receives input
suprachiasmatic nucleus
from retina
Hypothalamus: Savage behavior +
dorsomedial nucleus
obesity from stimulation
Hypothalamus: Savage behavior +
ventromedial nucleus
obesity from destruction
Hypothalamus: Stimulation leads to
eating and destruction further leading lateral nucleus
to starvation
Hypothalamus: Regulates release of
preoptic nucleus
LH and FSH
Hypothalamus: Destruction results in
supraoptic nucleus
neurogenic diabetes inspidius
Hypothalamus: releases hormones
arcuate nucleus
affecting the anterior pituitary
Hypothalamus: which nucleus
lateral nucleus
regulates appetite
Neuro: decreased GABA + serotonin
BUT increased norepinephrine is seen anxiety disorders
in
Sleep stages: Stage N1 theta waves
Sleep stages: Stage N2 bruxism (teeth grinding) sleep spindles and K complexes
Sleep stages: Stage N3 delta waves sleep walking bed wetting
Sleep stages: REM beta waves lose muscle tone
Sleep stages: Relaxation alpha waves
Sleep stages: Rx decreases bed wetting
imipramine
(nocturnal enuresis)
Sleep stages: Rx MOA of impramine TCA
Sleep stages: Desmopressin (DDAVP)
decreasing urination
can aslo be used as Rx for
Sleep stages: Rx indomethacin
renal blood floow
decreases
Sleep Rxs: this med gives vivid dreams
and should not be used for more than 3 melatonin
months
Sleep Rxs: herbal remedy found OTC valerian
Sleep Rxs: first-line treatment for
anti-histamines
difficulty sleeping
Sleep Rxs: this med increases risk of
Trazodone
priaprism
Sleep Rxs: this med increases REM
Trazodone
sleep
Sleep Rxs: TCAs such as amitriptyline,
arrhythmais (obtain EKG prior to use)
doxepin increases cardiac risk of
Sleep Rxs: ADDICTIVE med that
benzodiazepines
should only be used short-term
Sleep Rxs: Popular meds acting on
zolpidem + zalepon
benZo receptors
Sleep Rxs: only med per FDA
eszopiclone
approved for long term
Sleep Rxs: nonaddictive med because
it works on melatonin receptors rather ramelteon
than GABA reeptors
Narcolepsy Rxs: first-line treatment modafinil
Cataplexy Rx vanlafaxine, fluoxetine, or atomoxetine
Sleep Rxs: GHB can assist in sleep and
cataplexy
reduce
Brain embryology: Forebrain develops
telencephalon -> cerebral hemispheres + diencephalon ->thalamus
into
Brain embryology: Midbrain develops
mesencephalon -> midbrain
into
Brain embryology: Hindbrain develops metencephalon + myelencephalon -> cerebellum, pons, and
into medulla
Brain embryology: NTD avoided with
sonogram + quadruple screening
routine
Brain embryology: NTD indicative
elevated
when alpha fetoprotein levels are
Neuro Embyology: NTD defect
increased
indicative with AFP levels are
Neuro Embyology: AFP levels when
down's syndrome
decreased are indicative of
Neuro Embyology: Meningcele is a
meninges
herniation JUST of the
Neuro Embyology: Myelomeningocele
spinal cord + meninges
occurs with hernation of both
Neuro Embyology: No brain tissue
separating amniotic fluid in the brain is anencephaly
indicative of
Neuro Embyology: Holoprosencephaly
of the brain fail to separate
occurs when the hemispheres
Neuro Embyology: Cleft palate
holoprosencephaly
together with cyclopia are indicative of
Neuro Embyology: Sonic Hedgehod
holoprosencephaly
Gene mutations will be seen in patients
with
Neuro Embyology: Posterior fossa
Chiari Malformations
(hindbrain) abnormalities are seen in
Neuro Embyology: Chiari
Malformations occurs when forman magnum
cerebellum herniates download through
Neuro Embyology: Syringomyelia is
canal in spinal cord
an enlargement of the central
Neuro Embyology: Compression of the
spinothalamic tract occurs and is syringomyelia
referred to
Neuro Embyology: HALLMARK:
Cape-like, bilateral loss of pain and upper extremities
temperature sensation are seen in
Neuro Embyology: HALLMARK:
weakness and atrophy of hands
Anterior horn damage produces
Neuro Embyology: Spinal cord trauma
syringomyelia
over time can produce
Neuro Embyology: Herniation of
Chiari I Malformation
cerebellar tonsils are seen in
Neuro Embyology: BOTH herniation
Chiari II Malformation
of cerebellar tonsils and vermis
Neuro Embyology: Hydrocephalus can
Chiari II Malformation
be seen in patients with
Neuro Embyology: Stenosis of
aqueduct caused by a Chiari II
hydrocephalus
malformations along with which other
abnormality
Neuro Embyology: Thoracolumbar
Chiari II Malformation
Myelomeningocele is associated with
Neuro Embyology: Second Posterior
Dandy-Walker Syndrome
Fossa malformation
Neuro Embyology: enlarged posterior
Dandy-Walker syndrome
fossa is indicative of
Neuro Embyology: cerebellar vermis
Dandy-Walker syndrome
fails to develop is indicative of
Neuro Embyology: Dilation of the 4th
ventricle is associated with
Dandy-Walker syndrome
hydrocephalus indicative of which
diagnosis
Neuro Embyology: Brachial Apparatus
six; 6
is composed of how many tissues
Neuro Embyology: Bracial Apparatus
Mesoderm
arch is derived from
Neuro Embyology: Bracial Apparatus
Ectoderm
clef is derived from the
Neuro Embyology: Bracial Apparatus
Endoderm
pouch is derived from the
Neuro Embyology: Bracial Apparatus
Middle ear cavity Eutachian tubes Mastoid air cells
Pouch #1 will give rise to
Neuro Embyology: Bracial Apparatus
epithelial lining of the tonsils
Pouch #2 will give rise to
Neuro Embyology: Bracial Apparatus
inferior parathyroid gland + thymus
Pouch #3 will give rise to
Neuro Embyology: Bracial Apparatus
superior parathyroid gland
Pouch #4 will give rise to
Neuro Embyology: DiGeorge
Syndrome occurs when abnormal 3rd and 4th brachial pouches
development
Neuro Embyology: Pt. with absent
DiGeorge Syndrome
thymus and parathyroid glands
Neuro Embyology: Hallmark Triad in
Absent thymus + hypocalcemia + T-cell deficiency
DiGeorge Syndrome
Neuro Embyology: Brachial Arches
Mesoderm derivatives
are
Neuro Embyology: Bracial Arch #1
Cartilage + Muscle + Nerve
produces CMN, which are
Neuro Embyology: Treacher Collins Series of facial abnormalities
Syndrome
Neuro Embryology: HALLMARK of
Abnormal mandible + malleus
Treacher Collins Syndrome includes
Neuro Embryology: Brachial Arch #2 Stapes + Styloid Process + Stylohyoid ligament + lesser horn of
produces S structures such as the hyoid
Neuro Embryology: Hemispatial
Parietal lobe (non dominant)
neglect indicates a lesion in the
Neuro Embryology: Pt. neglects half of
Hemispatial neglect
a side of their body as nonexistent
Medical Term used for Crossed-Eyed
Strabismus or tropia
pt.
Golgi Apparatus modifies with amino
S.A.T. Serine, Asparagine, Theronine
acids
Astrocyte foot processes, basement
membrane, capillary lumen are the basement membrane
components of the
red nucleus, medial meniscus, superior colliculus, cerebral
Midbrain is composed of aqueduct, medical geniculate body, spinothalamic tract,
corticobulbar tract, corticospinal tract, substantia nigra
Inferior Colliculus is located in the caudal midbrain
Superior Colliculus is located in the rostral midbrain
CN located in the midbrain Oculomotor and Trochlear Nerves
Pons contains the abducens (CNVI) nerve
MLF is found in the pons
Medial Lemnicus travels done the medial aspect of the pons
Corticospinal tract is located in the medial aspects of the pons
Lateral part of pons is supplied by AICA
Cranial nerves will __________ as you
increase in number
go down the brainstem
hypoglossal nerve (CN XII), medial lemniscus, and meduallry
Medulla medial aspect contains
pyramid
Anterior Spinal Artery supplies the medial aspect of the medulla
PICA supplies the lateral aspect of medulla
Lateral aspect of medulla is composed vestibular (CN VIII) nuclei, inferior cerebellar peduncle, spinal
of trigeminal tract and nucleus and lateral spinothalamic tract
Spinal Tracts: Dorsal Columns are
fasiculus cutaneous and fasciculus gracilis
composed of
Spinal Tracts: Dorsal Columns
ascend
decussate in the medulla to then
Spinal Tracts: Dorsal Columns become
medial lemniscus
what
Spinal Tracts: Medial Lemniscus goes
VPL of thalamus
into the
Spinal Tracts: VPL is located in the thalamus
Spinal Tracts: Dorsal Columns work
contralateral
on which side
VPL is the _____ port of the thalamus sensory
Spinothalamic tract decussates at the anterior white commisure
Spinothalmic tract ascends contralateraly
Which tract travels contralaterally in
spinothalamic
the brainstem
Information to the Corticospinal Tract
internal capsule
comes from
Spinothalamic tract ascends as
desends
Corticospinal Tract
Norepinephrine is increase in anxiety
mania
and
Norepinephrine is decreased in depressionb
Raphe Nucleus is the center of serotonin
In depression, serotonin is decreased
The levels of serotonin are decreased
anxiety
in
CPK, Adolase,and Myoglobin are osteomarkers
This condition prevents thymidine xeroderma pigmentosa
repair dimers from UV damages
Dexamethasone Suppression Test
Dopamine levels in Schizophrenia increased
Dopamine levels in Depression decreased
Dopamine levels in Parkinson's decreased
Microglia HALLMARK seen in HIV multi-nucleated giant cells
Bitemporal Temporal Lobe location of HSV encephalitis
Decreased levels of Vitamin B6 lead to seizures
GABA is decreased n conditions such
huntington's
as anxiety and
The nucleus acubens houses
GABA
neuroinhibitors such as
Injury to axillary nerve results for
neck of the humerus
damaged to the surgical
P. Circumflex Humeral Artery supplies
surgical neck of the humerus
the
Perforin + Granzyme B is released as a
cytotoxic T cells and NK cells during bacterial infection
result of
Clathrin works in the (hint: starts with
cytosol
a C)
Process of receptor mediated
clathrin is activated which activates adaptins
endocytosis
Center of Huntington's disease caudate + putamen
Receptor locations: Tyrosine Kinase plasma membrane
Receptor locations: G-protein coupled cytosol
Receptor locations: steroids cytoplasm
Retro peritoneal Organs S.A.D.P.U.C.K.E.R
Destruction of Alpha 3 chain of Type
Goodpasture Syndrome
IV Collage results in
Hypotension, Tachycardia, and Cool
hypovolemic shock
Extremities are sx of
How to diagnose lactose intolerance pH of stool
MOA inhibit substance flow across cell
membranes thus inhibiting primary PPI
active transport
Zileuton, Montelukast, Zafirlukast are
asthma
agents used to treat
Transplant rejections primarily occur
antibodies against antigens
due to preformed
Langerhans Histocytosis HALLMARK birbeck granules
Tennis racket shaped cytoplasmic
langerhans histocytosis
organelles
Encapsulated bacteria Even Some Pretty Nasty Killers Have Shiny Bodies
The most injure organ in blunt trauma
spleen
(not a foregut structure)
Bilateral temporal visual deficit seen in pituitary adenomas
Payer Patches seen in small intestine (ileum)
Rx producing antibody against CD20 rituximab
Causative agent of barking cough croup
Meningiomas found in parietal lobe
Meningiomas produce lower limb
hemineglectp
sensory loss and
inability to consume food; pt consumes items that are non-food
PICA Disorder
such as ice, hair, paper
Rx for phobia benzodiapine
Agents in innate immunty neutrophils, dendrites, macrophages, and complement
IL-1 (fever), IL-2 (T-cells), IL-3 (bone marrow), IL-4 (IgE + IgG
Hot T-Bone Steak
production), IL-5 (IgA + Eosinophils production)
Active Immunity involves b + t cells
IFNs released by virus infected cells IFN-alpha and IFN-beta
Rx immunosuppreant inhibiting
calcineurin, production of IL-22, and Cyclosporin
T-Cell
HALLMARK in Crohn's Disease transmural inflammation
HALLMARK Ground Glass
NRDS (Neonatal Respiratory Distress Syndrome)
Apperance of lungs
HALLMARK Air Space and
NRDS (Neonatal Respiratory Distress Syndrome)
interstitial opacities in lungs
Antidote for acetominophen overdose N-Acetylcystine
N-Acetylcystine leaves disulfide bonds
muscous glycoproteins
within
Extension seen in obstructive lung
expiratory phase (FEV down)
disease
Asthmatic antigen cross linking IgE on pre-sensitized mast cells
Levels in COPD FEV down; FVC same
Restrictive Lung Disease leads to
TLC; normal FEV1:FVC ratio
decreased
Side effect of Tamoxifen endometrial hyperplasia
Disorder in which pt. complains of
somatization
symptoms that are not even there
Thrombocytopenia can be caused by
heparin
this agent (hint: thrombolytic)
Thrombocytopenia can be caused by
beta lactam antibiotics
this agent (hint: antibiotic)
Thrombocytopenia can be caused by
vancomycin
this agent (hint: anti bacterial)
Splenomegaly seen in this kissing
EBV
disease
Thrombocytopenia can be caused by
this agent (hint: used to treat gram + linezolid
bacteria)
Rx used in treatment of CMV foscarnet
Foscarnet does not require intracelullar activation
Arsenie Poisioning is treated with this
dimercaprol
agent
Poision Ivy is mostly encountered in hikers, forests, and wooded areas
Neisseria Gonnorhea will elevate neutrophils (think N in neiserria)
Rx used treat general protozas metrodiazole
Rx used treat trophozoites metrodiazole + tinidazole
Rx used treat cysts iodoquinol + paromomycin
Rx swish and swallow method us used
oral candidasis
with nyacin to treat
Erythrocytosis in men should be <52%
Erythrocytosis in women should be <48%
Activation of Leukotriene C4 is found
asthma
in
HALLMARK for asthma histology charcot-leyden crystals
Pharm: direct acting is an agonist
Pharm: inhibition of action is a anatagonist
Organophosphate Poisioning is seen
farmers
primarily in
Adenosine Deaminase Deficency
SCID
results in this condition
A-1 Adrenergic causing adverse
terazosin + doxazosin
reaction of orthostatis hypotension
Raymonds phenomenon leads to cerebral edema
Asprin given to children younger than
raymond's phenomenon
the age of 12 will results in
Disease arsing from deficiency in
i-cell
mannose-6-phosphate
Sx clouded cornea, restruct joint
movement, and course facial features i-cell disease
seen in
This bacteria's polysaccharies capsule
step. pneumo
gives it it's virulence
DNA mismatch repair will cause hereditary nonpolyposis colorectal cancer
Rx agent used for motion sickness scopolamine
HALLMARK fishy odor with thin
bacterial vaginosis
discharge indicative of
HALLMARK frothy odor with
trichamonasis
green/white discharge indicative of
HALLMARK cottage cheese discharge
candidasis vaginosis
indicative of
Water Deprivation Test is done to pt.
diabetes insipidus
who is suspected to have
Desmopressin (ADH) Challenge will
central or nephrogenic
indicate whether the dx is
This syndrome's onset may be due to
cushings syndrome
an adverse reaction to steroids
Amyloidosis stains congo red
HALLMARK apple green bigeringate amyloidosis
HALLMARK histology showing
multiple myeloma
rouleaux (poker chips like)
Adverse Reactions: Anti-hypertensive
reflex tachycardia; useful for pt. with bradycardia
Nifedipine
Adverse Reactions: Anti-hypertensive
prolong PR interval
Beta Blocker
Olgiodendrocyte destruction seen in multiple sclerosis
Retro peritoneal organ most injured in
pancreas
car accidents
Abnormalty seen in hereditary
RBC membrane cytoskeleton
spherocytosis
HALLMARK Interstitial Fibrosis honeycomb lung on x-ray
Honeycomb lung on x-ray is indicative
intersistial fibrosis
of
Rx acetylcysteine decreases levels of mucous
1st line indicator in diabetic
albumin
nephropathy
Generalized Anxiety Disorders are
buspirone
treated with
Crackles on lung auscultation mostly
pulmonary edema
heard when pt has
HALLMARK Asbestosis dumb bells appearance
Asbestosis histology will show enlongated structures with clubbed ends
Myocardial Infarction will produce prostagladins
Prostaglandins produced in MI will inhibit platelet aggregation
Cytochrome oxidase inducers include
Rapid correction of
HYPONATUREMIA will lead to this central pontine myelinolysis
condition
MRI shows increased signal intensity
rapid correcton of hyponatermia
to the pons indicating
Sx of central pontine myelinolysis paralysis, dysarthria, dysphagia, diplopia, loss of consciousness
Low sodium levels need to be correct
slowly
very
Retinoic acid (vitmain A derivative)
HOX gene expression
alters
Which antibiotics are potentially
F.A.T.; Fluroquinolones, Aminoglycosides, and Tertatogens
teratogenic
Rx teratogenic in pregnant women MTX, Statins, Warfarin, Isorebinoin, DES, Thalidomide
Limb buds begin to form at week 21
Fetal movement begins at week 8
By which week does the mother begin
8
to experience fetal movement
Which vitamin should not be
supplemented in large amounts during vitamin A; alters HOX gene expression
pregnancy
Nuclear Localization Signals: Amino
rich in L.A.P: lysine, arginine and proline
acids
Nuclear Localization Signals: Essential
component of protein bound for or histones
residing in the nucleus
Where are histones located nucleus
Nuclear Localization Signals: Nuclear
pores recognize these signals and nucleus via ATPase
transport proteins into the
ATPase is the signal of transportation
nucleus
of proteins and into the
A single AA mutation may prevent nuclear transport
A deficiency leading to I-Cell Disease mannose phophorylation
Mannose phosphorylation secretion out
cell instead of into lysosomes for degradation
of
I-Cell Disease leads to death by the
8
childhood age of
Chaperones assist in folding and
ER and Golgi, etc
transport of polypeptides in the
Stress Chaperones rescue shock-
misfolding
stressed proteins from
If folding is not successful, the
degradation of damaged protein
chaperones facilitate
Degradation of damaged proteins
ubiquitin
involves
This is also known as a heat shock
ubiquitin
protein
Peroxisomes provide beta oxidation of
fatty and branched chain fatty acids
very long chain
Which cyclin-CDK complexes assist in
the progression from G1 phase to S Cyclin D and Cycline E; CDK 4 + CDK 2, respectively
phase
Which cyclin-CDK complexes assist in
the progression from G2 phase to M Cycline A and Cycline B; CDK 2 + CDK 1, respectively
phase
What molecule does the Golgi add to
proteins in order to direct the proteins
to the lysosomes
What are the different methods that a
cell uses to break down proteins
What is the most common cause of
fetal alcohol syndrome
intellectual disability in infants
Structural component of connective
tissue L.E.F. (leukocytes, endothelium, Vimentin
fibroblasts)
Desmin is the intermediate filament of
muscle (smooth, skeletal, and heart)
structural component
Cytokeratin is the intermediate
epithelial cells
filament of structural component
Glial fibrillary acid proteins is the
intermediate filament of structural astrocytes, schwann cells, other neuroglia
component
Peripherin is the intermediate filament
neurons
of structural component
Neurofilaments is the intermediate
axons within neurons
filament of structural component
Nuclear lamins is the intermediate
nuclear envelope and DNA within
filament of structural component
Which drugs act on microtubules (the
thiabendazole + mebendazole + griseofulvin + vincrestine +
microtuble growth voiding pure
pacixtel + vinblastine
chemicals)
What are the defects in Kartagner
Syndrome aka immotile ciliary primary ciliary dyskinesia
syndrome
During what week of fetal
development does organogensis take
place
What molecule provide the structural
framework for DNA and nuclear nuclear lamins
envelope
The first step in the signaling cascade
that it initiated by tyrosine kinase autophosphorylation
receptors is
What type of protein is PDGF single-pass transmembrane protein
Insulin and Insulin Growth Factor-1
2 alpha subunits and 2b subunits
receptors
Which arachiodonic acid product
prostaglandins
causes: increased bronchial tone
Which arachiodonic acid product
PGF-12
causes: decreased bronchial tone
Which arachiodonic acid product
thromboxane
causes: increased platelet aggregation
Which arachiodonic acid product
PG-12
causes: decreased platelet aggregation
Which arachiodonic acid product
prostaglandins
causes: increased uterine tone
Which arachiodonic acid product
PG-12
causes: decreased uterine tone
Which arachiodonic acid product
thromboxane
causes: increased vascular tone
Which arachiodonic acid product
PG-12
causes: decreased vascular tone
What are the two most abundant
phospholipids and cholesterol
substances in the plasma membrane
What drugs act on the arachidonic acid
Cycloxygenase (NSAIDs)
pathway
What are the stages that an embroy
goes through between conception and cleavage, blastula, and gastrulation stages
the development of an inner cell mass
The blastula eventually organizes itself
inner cell mass and outer trophoblast
into two layers
Trophoblast will become the placenta
Which cell types are constantly
regenerating themselves due to an
GI, bone marrow, hair follicles, nails
absence of the G0 phase and a short G1
phase
BCL-2 is major anti-apoptotic
mitochondrial permeability
regulator of
Any DNA damage or apoptotic sign
Bax (pro-apoptotic)
will activate
Bax creates channels in mitochondrial membrane
Cytochrome C moves from the mitochondria and into cytosol
Cytochrome C activates caspases
The primary initator of apoptosis caspases
Name death receptors TNF-a + Fas ligand
Cytotoxic T-Cells recognize foreign or infected cells
Cytotoxic T-Cells release which agents perforin and granenzyme B
Mechanism of Cell Injury: influx of mitochondrial permeability and activate phospholipases, proteases,
calcium will increase endonucleases and ATPase
Mechanism of Cell Injury:
cell damage through membrane lipid peroxidation, protein
accumulation of oxygen-derived free
modification and DNA breakage
radicals
Radiation Exposure may cause cell injury; DNA breakage, protein modification
What histology features may be seen in
infiltrate only if cause is alcohol is steatohepatitis
apoptotic liver cells
histologic changes including steatosis, inflammation, ballooned
Apoptotic liver cells under the
hepatocytes, MalloryDenk bodies, apoptotic hepatocytes, and
microscope will show
fibrosis or cirrhosis
What damaging events can cause mitochondria swell, lysosomes swell, damage to plasma
irreversible cell injury membrane and lysosomal membranes leads to enzyme leakage
What substances do cytotoxic T cells
and NK cells use to induce apoptosis in perforin and granenzyme B
the cells infected with a virus
Redox reaction may cause cell injury; DNA breakage, protein modification
What cellular enzymes are responsible
superoxide dismutase
for handling oxygen free radicals
Transition metals may cause cell injury; DNA breakage, protein modification
Nitric Oxide may cause cell injury; DNA breakage, protein modification
Leukocyte Oxidative Burst may cause cell injury; DNA breakage, protein modification
Reperfusion injury may cause cell injury; DNA breakage, protein modification
Acute inflammation: release of
neutrophils, histamine, bradykinin, serotonin
mediators
Acute inflammation: mediators in
prostagladins and nitrix oxide
vasodilation
Tissue remodeling by
C5a
metalloproteinses contains
Polymyalgia rhematica laboratory
elevated ESR
indication
C-Reactive Protein in the acute phase
liver
is synthesized by the
Oposonization of bacteria and
C-Reactive Protein
activation of complement is done by
Temporal arteritis laboratory indication elevated ESR
Acute inflammation: mediators
bradykinin, serotonin, histamin, C3a + C5a, Leukotrienes, PAF,
increased vascular permeability fluid
oxygen free radicals
exudation
Can C-Reative Protein be lowered by
yes
smoking cessation
Maliganancy laboratory indication elevated ESR
Elevations in this marker is a strong
predictor or MI, Stoke, PAD, and C-Reactive Protein
sudden cardiac death
True o False: C-Reactive Protein can
true
be lowered by exercise/weight loss
Which metals are known to faciltate
metallprotienases
the generation of oxygen free radicals
Which tumor suppressor proteins
prevent the progression of the cell in S p53 and Rb
phase
Cutaneous wound healing: week-
collagen production (Type III and Type I)
months
Cutaneous wound healing: 0-3 hours hemorrhage and clotting
Cutaneous wound healing: 1-3 days macrophage infiltration, granulation tissue, epithelization
Cutaneous wound healing: 12-24 hours acute inflammation (PMNs)
Infection, inflammation (e.g.
elevated ESR
osteomyelitis) laboratory indication
Ehlers-Danlos Syndrome sx (3-D
Type III collagen deficiency
pneumonic)
Diseases activity in RA and SLE elevated ESR
Which AA are found in large
proline + lysine
concentrations in collagen
What is the role of Vitamin C in
hydroxylation of proline and lysine
collagen
Sx hyperflexible koints, archnodactyly,
aortic dissection, lens dislocation are ehlers danlos
all indicative of
Sx hereditary nephritis, cataracts,
sensorineural hearing loss
What is the underlying dysfunction in Defect in lysosomal trafficking regulator gene (LYST);
Chediak-Higashi syndrome Microtubule dysfunction in phagosome-lysosome fusion;
Chediak-Higashi Syndrome is an autosomal recessive
How does having a high cholesterol
separate the phospholipids so that the fatty acid chains can't come
content in the plasma membrane affect
together and cyrstallize.
its function
A man working out at the gym building
hypertrophy
muscle would be considered
What can happen to the cells of the
metaplasia of simple squamous to columnar epithelium leading to
lower espohagus in response to chronic
GERD or Barret's Esophagus
acid reflux
aging or "wear-and-tear" pigments, found in the liver, kidney,
What is lipofuscin granule
heart muscle, retina, adrenals, nerve cells, and ganglion cells
Four major dopaminergic pathways mesocortical pathway
Four major dopaminergic pathways mesolimbic pathway
Four major dopaminergic pathways nigrostriatial pathway
Four major dopaminergic pathways tuberoinfundibular pathway
Which nervous system cell matches the olgiodendrites
follow description: fried egg under
histology staining
Which nervous system cell matches the
follow description: form
mircroglia
multinucleated giant cells in the CNS
when infected with HIV
Which nervous system cell matches the
follow description: myelinates mutiple
CNS axons
Which nervous system cell matches the
follow description: myelinates one schwann cells
PNS axon
Which nervous system cell matches the
follow description: damaged in schwann cells
Guillain-Barre syndrome
Which nervous system cell matches the
follow description: damaged in ogliodendrites
multiple sclerosis
Which nervous system cell matches the
follow description: macrophase of the microglia
CNS
Which nervous system cell matches the
follow description: cells of the blood-
brain barrier
What is the main inhibitory
GABA
neurotransmitter of the CNS
In which diseases is GABA altered huntington's and anxiet
What organelle becomes hypertrophied
in hepatocytes with chronic smooth endoplasmic reticulum (SER)
phenobarbital use?
What enzyme mitigates the aging
effects of cellular division by telomerase
maintaining chromosomal right
What is currently known as the most
effective way of prolonging life span
Which area of the hypothalamus
paraventricular nucleus
regulate the autonomic nervous system
Which neurotransmitter have altered
GABA
levels in anxiety disorder
Which collagen is typically deficient in
Type III
Ehlers-Danlos syndrome
Which collagen is typically deficient in
Type I
osteogensis imperfecta
Which drug is used to shorten Stage
imipramine
N3 sleep
What is the sleep pattern in a patient
with narcolepsy
In what stage of sleep are night terrors
Stage N3
found
What arachidonic acid product has
COX-1 + COX-2
actions that oppose that of prostacyclin
What brain structure is responsible for
extraocular movements during REM paramedian pontine reticular formation/conjugate gaze center
sleep
What two nerves are tested with the
vagus + glossopharyngeal
gag reflax
Unilateral facial drooping involving bell's palsy; LMN facial nerve (CN VII) palsy; UMN lesions spare
the forehead the forehead
What organelle and cytochrome are
particularly important in intrinsic mitochondria + cytochrome C
apoptosis
During what sleep stage would a man
have variable blood pressure,
REM
penile/clitoral tumescence, and
variable EEG
A 19-year-old patient presents with a
furnuncle on his philtrum, and the
cavernous sinus becomes infected.
What neurological deficits might you
see in this patient?
Which areas of the hypothalamus
regulate the autonomic nervous Anterior hypothalamus : Parasympathetic Posterior: Sympathetic
system?
Cranial nerve 1 (Olfactory) Site of exit
Cribiform Plate Smell Cribiform plate fracture or Kallmann
from the skull? Function? Lesions?
syndrome Smell something
How to test the function?
Cranial nerve 2 (Optic) Site of exit
from the skull? Function? How to test Optic canal; [Sight] [Smell] [Eye chart]
the function?
Superior orbital fissure Parasympathetic: Cillary and sphincter
Cranial nerve 3 (Occulomotor) Site of
muscles Innv. MR, SR, IR, IO (Extraoccular m.)
exit from the skull? Function? Lesions?
Transtentorial/Uncal Herniation, Diabetes, Weber syndrome
How to test the function?
Follow my finger, pupillary light reflex, and convergence
Form of stroke that causes occlusion of PCA. Causes contralateral
Describe Weger Syndrome
hemiparesis and CN 3 palsy
Cranial nerve 4 (Trochlear) Site of exit
Superior orbital fissure Innv. SO Head trauma Move eye by
from the skull? Function? Lesions?
following finger
How to test the function?
Cranial nerve 5 (Trigeminal)-V1 Site
V1 (Opthalmic): Superior orbital fissure Sensory for Medial nose
of exit from the skull? Function?
and forehead Path: Trigeminal neuralgia Test: Facial sensation
Lesions? How to test the function?
Foramen ovale Motor fxn. of muscle of mastication (open and
Cranial nerve 5-V3 (Mandibular) Site
close jaw) Sensory: lower lip, lateral face, lower border of
of exit from the skull? Function?
mandible Lesion: Jaw will deviate towards the side of lesion when
Lesions? How to test the function?
opening Test: facial sensation, open jaw
Which muscle opens the jaw Lateral Pterygoid
Which muscle closes the jaw Masseter, Temporalis, Medial pterygoi
Cranial nerve 6 (Abducens) Site of exit Superior orbital fissure Innv. lateral rectus Medial inferior pontine
from the skull? Function? Lesions? syndrome (contralateral hemiparesis and loss of tactile and
How to test the function? vibrations + lesion of CN 6) Test with follow my finger
Internal acoustic meatus Parasymp: Lacrimal, submandibular, and
Cranial nerve 7 (Facial) Site of exit
sublingual glands Innv. Facial expression, stapedius, stylohyoid,
from the skull? Function? Lesions?
posterior belly of digastric m. Taste anterior 2/3 of tongue Bells
How to test the function?
Palsy (includes muscle of forehead) Test:Wrink
Name the 5 branches of the Facial
Temporal Zygomatic Buccal Marginal mandibular Cervical
Nerve (CN VII)
Cranial nerve 8 (Vesitbulacocchlear) Internal acoustic meatus Equilibrium and Hearing Acoustic
Site of exit from the skull? Function? schwannoma (vertigo, nystagmus, nausea and vomiting) Hearing
Lesions? How to test the function? and nystagmus
Jugular foramen Parasym: Parotid gland, stylopharyngeus m
Cranial nerve 9 (Glossopharynageal)
Sensory: pharynx, middle ear, auditory tube, carotid body and
Site of exit from the skull? Function?
sinus, external ear, posterior 1/3 of tongue Lesions: PICA infarct
Lesions? How to test the function?
Test: Gag relfex
Jugular foramen Symp: body viscera, laryngeal & pharyngeal m.
Cranial nerve 10 (Vagus) Site of exit
Sensory: Trachea, External ear, viscera of esophagus, epiglottis
from the skull? Function? Lesions?
Lesion: Thyroidectomy, PICA infarct Test: Gag reflex and saying
How to test the function?
ahh (elevates palate) Uvula deviates away from lesion
Cranial nerve 11 (Accessory) Site of
Jugular foramen Innv. SCM and Trapezius M. Lesion: PICA
exit from the skull? Function? Lesions?
infarct Test: Turn head and shrug shoulders
How to test the function?
Cranial nerve 12 (Hypoglossal) Site of
Hypoglossal canal Intrinsic tongue muscles Lesions: Anterior
exit from the skull? Function? Lesions?
spinal a. infarct Tongue protudes towards lesion Test: Lalalala
How to test the function?
Corneal Reflex How to test? Normal Touch cornea with cotton N: Blink Afferent: V1 (opthalmic)
reflex Afferent: Efferent: Efferent: CN 7 (temporal branch)-closes eye (orbicularis oculi m.)
Lacrimal Reflex How to test? Normal Touch cornea with cotton N: Lacrimate Afferent: V1 (opthalmic)
reflex Afferent: Efferent: Efferent: CN 7 (temporal branch)
Jaw Jerk Reflex How to test? Normal Tap on chin Normal (no response) Afferent: V3 (Mandibular)
reflex Afferent: Efferent: Efferent: V3 (Masseter) UMN lesion if different
Pupillary Reflex How to test? Normal Shine light in eye Normal: Pupillary constriction Afferent: Optic
reflex Afferent: Efferent: nerve Efferent: Parasymp CN 3
Gag Reflex How to test? Normal reflex Stick tongue depressor in mouth and say ahhh Normal: Gags
Afferent: Efferent: Afferent: CN 9 Efferent: CN 10
Midbrain contains which CN nuclei? CN 3 and 4
Pons contains which CN nuclei? CN 5-8
Medulla contains which CN nuclei CN 9, 10, 12
Spinal cord contains which CN nuclei? CN 11
What does the Pineal body contain melatonin secretion, circadian rhythm
What does the Superior colliculus
conjugate vertical gaze center
contain
What does the Inferior colliculus
auditory information
contain
What nerves run through Cavernous
CN 3, 4, 6, branches (V1 and V2) of CN 5
sinus
What are other structures that run
Internal Carotid a optic chiasm pituitary gland sphenoidal sinuses
through Cavernous Sinus
The reticular activating system include Reticular formation Mesencephalic nucleus Thalamic intralaminar
which areas nucleus Dorsal hypothalamus Tegmentum
The reticular activating system
arousal and sleep awakening
function
The Vagal nuclei are nucleus solitarius nucleus ambiguus dorsal motor nucleus
Nucleus solitarius Visceral sensory info: Taste Baroreceptors gut
Nucleus solitarius
distension (CN 7,9,10)
Nucleus Ambiguus Motor innv. Pharynx Larynx Upper Esophagus (CN 9,10,11)
Dorsal Motor Nucleus Autonomic (Parasymp) info: Heart Lungs Upper GI (CN 10)
Which sympathetic chain starts from
what spinal cord segment and ends at T1-L3
what segment
Horner Syndrome will cause sympathetic denervation of the face
Ptosis (drooping of eyelid) Anhidrosis (absence of sweating) and
Describe Horner syndrome symptoms
flushing (rubor) of affected side Miosis (constriction of pupils)
Spinal cord lesion above T1 assc.(superior cervical ganglion).
What is caused by Horner syndrome Pancoast tumor (pre-ganglionic) Brown-Sequard syndrome Late
stage syringomyelia
Open Jaw: Temporalis, medial pterygoid cLose Jaw: Lateral
What are the muscles of mastication
pterygoid Masseter m.
Unilateral facial drooping involving
Bell's Palsy
the forehead
Sx Ptosis, miosis and anhidrosis are
Horner Syndrome
indicative of
What G protein classes do alpha
1= Gq 2 =Gi
receptors stimulate?
What G protein classes do beta 1 =Gs 2 =Gs
receptors stimulate?
What G protein classes do muscarinic
M1 =Gq M2 =Gi M3 =Gq
receptors stimulate?
What G protein classes do
D2= Gs
dopaminergic receptors stimulate?
What is the treatment for
methotrexate vincristine vinblastine
choriocarcinoma
What is the treatment for AML cytarabine (arabinofuranosyl)
What is the treatment for CML imatinib
Which Rx prevents breast cancer tamoxifen
What is the treatment for breast cancer etopside, bleomycin, cisplatin, ifosfamide
What is the rate-limiting enzyme in
Purine: PPRP (phosphoribosyl pyrophosphate synthetase II)
purine synthesis?
What is the rate-limiting enzyme in
Pyrimidine: Carbamoyl phosphate synthetase II
pyrimidine synthesis?
Compares a group of people with the disease vs those without the
Case-Control study disease Looks for prior exposure or risk factor
Retrospective/Observational
Compares a group with a given exposure or risk factor to a group
Cohort Study without such exposure Can be prospective/retrospective Relative
risk
Collects data from a group of people to assess frequency of disease
Cross-sectional study (and related risk factor) at a particular point in time Disease
prevalence Can't estimate causality
Compares the frequency with which both monozygotic twins or
Twin Concordance Study
both dizygotic twins develop the same disease
In which phase of meiosis is a primary
oocyte arrested until just prior to Prior ovulation: Prophase of meiosis I
ovulation?
In performing a lumbar puncture to
Skin Subcutaneous fat Supraspinal ligament Interspinal ligament
obtain a sample of CSF, what
Ligamentum flavum Epidural space Dura Matter Subdural space
structures are pierced, starting with the
Arachnoid matter Subarachnoid space
most exterior?
What medications are used in the Endothelin receptor antagonists (Bosetan) MOA: Competitive
treatment of pulmonary hypertension antagonize endothelin-1 receptor PDE-5 inhibitor (Sildenafil)
MOA: Inhibit cGMP PDE5 and prolong vasodilatory effect of NO
Prostacyclin analogs (epoprostenol & iloprost) MOA: Prostacyclin
Does the notochord become the neural No, Notochord is derived from mesoderm; Notochord becomes
tube nucleus pulposus of IV disk
From where does the amygdala receive Inputs: Limbic cortex Neocortex of parietal, temporal, and
inputs occipital lobe
To where does the amygdala send
Output: Hypothalamus, Thalamus, Septum, Hippocampus
output
Narcoleptic sleep episodes begin with
Narcolepsy sleep episodes start at REM sleep
what stage of sleep
Awake: Beta waves Awake but relaxed: alpha waves Stage 1:
What are the different stages of normal Light sleep (Theta waves) Stage 2: Deeper sleep, Bruxism (Sleep
sleep spindles and K complexes) Stage 3: Deepest non-REM sleep
(delta-slow wave)
Which T cell type regulates the T-Helper Cells 2 -Augment humoral (Plasma cell) response
humoral response Generate IL-4 and IL-5
What is a reason why a woman might
have primary amenorrhea (hint: turner syndrome
congential)
Cystic cavity within spinal cord (C8-T1) Crossing anterior spinal
What is a syringomyelia
commissural fibers are damaged
What symptoms are commonly seen in Cape-like bilateral loss of pain and temp sensation in upper
patients with syringomyelia extremities (fine touch preserved)
What type of skin cancer is associated
Squamous cell carcinoma
with arsenic exposure in coal miners
Which one will be able to increase the
Phenylephrine
blood pressure of a hypotensive patient
D3 from sun exposure in skin. D2 ingested from plants. Both
What are the steps in the conversion of
converted to 25-OH in liver and to 1,25-(OH)2 (active form) in
vitamin D to its active form in the body
kidney
By what mechanism does vitamin D Increases absorption of dietary Ca2+ and Phosphate Increases
help prevent osteoporosis bone resorption -> Increased Ca2+ and Phosphate
What landmarks are used when placing Between the medial and lateral heads of the sternocleidomastoid
an internal jugular central venous muscle and lateral to the carotid artery in most cases
catheter
What is a reason why a woman might
have primary amenorrhea? imperfortate hymen
(hint:hymen)
What is a reason why a woman might
have primary amenorrhea? (hint: mullerian delayed agenesis
congential)
What can occur if a MAO inhibitor
Serotonin syndrome
(MAOI) is added to an SSRI
Treatment: Stop serotonergic drugs Benzodiazepine Supportive
Rx for Serotonin Syndrome
care Cyproheptadine (5-HT2 receptor antagonist)
Clavulanic acid, sulbactam, and
tazobactam aid penicillins in their
Beta-lactamase inhibitor
activity against bacteria through what
mechanism
What is the most common benign
pleomorphic adenoma
tumor of the salivary gland?
Warthin tumor: (papillary cystadenoma lymphomatosum) is a
What is a Warthin's tumor
benign cystic tumor with germinal centers
What cranial nerve goes through the
CN VII goes through parotid gland
parotid gland
Membranous glomerular diseases
Basement membrane Membranous nephropathy
involve thickening of what structure
Which bacteria are most commonly
Bacteria: Staph a. and Viridans strep
responsible for sialadenitis
What condition most commonly
Predisposes: sialolithiasis (stone obstruction of salivary gland)
predisposes a patient to sialadenitis
What important secretory products are
secreted from the following cells of the Gastrin
GI tract? (hint: G cells)
What important secretory products are
secreted from the following cells of the Cholecystokinin (CCK)
GI tract (hint: I cells)
What important secretory products are
Secretin
secreted from the following cells of the
GI tract? (hint: S cells)
What important secretory products are
secreted from the following cells of the Somatostatin
GI tract? (hint: D cells)
What important secretory products are
secreted from the following cells of the Gatric acid and IF
GI tract (hint: pariteal cells)
What important secretory products are
secreted from the following cells of the Pepsinogen
GI tract (hint: chief cells)
What is the antidote for warfarin
Warfarin overdose: Vitamin K, plasma (if active bleeding)
anticoagulation or warfarin overdose
What is the antidote for heparin
Heparin overdose: Protamine sulfate
overdose
Most common cause of neural tube
Folate Deficiency
defects
Most common cause of congenital
Alcohol
malformations in the US
Most common cause of congenital
Fetal Alcohol Syndrome
mental retardation in the US
Sx Hyperflexible joints,
arachnodactyly, aortic dissection, lens Marfan's Syndrome
dislocation
Sx Hereditary nephritis, cataracts,
Alport Syndrome
sensorineural hearing loss
Sx Ptosis, miosis, anhidrosis Horner's Syndrome
Amyloid deposits in gray matter of the
Senile plaques in Alzheimer's Disease
brain
HALLMARK Drooling farmer Organophosphate poisoning
Sx Inability to breastfeed, amenorrhea,
Sheehan's Syndrome
cold intolerance
Sx Infertility, galactorrhea, bitemporal
Prolactinoma
hemianopsia.
Most common causes of Cushing 1) Exogenous Steroids 2) Ectopic ACTH--small cell lung cancer
syndrome (4) 3) Cushing Disease--pituitary tumor 4) Adrenal Adenoma
Most common tumor of the adrenal
Adrenal Adenoma
gland
Most common tumor of the adrenal
Pheochromocytoma
medulla (in adults)
Most common tumor of the adrenal
Neuroblastoma
medulla (in kids)
Most common cause of primary
Adrenal Adenoma
hyperaldosteronism
Medical treatment for
Spironolactone or Eplerenone
hyperaldosteronism
Pheochromocytoma, medullary thyroid
MEN 2A
cancer, and hyperparathyroidism
Pheochromocytoma, medullary thyroid
MEN 2B
cancer, and mucosal neuromas
Adrenal disease associated with skin
Addison's Disease
hyperpigmentation
HTN, hypokalemia, metabolic acidosis Conn Syndrome
Most common thyroid cancer Papillary Carcinoma
Cold intolerance Hypothyroidism
Enlarged thyroid cells with ground-
Papillary Carcinoma of the thyroid
glass nuclei
Most common infections seen in Bugs producing catalase: Candida Aspergillus Staph aureus
chronic granulomatous disease Klebsiella E. coli
Eczema, recurrent URI, high serum
Hyper-IgE Syndrome aka Job's Syndrome
IgE
Large lysosomal vesicles in phagocytes Chediak-Higashi Syndrome
Dark purple nodules on the skin in an
Kaposi's Sarcoma
HIV infected patient
Large cells with owl's eye inclusions CMW
Treatment of CMV Ganciclovir
Most common opportunistic infection PCP
in HIV patients
Drug used to prevent Pneumocystis
TMP-SMX
pneumonia
Dysphagia, glossitis, and Fe deficiency
Plummer-Vinson Syndrome
anemia
Hematemesis with retching Mallory-Weiss Tear
Specialized columnar epithelium seen
Barrett Esophagus
in a biopsy from the distal esophagus
Biopsy of a patient with esophagitis
reveals large pink intranuclear
HSV
inclusions and host cell chromatin that
is pushed to the edge of the nucleus
Biopsy of a patient with esophagitis
reveals enlarged cells, intranuclear and
CMV
cytoplasmy inclusions, and a clear
perinuclear halo
An esophageal biopsy reveals lack of
ganglion cells between the inner and Achalasia
outer muscular layers
Biopsy of a mass in the parotid gland
reveals a double layer of columnar
Warthin's Tumor
epithelial cells resting on a dense
lymphoid stroma
Protrusion of the mucosa in the upper
Esophageal web
esophagus
Outpouching of all layers of the
Epiphrenic Diverticulum
esophagus found just above the LES
Basal cell hyperplasia, eosinophilia,
and elongation of the lamina propria Chronic GERD
papilla seen in biopsy of the esophagus
Goblet cells seen in the distal
Barrett Esophagus
esophagus
A PAS stain on a biopsy obtained from
a patient with esophagitis reveals Candida
hyphate organisms
Esophageal pouch found in the upper
Zenker Diverticulum
esophagus
Stomach biopsy reveals neutrophils
above the BM, loss of surface
Acute Gastritis
epithelium, and fibrin-containing
purulent exudate
Stomach biopsy reveals lymphoid
aggregates in the lamina propria,
Chronic Gastritis
columnar absorptive cells, and atrophy
of glandular structures
Diffuse thickening of gastric folds,
elevated serum gastrin levels, biopsy
Zollinger-Ellison Syndrome
reveals glandular hyperplasia without
foveolar hyperplasia
Mucin-filled cell with a peripheral
Signet Ring cells
nucleus
Most common type of stomach cancer Adenocarcinoma
Ovarian metastases from gastric cancer Krukenberg tumor
Gastric ulcerations and high gastrin
Zollinger-Ellison Syndrome
levels
Acute gastric ulcer associated with
Cushing Ulcer
elevated ICP or head trauma
Acute gastritis ulcer associated with
Curling Ulcer
severe burns
Painless jaundice Pancreatic Cancer
Most common cause of acute
Gallstones & alcohol
pancreatitis
Most common cause of chronic
Alcohol
pancreatitis
Small intestinal mucosa laden with
distended macrophages in the lamina
propria that are filled with PAS+ Whipple's Disease
granules and rod-shaped bacilli seen by
electron microscopy
Gluten Sensitivity Celiac Sprue
Total or subtotal atrophy of the small
bowel villi, plasma cells, and
lymphocytic infiltration into the lamina Celiac Sprue
propria and epithelium, and
hyperplasea/elongation of the crypts
Weight loss, diarrhea, arthritis, fever,
Whipple's Disease
adenopathy and hyperpigmentation
Anti-transglutaminase/anti-
Celiac Disease
gliadin/anti-endomysial antibodies
Vitamin given to pregnant women to
Folate
prevent NT defects
Newborn with chronic diarrhea, failure
SCID
to thrive, and chronic Candida
TG accumulation in hepatocytes Fatty Liver
Eosinophillic inclusion in the
Mallory Bodies from Alcoholic Hepatitis
cytoplasm of hepatocytes
Cancer closely linked to cirrhosis Hepatocellular Carcinoma
Severe hyperbilirubinemia in a neonate Crigler-Nagir, Type I
Mild, benign hyperbilirubinemia Gilbert's or Crigler-Nagir, Type 2
Hepatomegaly, abdo pain, ascites Budd-Chiari Syndrome
Green/yellow corneal deposits Wilson's Disease
Low serum ceruloplasmin Wilson's Disease
Cirrhosis, diabetes, and
Hemochromatosis
hyperpigmentation
Preferred anticoagulant for immediate
Heparin
anticoagulation
Preferred anticoagulant for long-term
Warfarin
anticoagulation
Preferred antocoagulant during
Heparin
pregnancy
A boy with self-mutilating behavior,
Lesch-Nyan Syndrome
mental retardation, and gout
Elevated Uric Acid Levels (4) Gout Lesch-Nyhan Tumor Lysis Loop or Thiazide Diuretics
Bluish-colored lines on the gingivae Lead Poisoning: Berton's Lead Lines
Causes of hypochromic, microcytic
Fe Deficiency Thalassemia Lead Poisoning
anemia
Hypersegmented neutrophils Megaloblastic Anemia: B12 or Folate Deficiency
Skull x-ray shows a "hair-on-end"
Marrow hyperplasia: Sickle-Cell Anemia or B-Thalassemia
appearance
Basophilic Stippling of RBCs Lead Poisoning
Painful cyanosis of the fingers and
Cold agglutinins
toes, with hemolytic anemia
Red urine in the morning, and fragile
Paroxysmal Nocturnal Hemoglobinuria
RBCs
Basophilic nuclear remnants in RBCs Howell-Jowell Bodies
Autosplenectomy Sickle Cell Disease
Hydroxyurea (blocks ribonucleoside reductase stopping UDP-
Drug used to treat sickle cell disease
>dUMP)
Antiplatelet Antibodies Immune Thrombocytopenic Purpura
Bleeding disorder with GpIb
Bernard-Soulier
deficiency
Most common inherited bleeding
von Willebrand Disease
disorder
Cancer most commonly associated
Hodgkin Lymphoma-exhibits B symtpoms
with noninfectious fever
HALLMARK "Smudge" Cells CLL
HALLMARK "Punched out" lytic
Multiple Myeloma
lesions
HALLMARK Sheets of lymphoid cells
Burkitt Lymphoma
with a "starry sky" appearance
HALLMARK RBCs clumped together
Rouleaux Formation-Multiple Myeloma
like a stack of coins
HALLMARK Monoclonal AB spike Multiple Myeloma MGUS Waldenstrom's Macroglobulinemia
HALLMARK Reddish-pink rods in the
Auer Rods-AML
cytoplasm of leukemic blasts
HALLMARK Large B Cells with
bilobed nuclei and prominent "owl Reed-Sternberg cells-Hodgkin Lymphoma
eye" inclusions
Most common leukemia in children ALL
Most common leukemia in adults CLL
HALLMARK Characteristic Auer
AML
Rods
Myelodysplastic syndromes have a
AML
tendency to progress to this
Myeloproliferative disorders may
AML
progress to this
More than 20% blasts in the marrow Acute Leukemia
Leukemia with more mature cells and
Chronic Leukemia
<5% blasts
HALLMARK PAS (+) acute leukemia ALL
HALLMARK Commonly presents
ALL
with bone pain
HALLMARK Numerous basophils,
splenomegaly, and negative for CML
leukocyte alkaline phosphatase
HALLMARK Always positive for the
CML
Philadelphia Chromosome
Acute leukemia positive for peroxidase AML
Solid sheets of lymphoblasts in marrow ALL
PAS (-) acute leukemia AML
Rx Treatment choice for Rickets or
Vit D
Osteomalacia
Swollen gums, poor wound healing,
bleeding mucous membranes, and Scurvy- Vit C deficiency
spots on the skin
Most common cancer of the appendix Carcinoid Tumor
Most common surgical emergency Appendicitis
GI hamartomas, hyperpigmentation of
Peutz-Jegher's
the mouth, hands and genitalia
Multiple colon polyps, osteomas, soft
Gardner's Syndrome
tissue tumors
Severe RLQ pain with rebound
Appendicitis
tenderness
HALLMARK Apple core" lesion on
Colon Cancer
barium enema
Most common cause of RLQ pain Appendicitis
Most common cause of LLQ pain Diverticulitis
Most common cause of RUQ pain Cholecystitis
Colonoscopsy reveals very friable
mucosa extending from the rectum to Ulcerative Colitis
the distal transverse colon
Most common site of colonic
Sigmoid colon
diverticula
HALLMARK "String sign" on contrast
Crohn's Disease
x-ray
HALLMARK "Lead pipe" appearance
Ulcerative Colitis
of colon on contrast x-ray
Food poisoning due to exotoxin B. cereus [rice]; S. aureus [all others]
Osteomyelitis in sickle cells disease Salmonella
Food poisoning as a result of
S. aureus preformed toxin
mayonnaise sitting out too long
Diarrhea caused by gram (-) nonmotile
Shigella
organism that does not ferment lactose
Rice-water stool Vibrio cholera
Diarrhea caused by an S-shaped
Yersinea
organism
Food poisoning from reheated rice B. cereus
Diarrhea caused by gram (-) motile
Salmonella
organism that doesn't ferment lactose
Most common cause of "traveler's"
ETEC
diarrhea
Diarrhea after a course of antibiotics C. difficile
Diarrhea caused by gram (-) lactose
E. coli
fermenting bacteria, no fever
Diarrhea caused by gram (-) comma-
Vibrio
shaped organism, no fever
Diarrhea + recent ingestion of water
Giardia/Entamoeba histolytica
from a stream
Food poisoning from undercooked
O157:H7
hamburger meat
Ring-enhancing brain lesion in an
Toxoplasma
HIV-infected patient
Treatment for Trichomonas vaginalis Metronidazole for patient & sex partner
Most common protozoal diarrhea Giardia Lamblia
Most common helminth infection in
Enterobius vermicularis
the US
Second most common helminth
Ascaris lumbricoides
infection in the US
Cause of malaria Cause of malaria Plasmodium
What enzyme catalyzes peptide bond
rRNA (ribozyme) Peptidyl transferase
formation during protein synthesis
What conditions are associated with
HbC Thalassemia Aplenia Liver disease
target cells (HALT)
What name is given to anemia
resulting from mechanical destruction
Macroangiopathic hemolytic anemia
of erythrocytes due to aortic stenosis or
prosthetic heart values
Where do platelets come from megakaryocytes
Primary hemostasis platelet plug
What is inside an endothelial cell vWF Factor VIII Thromboplastin tPA PGI2
Several subunits linked by disulfide bonds, synthesized by
What is vWF made up of
endothelial cells and megakaryocytes
Where are vWF specifically stored in
Weibel-palade bodies and alpha-granules of platelets
the endothelial cells
Complexes with factor VIII Platelet adhesion to vessel wall and
vWF function
other platelets (def > Increased bleeding time)
A deficiency of vWF would look like
Factor VIII (Increased PTT) Hemophilla A
what coagulation cascade def
vWF uses what glycoprotein to bind to
GpIb
platelets
vWF COX ADP, PDGF, serotonin, fibrinogen, lysosomal
Platelet content (10)
enzymes, thromboxane A2, calcium, thrombin
What does ADP do Helps platelets adhere to endothelium by realising GpIIb/IIIa
Binds to fibrinogen and forms cross-links among platelets
GpIIb/IIIa function
(aggregation)
NSAID Inhibits COX 1 and 2 (permanently by covalent
Aspirin
acetylation)
Leads to inhibition of TXA2 (decreases platelet aggregation) and
Aspirin
Prostaglandin
Aspirin Increases BT, no effect on PT or PTT
Antipyretic (Reduce fever) Analgesic (reduce pain) Anti-
Aspirin indications inflammatory Antiplatelet Acute MI Acute coronary syndromes
Acute thrombotic stroke MI prevention
Gastric ulceration Tinnitus Hyperventilation Respiratory alkalosis
Aspirin side effects Chronic use: Acute renal failure, interstitial nephritis, Upper GI
bleeding
Why should one not give children with Reye syndrome (Hepatic encephalopathy Liver problems
a viral sickness aspirin Hypoglycemia Mitochondrial abnormalities
Ticlopidine, Clopidogrel, Ticagrelor, Blocks ADP receptors -Prevents expression of GpIIb/IIIa - Inhibits
Prasugrel platelet aggregation
Why would you give an ADP receptor
You give them these drugs in case of allergy to aspirin
inhibitor instead of aspirin
Acute MI Coronary stenting Decreased incidence or recurrence of
ADP receptor inhibitor indications
thrombotic stroke (Post-MI) Aspirin allergy
ADP receptor inhibitor side effects Neutropenia (ticlopidine) TTP (ticlopidine)
Abciximab, eptifibatide, tirofiban
Inhibits Gp IIb/IIIa (directly) - Inhibits platelet aggregation
Glycoprotein IIb/IIIa Inhibitors
What is Abciximab made from Monoclonal antibody Fab fragments
What are Glycoprotein IIb/IIIa Unstable angina (NSTEMI) Percutaneous transluminal coronary
Inhibitors indications angioplasty
Platelet disorder labs PT, PTT, INR= Normal Bleeding time = increased
Bleeding from mucous membranes Epistaxis Petechiae
Platelet disorder clinical presentation
(microhemorrhages) Purpura
Immune thrombocytopenia purpura (ITP) Thrombotic
Platelet disorders thrombocytopenia purpura (TTP) Bernard-Souiler syndrome
Glanzmann thrombashenia
Low platelet count Increased bleeding time Increased
ITP lab findings
megakaryocytes (biopsy)
ITP be either acute or chronic. Who is
more prone to acute ITP and chronic Acute= children Chronic= women of childbearing age
ITP
ITP can be associated with Viral sickness or Lupus
ITP treatment Steroids IV Ig Splenectomy
Decreased platelet count Increased bleeding time Schistocytes
TTP lab findings
Increased LDH
TTP is assc. with what other disorder HUS (Hemolytic Uremic syndrome)
What bacteria is HUS associated with;
E. coli O157:H7
especially kids
No change/decreased platelet count Increased Bleeding time Also
Bernard-Soulier disorder labs
no agglutination on ristocetin cofactor assay
Defect in platelet plug formation Decreased GpIIb/IIIa (defect in
Glanzmann thrombasthenia
platelet-platelet formation) Low aggregation
Increased bleeding time Normal platelet count Agglutination with
Glanzmann thrombasthenia labs
ristocetin cofactor assay
Thrombocytopenia caused by other
Heparin induced Aplastic anemia
factors (Low platelet count)
Defect in vWF > Increased PTT (similar to Factor 8 def) Defect in
von Willebrand disease (AD)
platelet plug formation: decreased vWF > defect platelet-vWF
adhesion
No change in platelet count Increased bleeding time No change in
von Willebrand disease lab findings
PT Increased PTT
Mixed platelet and coagulation
von Willebrand disease DIC
disorders
von Willebrand disease diagnosis Ristocetin cofactor assay (Decreased agglutination= (+)
(DDAVP) Desmopressin: releases vWF from Weibel-pallade
von Willebrand disease treatment
bodies in endothelium
DIC (Disseminated Intravascular Widespread activation of clotting Consumes platelets and
Coagulation) coagulations factors Hypercoagulable (Bleed time increased)
Low platelets Increased bleeding time Increased PT and PTT
DIC (Disseminated Intravascular
Schistocytes Increased fibrin split products (D-dimers) Decreased
Coagulation) lab findings
fibrinogen (consuming) Decreased factors V and VIII (consuming)
STOP Making New Thrombi Sepsis (Gm -) Trauma Obstetric
DIC (Disseminated Intravascular
complications acute Pancreatitis Malignancy Nephrotic syndrome
Coagulation) causes
Transfusion
What else can increase d-dimers (fibrin
Snake Venom
split products)
What cytokines are increased in DIC TNF and IL-1
Activation - Secretion of ADP, PDGF, serotonin, fibrinogen,
lysosomal enzymes, thromboxane A2, calcium, thrombin -
Platelet Stimulation
Thrombin: fibrinogen fibrin - Thromboxane A2
vasoconstriction and platelet aggregation
What is the underlying
pathophysiology in thrombotic Deficiency of the metalloprotease ADAMTS 13
throbocytopenic purpura (TTP)
Anti-GpIIb/IIIa antibodies binds to receptors on platelets and then
What is the cause of ITP
immune systems removes platelets by splenic macrophages
What is the classic triad of HUS? Hemolysis Uremia Thrombocytopenia
What is the classic pentad of TTP Nasty Fever Torched His Kidneys
What is the defect in Bernard-Soulier
Decreased GpIb = defect in platelet to vWF adhesion
disease
What is the life span of a platelet
8-10 days
(which is also the maximum life of
platelets after a transfusion)
What molecule is expressed on the
surface of a platelet after it becomes GpIIb/IIIa
activated
NSAIDs inhibit the production of
which substance important in platelet Thromboxane A2
aggregation?
After a normal spontaneous vaginal
delivery,the new mom bleeds profusely
DIC: Decreased platelets Increased bleeding time Increased PT
from her vagina and later from her
and PTT Increased D-Dimer
gums. What abnormal lab values
would you suspect
What is the mechanism of action of
each of the following drugs? Directly or Indirectly aid conversion of plasminogen to plasmin
Streptokinase
What is the mechanism of action of
Inhibits COX 1 and 2
each of the following drugs? Aspirin
What is the mechanism of action of
each of the following drugs? ADP receptor inhibitor
Clopidogrel
What is the mechanism of action of
each of the following drugs? Glycoprotein IIb/IIIa Inhibitor
Abciximab
What is the mechanism of action of
Glycoprotein IIb/IIIa Inhibitors
each of the following drugs? Tirofiban
What is the mechanism of action of
each of the following drugs? ADP receptor inhibitor
Ticlopedine
What is the mechanism of action of
Low-molecular weight heparin Activatior of antithrombin
each of the following drugs?
(decreases thrombin and factor Xa)
Enoxaparin
What is the mechanism of action of
each of the following drugs? Glycoprotein IIb/IIIa Inhibitors
Eptifibatide
Antiplatelet antibodies ITP
Bleeding disorder with gpIb deficiency Bernard-Soulier syndrome
Most common inherited bleeding
vWF disease
disorder
What are the 3 different mechanisms
Ubiquitin protein ligase Lysosomes Calcium-dependent enzymes
cells employ to break down proteins
Which medication used in the
treatment of HIV is known for causing NRTIs (Zidovudine)
bone marrow suppression
What is the treatment for lead Adult vs Children Adult: EDTA and Succimer Children:
poisoning Succimer, EDTA, Dimercaprol
A decrease in serum haptoglobin Increase LDH (lactate
Intravascular hemolysis is going show dehydrogenase) Schistocytes Increased reticulocytes
Hemoglobinuria/Hemosiderinuria Urobilinogen in urine
a decrease in serum haptoglobin Increase LDH (lactate
Intravascular hemolysis is going show dehydrogenase) Schistocytes Increased reticulocytes
Hemoglobinuria/Hemosiderinuria Urobilinogen in urine
Macrophages in spleen clear RBCs Spherocytes in peripheral
smear Increase LDH (lactate dehydrogenase) No
Extravascular hemolysis is going show
hemoglobinuria/hemosiderinuria Increased unconjugated bilirubin
(jaundice)
Mechanical hemolysis (Prosthetic valve) Paroxysmal nocturnal
Intravascular hemolysis is caused by
hemoglobinuria Microangiopathic hemolytic anemia
Hereditary spherocytosis G6PD deficiency Pyruvate Kinase Def
Extravascular hemolysis diseases (5)
HbC defect Sickle cell anemia
Hereditary spherocytosis G6PD deficiency Pyruvate Kinase Def
Intrinsic hemolytic normocytic anemia
HbC defect Sickle cell anemia Paroxysmal nocturnal
diseases (Problem within RBCS)
hemoglobinuria
Defect in protein interacting with RBC membrane skeleton and
Hereditary spherocytosis
plasma mem.
What is MCHC Mean corpuscular hemoglobin concentration
Hereditary spherocytosis clinical
Splenomegaly Aplastic crisis (Parvovirus B19
findings
Hereditary spherocytosis diagnostic
Osmotic fragility test- Increase lysis
test?
Hereditary spherocytosis treatment? Splenectomy (HOWELL-JOLLY bodies)
Most common enzymatic disorder of RBCs X-linked recessive
G6PD deficiency Defect in G6PD > Decreased glutathione > increased RBC
susceptibility to oxidant stress
Oxidative stress factors are Sulfa drugs, antimalarials, infections, fava beans
Blood smears shows RBCs with Heinz bodies (clumps of
G6PD deficiency labs
hemoglobin) and bite cells (macrophages cause this)
G6PD deficiency symptoms Back pain, hemoglobinuria a few days after oxidant stress
AR Decreased ATP > rigid RBCs (swelling & hemolysis)
Pyruvate Kinase Deficiency
Swelling due to defect in Na+/K+ ATPase Common in newborns
Increased complement-mediated RBC lysis (impaired synthesis of
Paroxysmal nocturnal hemoglobinuria
GPI anchor for decay-accelerating factor that protects RBC
Characteristics What test would you
membrane from complement) Missing CD55 and CD59 = flow
use to test for it?
cytometry (+) Ham's test=RBC lysis at low pH
Paroxysmal nocturnal hemoglobinuria
Coomb (-) hemolytic anemia Pancytopenia Venous thrombosis
triad
HbS point mutation causes a single amino acid replacement in
Sickle cell disease
Beta chain (Sub. glutamic acid with valine)
What factors cause sickling of RBCs Low O2 tension Dehydration High altitude Acidosis
What infectious disease are people
with sickle cell anemia resistant to? Heterozygous (sickle cell trait) = resistance to malaria 8% of
What type of people are more prone to African americans
this trait?
Why are newborns at first
Because of the increased HbF in and decreased HbS
asymptomatic to sickle cell disease
Aplastic crisis (parvovirus B19) Autosplenectomy (howell-jolly
bodies) and Infections from SHiNE SKiS Salmonella
Sickle cell complications
osteomyelitis Painful crises (vaso-oclusive) Renal papillary
necrosis due to hypoxemia and microhematuria
What can be seen on skull x-ray in "Crew cut" on skull due to marrow expansion from increased
sickle cell patients erythropoiesis
Sickle cell disease treatment Hydroxyurea (increased HbF) Hydration
Heterozygous for hemoglobin C: Asymptomatic Homozygous for
HbC defect Common in newborn
hemoglobin C: -Mild hemolysis and splenomegaly HbC point
mutation causes a single amino acid replacement in Beta chain
(Sub. glutamic acid with lysine)
Extrinsic hemolytic normocytic anemia Autoimmune hemolytic anemia Microangiopathic anemia
diseases (Problem outside RBCS) Macroangiopathic anemia Infections
Autoimmune hemolytic anemia Warm agglutinin (IgG) Cold agglutinin (IgM) Coombs (+)
RBCs are damaged when passing through obstructed or narrow
Microangiopathic anemia
vessel lumina
Microangiopathic anemia can be
DIC, TTP/HUS, SLE, and Malignant HTN
caused by
"Helmet cells" Seen on blood smear due to mechanical destruction
Schistocytes
of RBCs
Prosthetic heart valves and aortic stenosis may cause hemolytic
Macroangiopathic anemia anemia, secondary to mechanical destruction Present with
schistocytes
Infections causing hemolytic anemia Malaria (Plasmodium spp.) Babesia
A child anemic since birth has now
been cured with splenectomy. What is Hereditary spherocytosis
the disease
What is the difference between the HbS is a point mutation of B-globin (by sub. of glutamic acid for
hemoglobin S defect and the valine) HbC is a point mutation of B-globin (by sub. of glutamic
hemoglobin C defect? acid for lysine)
RBC agglutination with the addition of antihuman antibody
Coombs' (+) because RBCs are coated with immunoglobulin or complement
proteins
Direct Coombs' (DAT) "Type and screen"
Cold: Antibodies against RBCs that interact more strongly at low
What is the difference between a warm
temps (4C) than at body temp Warm: Antibodies that react
agglutinin and a cold agglutinin
against RBC protein antigens at body temperature
What are schistocytes Mechanically destructed RBCs
What are two protozoal diseases that
Plasmodium spp. Babesia
can cause hemolytic anemia
What findings are associated with Anemia Jaundice Pigmented gallstones Splenomegaly (+) osmotic
hereditary spherocytosis fragility test Coomb (-) Spherocytes in peripheral smear
Cold Agglutinins Antibodies against RBCs that interact more strongly at low temps
(4C) than at body temp
Antibodies that react against RBC protein antigens at body
Warm Agglutinins
temperature = chronic anemia
HALLMARK Ham's test Paroxysmal nocturnal hemoglobinuria
HALLMARK Heinz bodies G6PD deifcency
HALLMARK DEB test Fanconi anemia
HALLMARK Osmotic fragility test Hereditary spherocytosis
HALLMARK Basophilic stippling Lead poisoning Thalassemia
Sx Painful cyanosis of the fingers and
Cold agglutinins
toes, with hemolytic anemia
Autosplenectomy Sickle cell disease
Drug used to treat sickle cell disease Hydroxurea
What is the target HgbA1C for every
<7.0
diabetic patient
What are the fiver hereditary Antithrombin deficiency Factor V Leiden Protein S deficiency
thrombosis syndrome Protein C deficiency Prothrombin gene mutation
Which Mycobacterium spp. fits each of
the following description? Causes Mycobacterium leprae
leprosy
Which Mycobacterium spp. fits each of
the following description? Causes
M. tuberculosis
pulmonary TB-like symptoms in
COPD patients
Which Mycobacterium spp. fits each of
the following description? Causes M. Scrofulaceum
cervical lymphadenitis in children
Which Mycobacterium spp. fits each of
the following description? Causes a M. avium-intraceullulare
disseminated disease in AIDS patients
Which Mycobacterium spp. fits each of
the following description? Hand M. marinum
infection in aquarium
Atrial fibrillation What are some Irregularly spaced ORS complexes Irregular baseline No steady
charcteristics? rhythm (SA node dysfunction) (Multiple SA nodes) No
coordinated atrial contraction = no discrete P waves
Atrial fibrillation can cause Atrial stasis (pooling) and lead to thrombosis>emboli
SVT= Supraventricular tachycardia It is caused by the increased
Atrial fibrillation can also cause SVTs?
signals by the multiple SA nodes to the AV node can cause
What is an SVT? How is it caused?
frequent ventricular depolarization
Atrial fibrillation treatment? New vs
New: <48 hrs = synchronized cardioversion
Chronic
Chronic: Heparin, enoxaparin Coumadin Rate control (digoxin,
Rate control drugs? Rhythm control
Beta-blockers, CCBs) Rhythm control (sotalol, amiodarone)=
drugs?
Potassium channel blockers
Why do you not cardioverse a chronic This can cause a thrombus to dislodge and form emboli, due to the
atrial fib heart resetting the contraction
HALLMARK Atrial flutter
Sawtooth appearance on EKG
characteristics
First Degree Heart Block PR interval prolonged (>200 msec or 5 little boxes) Asymptomatic
First Degree Heart Block can be
CCBs (Toxicity= Increased PR interval)
caused by what drug
What bacterial infection can cause an
Borrelia burgdorgeri (Lyme disease)
AV node block
Second degree (Mobitz 1) Heart Block Unsteady rhythm PR intervals increase, until beat drop Every QRS
(Wenckebach) What is the major fact complex is followed by a P-wave, but every P-wave doesn't
about it? followed by a QRS complex Benign
What type of pattern is seen in Second
Regularly Irregular
degree (Mobitz 1) Heart Block
Second degree (Mobitz 2) Heart Block
Unsteady rhythm No increae in PR intervals except for DROP
Characteristics? What can is progress
(abrupt) Can progress to 3rd degree block Treated with pacemaker
to? What is it treated with?
Steady rhythm Decreased HR Atria and Ventricular beat
Third degree heart block
independently (SA no communicating with AV) Narrow QRS
Characteristics? What bacterial disease
complex Ventricular rate is slower, atrial faster Treated with
assc. with? Treatment?
pacemaker Lyme disease
Ventricular pre-excitation syndrome Accessory conduction
Wolf-Parkinson-White Syndrome pathway from atria to ventricle Bypass AV node (shortened PR
interval) Goes through bundle of Kent Ventricles partially
depolarize earlier
Ventricles partially depolarizing earlier
Delta waves with widened QRS and shortened PR intervals
brings about what characteristics
Wolf-Parkinson-White Syndrome can
Reentry ciruit > SVTs
result it in
Wolf-Parkinson-White Syndrome treat
Class IA (procainmide) and III (amiodarone)
with
Paroxysmal Supraventricular Originates above the AV node Can be caused by AV nodal
Tachycardia Caused by reentrant tachycardia Narrow QRS
Delayed heartbeat no originating from SA node, signal is coming
from AV junction Protective mechanism is SA node is
Junctional Escape Rhythm dysfunctional, then another area of heart fires off heart beats P-
wave location can change depending on where signal originated
from
Ventricular premature beats or PVC Early occurring wide QRS complexes Microentery at Purkinje
(Premature ventricular contraction) fibers No P waves, widened QRS
Ventricular bigeminy Occurs when a ventricular premature beat follows each sinus beat
Ventricular trigeminy 2 sinus beats followed by a ventricular premature beat
Failure for SA and AV nodes So no P waves and get widened QRS
complexes Longer than avg R to R intervals persistent rhythm
Ventricular Escape Rhythm
leads to ventricular complexes Persistence can lower rate < sinus
rhythm
Ventricular Tachycardia (VT) 3 or more successive ventricular complexes
Nonsustained VT: -Series of repetitive ventricular beats -Duration
Nonsustained vs Sustained
<30secs Sustained VT: Duration >30secs
Rhythm is normal Slight irregularity of R-R intervals QRS axis
Monomorphic VT
shifted to left Width of QRS >0.6 secs
What happens if Monomorphic VT is
hemodynamic collapse; Treat: async defibrillation and CPR
sustained? treat with?
Polymorphic VT Shifting sinusoidal waveforms Progress to V. Fib
Torsades de Pointes
Wide QRS and tachycardia Long QT interval predisposes
What causes Torsades de Pointes?
Hypokalemia Hypomagnesemia Drugs Treat: Magnesium sulfate
Treatment?
Ventricular Fibrillation A completely erratic rhythm with NO identifiable waves. Fatal
w/o CPR or defib
What is the initial treatment for
Monomorphic: defibrillation and CPR
ventricular fibrillation
What is the hallmark of a third degree
independent beating of atrium and ventricle
heart bloc
What drugs are know to prolong QT Antibiotics (macrolide) Antiemetics (ondansetron)
interval, increasing the likelihood of Antidepressants (TCA) Antiarrhythmics (Class 1A and 3)
torsades in those at risk Antipsychotics (haloperidol)
What are the two different types of
Mobitz type 1: Increasing PR interval then drop Mobitz type 2: No
second degree AV block? How do they
warning, normal PR interval + drop
differ?
Why is warfarin anticoagulation
important in patients with chronic atrial Prevents any thrombosis to form due to atrial stasis (pooling)
fibrillation?
What is the fibrous band attached to
the testis and scrotum that aids in Gubernaculum Female Remnant: Ovarian ligament + Round
normal testicular descent? What is this ligament of uterus
structure called in females?
Hereditary hemorrhagic telangiectasia AD disorder of blood
What is Osler-Weber-Rendu vessels. Findings: telangiectasia, recurrent epistaxis, skin
syndrome? discolorations, arteriovenous malformations (AVMs), GI bleeding,
hematuria.
Which studies use odds ratios, and
Case-control uses Odds ratio Cohort uses Relative Risk
which use relative risks?
What are the effects of NSAIDs' Increased vascular tone-vasoconstriction Increased bronchial tone-
inhibition of PGE2? bronchoconstriction
What role do prostaglandins and prostaglandins: Dilate afferent arteriole (Increases RPF, GFR, so
angiotensin II play on the renal FF remains constant) Angiotensin II: Constricts efferent arteriole
arterioles? Decreases RPF, Increases GFR, so FF increases)
What is the mechanism of action of the
Antimuscuranic (M1)
following antiemetics? Scopolamine
What is the mechanism of action of the
Histamine antagonist D2 dopamine antagonist
following antiemetics? Promethazine
What is the mechanism of action of the
D2 dopamine antagonist
following antiemetics?
Prochlorperazine
What is the mechanism of action of the
following antiemetics? D2 receptor antagonist
Metoclopramide
What is the mechanism of action of the
5-HT3 antagonist
following antiemetics? Ondansetron
What are the 3 endogenous androgens,
in order from the most potent to the DHT > testosterone > androstenedione
least potent
JG cells secrete renin in response to decreased renal blood
How does a decrease in renal artery
pressure, decreases NaCl delivery to distal tubule, and Increases
pressure cause an increase in blood
sympathetic tone (1). Angiotesinogen > Angiotensin I >
pressure
Angiotensin II
What physiologically is taking place in Gases (especially nitrogen) that had dissolved in the blood at high
decompression sickness pressures form gas bubbles that can occlude blood vessels
Is a 34-year-old schizophrenic patient
having active hallucinations, who is
No
not oriented to time, place, or person,
able to legally agree to a plan of care?
Patient is 18 years old or otherwise legally emancipated Patient
What factors must be in place in order
makes and communicates a choice Patient is informed (knows and
for a patient to have the capacity to
understands) Decision remains stable over time Decision is
make a decision?
consistent with patient's values and goals, not clouded by a m
The thalamus is a relay station that
conducts signals to and from the brain.
Which thalamic nuclei are the relay
MGN (Medial Geniculate)
stations for each of the following body
sensations/activities? Auditory
sensation
The thalamus is a relay station that
conducts signals to and from the brain.
Which thalamic nuclei are the relay LGN (Lateral Geniculate)
stations for each of the following body
sensations/activities? Visual sensation
The thalamus is a relay station that
VL (Ventral Lateral)
conducts signals to and from the brain.
Which thalamic nuclei are the relay
stations for each of the following body
sensations/activities? Motor to the
body
The thalamus is a relay station that
conducts signals to and from the brain.
Which thalamic nuclei are the relay VPM (Ventral Posteromedial)
stations for each of the following body
sensations/activities? Facial sensation
The thalamus is a relay station that
conducts signals to and from the brain.
Which thalamic nuclei are the relay VPL (Ventral Posterolateral)
stations for each of the following body
sensations/activities? Body sensation
What is PPAR- Peroxisome Proliferator-activated Receptor Gamma
Plays a role in adipocyte differentiation Nuclear transcription
How is it relevant to the treatment of
regulator Through Glitazones/thiazolidinediones It increases the
diabetes mellitus?
insulin sensitivity in peripheral tissue
A 23-year-old man recently finished
treatment to eradicate the lice
infestation that he acquired during a
Borrelia recurrentis
recent trip to Mexico. What organism
might be responsible for this man's
recurrent fever?
What germ layer gives rise to each of
Retina Neuroectoderm
the following adult structures
What germ layer gives rise to each of
Salivary glands Surface ectoderm
the following adult structures
What germ layer gives rise to each of
Pancreas Endoderm
the following adult structures
What germ layer gives rise to each of
Muscle of Abdominal wall Mesoderm
the following adult structures
What germ layer gives rise to each of
Thymus Endoderm
the following adult structures
What germ layer gives rise to each of
Spleen Mesoderm
the following adult structures
What germ layer gives rise to each of
Aorticopulmonary septum Neural Crest
the following adult structures
What germ layer gives rise to each of
Anterior pituitary Surface ectoderm
the following adult structures
What germ layer gives rise to each of
Posterior pitutary Neuroectoderm
the following adult structures
What germ layer gives rise to each of
Bones of skull Neural Crest
the following adult structures
What germ layer gives rise to each of
Cranial nerves Neural Crest
the following adult structures
Left to Right Shunts; what are they?
ASD, VSD, PDA
(3)
What are the clinical uses for 1st
(PEcK), 2nd (HEN PEcKS), 3rd, and 1st gen- Gm (+), Proteus m., E.coli, Klebsiella
4th generation cephalosporins
What are the clinical uses for 2nd gen- Haemophilus influenzae, Enterobacter a., Neisseria spp., Gm (+),
cephalosporins Proteus m., E.coli, Klebsiella, Serratia m.
What are the clinical uses for 3rd gen- Serious gram-negative infections resistant to other Beta-lactams,
cephalosporins Pseudomonas, Neisseria gonorrhea
What are the clinical uses for 4th gen (Cefepime)- Gm (-), increase activity against Pseudomonas and
cephalosporins Gm (+) organisms
What are the classic symptoms of (BFDR) Brochospasm and wheezing cutaneous flushing Diarrhea
carcinoid syndrome Right-sided valvular disease
Atrial Septal Defect (ASD) is caused
Hole in the interatrial septum
by
What type of murmur do you hear in
Loud S1; wide fixed spilt S2
an ASD?
What can be seen on X-ray in ASD? RVH
Ventricular septal defect Most common congenital heart defect
Defect in intraventricular septum 40% close in first 6 months of life
Ventricular septal defect can cause
If lesion is too large it can cause LV overload or HF
what
Patent Ductus Arteriosus (PDA) is Prostaglandin synthesis and low O2 tension f infant is born
caused by through C-section
What type of murmur do you hear in a Continuous, "machine-like" murmur Left upper sternal border of
Patent Ductus Arteriosus (PDA) the newborn
Patent Ductus Arteriosus (PDA)
Indomethacin (blocks PGE synthesis)
treatment
When a patient has Transposition of the great vessels, to allow
When is PDA to remain open some oxygenated blood to mix with the deoxygenated blood
(systemic)
long-standing L to R shunt Increased pulmonary flow> pathologic
Eisenmenger Syndrome Severe remodeling of vasculature > pulmonary arterial htn RVH occurs to
compensate > shunt becomes right to left
Eisenmenger Syndrome symptoms Later cyanosis, clubbing, polycythemia, and SOB
Infantile: Aortic narrowing near intersection of ductus arteriosus
Coarctation of the Aorta (infantile) ("juxtaductal") - only blood flowing to distal aorta is from PDA.
(Req open PDA)...Assc. with Turner Syndrome
Distal to ductus arteriosus, Once PDA is closed remnant =
Coarctation of the Aorta (adult)
Ligamentum aretiosum
Once PDA is closed the remnant is Ligamentum aretiosum
Hypertension in upper extremities and weak delayed in lower
Coarctation of the Aorta symptoms extremities (brachial-femoral delay) Rib notching on chest X-ray
(Dilated intercostal arteries) Can also lead to aortic regurg.> HF
Right to Left shunts Early cyanosis- "blue babies" (bypass pulmonary circuit)
Requires surgery or an open PDA right to left shunts
Truncus arteriosus (1 vessel) Transposition (2 switched vessels)
Right to Left Shunt Diseases (5 Ts)
Tricuspid atresia (3= Tri) Tetralogy of Fallot (4 =Tetra) TAPVR
(hint: TAPVR (5 letters in name)
(5 letters in name)
Failure of truncus arteriosus to divide into pulmonary trunk and
Persistent truncus arteriosus aorta Deoxy blood of RV mixes with Oxy blood of LV Cyanosis,
Respiratory distress, HF May have VSD
D-transposition of Great Vessels
Aorta and Pul. artery are switched so Aorta leaves RV and
(TGA) What does it req. to be
Pulmonary a. leaves LV. Separation of pulmonary and systemic
compatible with life? What medication
circulation Cyanosis is significant Prostaglandins
should be give to help shunts?
Who are more prone to TGA? Infant Diabetic Mothers
Infants suffer from shoulder dystocia, clavicular fractures, brachial
LGA (Large for gestational age)
plexus injury (Erb or Duchenne's palsy) Increased C-section
Tricuspid atresia Absent tricuspid valve and hypoplastic RV Require ASD and VSD
Caused by anterosuperior displacement of the infundibular septum
Tetralogy of Fallot
(MC cause of early cyanosis)
4 components of Tetraology of Fallot Pulmonary infundibular stenosis RVH Overriding aorta VSD
What helps with the spells of cyanosis Squatting=Increase SVR, Decreases R>L shunt, improves
in Tetralogy of Fallot cyanosis
Tetralogy of Fallot classic x-ray
RVH or Boot-shaped heart in infant
finding is
Total anomalous pulmonary venous
Pulmonary veins drain into right atria circulation (SVC, Coronary
return (TAPVR) Open or closed loop?
sinus) Closed loop Without shunt patient will die
What is needed to keep patient alive?
What is the most common congenital
VSD
cardiac anomaly
Associated with Maternal Lithium use Physical exam: widely split
Ebstein anomaly
S2, tricuspid regurgitation
Explain how the great vessels are Aorta and Pul. artery are switched so Aorta leaves RV and
attached in a transposition of the great Pulmonary trunk leaves LV. Separation of pulmonary and
vessels systemic circulation
What heart defect is associated with
deletion of Chromosome 22q11 Tetralogy of Fallot Persistent Truncus arteriosus
deletions
Describe blood flow through a PDA L > R shunt Aorta > Pulmonary artery
What heart defect is associated with Endocardial cushion defect; Can cause abnormal connection of all
Down syndrome 4 chambers of the heart
A 45-year-old man presents with a BP
of 160/90 on the right arm and 170/92
on the left arm.There are no palpable Coarctation of the Aorta
pulses in the feet/ankle. What problem
does this patient most likely have?
What heart defect is associated with
each of the following disorders: PDA Pulmonary artery stenosis
Congenital rubella
What heart defect is associated with
each of the following disorders? Turner Bicuspid aortic valves Infective endocarditis
syndrome Infantile coarctation
What heart defect is associated with
Aortic insufficiency Continuous machinery-like heart murmur
each of the following disorders?
PDA
Marfan syndrome
HALLMARK "Boot-shaped heart" RVH or Tetralogy of Fallot
HALLMARK Rib notching Coarctation of aorta
Most common congenital cause of
Tetralogy of Fallot
early cyanosis
Where does erythropoiesis take place
Fetus: Yolk Sac. Liver, spleen, bone marrow
in the fetus (4)?
Which bones in adults synthesize Adults:Vertebrae, Sternum, Pelvis, Ribs, cranial bones, and tibia &
RBCs (7)? femur
Rx Next step in treatment of otitis
Amoxicillin + Clavulanic acid (beta-lactamase inhibitor)
media if resistant to amoxicillin
Rx Prophylaxis against bacterial
1st gen cephalosporin, Aminopenicillins, and Penicillin V
endocarditis
Rx Increases the nephrotoxicity of
Cephalosporins
aminoglycosides
Rx Sufficient for the treatment of
Penicillin G
syphilis
Rx Single dose treatment for gonorrhea Ceftriaxone
What divides the right and left atria Septum primum and secundum
How is blood shunted from the right
Foramen ovale and ostium secundum
atrium to the left atrium in an embryo
What structure grows to close the
opening/canal between the atrial
Superior and Inferior endocardial cushion
chamber and ventricular chamber into
two smaller openings
What genetic abnormality is commonly
associated with endocardial cushion Trisomy 21
defects
Truncus arteriosus and Neural crest cells
Aorticopulmonary septum arises from
what type of cells
Which embryologic structure of the
heart gives rise to each of the following
Truncus arteriosus
adult structures? Ascending aorta and
pulmonary trunk
Which embryologic structure of the
heart gives rise to each of the following
Bulbis Cordis
adult structures? Smooth parts of the
left and right ventricles
Which embryologic structure of the
heart gives rise to each of the following
Primitive Ventricle
adult structures? Trabeculated parts of
the left and right ventricles
Which embryologic structure of the
heart gives rise to each of the following
Primitive Atrium
adult structures? Trabeculated left and
right atria
Which embryologic structure of the
heart gives rise to each of the following Left horn of sinus venosus
adult structures? Coronary sinus
Which embryologic structure of the
heart gives rise to each of the following
Right horn of sinus venosus
adult structures? Smooth part of the
right atrium
Which embryologic structure of the
heart gives rise to each of the following R. common cardinal vein and right anterior cardinal vein
adult structures? SVC
What vessel carries oxygenated blood
Umbilical vein
from placenta to fetus
Areas of shunting in fetal blood flow Ductus venosus Foramen Ovale Ductus arteriosus
What starts shutting down fetal
circulation, and gives rise to adult Breathing (respiration)
circulation
Common to hear what murmur in the
PDA
first 24 hrs of life, and then it
disappears
Which fetal vessel has the highest
Umbilical Vein
oxygenation
What structure divides the truncus
arteriosus into the aortic and
Aorticopulmonary septum (Spiral) Neural Crest cells
pulmonary trunks? What is the cellular
origin of this structure?
What causes the ductus arteriosus to Breathing Increased O2 and decreased prostaglandins
close [Indomethicin] (Drug to help close PDA)
Breathing Decrease resistance in pulmonary vasculature > increase
What causes the foramen ovale to close
in left atrial pressure vs. right atrial pressure
Order of oxygenation from highest to
Umbilical Vein> Ductus venosus> IVC> R. atrium
lowest (3)
Pituitary excess: Acromegaly, TSH-secreting tumor, ACTH-
secreting tumor GI endocrine excess: Carcinoid syndrome, ZES
What are some of the clinical uses for
syndrome, VIPoma, glucagonoma, insulinoma Diarrhea Reduce
somatostatin
splanchnic blood circulation: Cirrhosis with bleeding esophageal
varices
What is the name given to a thyroid
Mature teratoma (struma ovarii)
hormone-secreting teratoma
Constipation Lowering of the voice Menorrhagia Slowed mental
and physical function Dry skin with coarse, brittle hair Reflexes
Hypothyroidism symptoms
showing slow return phase Myxedema (facial/periorbital) Weight
gain Cold intolerance
Levothyroxine Synthetic analog of thyroxine (T4)
Why should you start Levothyroxine at
Can cause tachycardia, heat intolerance, arrythmias
a low does for elderly patients
Triiodothyronine Synthetic analog of T3
Poor brain development Pot-bellied Pale Puffy-faced Protuding
Cretinism findings? (6 P's)
umbilicus Protuberant tounge
Most common cause of hypothyroidism in the U.S. Autoimmune
Hashimoto thyroiditis
disorder Painless goiter (nontender)
Antithyroglobulin and Antithyroid peroxidase antibodies Assc.
Hashimoto thyroiditis diagnosis
with HLA-DR5 and HLA-B5
Hashimoto thyroiditis histology Dense infiltrates (germinal centers) of lymphocytes into thyroid.
findings? What are these cells called? Hurthle cells
Hashimoto thyroiditis can cause an
Non-Hodgikin lymphoma (Marginal Zone) (B-cell)
increased risk in what lymphoma?
Hashimoto thyroiditis can start off as
Thyrotoxicosis during follicular rupture
hyperthyroidism because?
Hashimoto thyroiditis can also be assc.
Diabetes, Sjrojen syndrome, and Pernicious anemia
with what other diseases (3)
Hypothyroidism with a PAINFUL (tender) goiter Self-limited
Subacute (de Quervain) Thyroiditis
disease
Subacute (de Quervain) Thyroiditis can
Hyperthyroidism
start off as what?
Subacute (de Quervain) Thyroiditis
Focal destruction with Granulomatous inflammation
causes what to the thyroid?
Subacute (de Quervain) Thyroiditis has
Increase
an increase or decrease of ESR?
Subacute (de Quervain) Thyroiditis is
HLA-B35, Effects Females (3:1)
associated with?
Subacute (de Quervain) Thyroiditis
Viral infection (Flu-like) or URI
often follows what?
Riedel's thyroiditis caused by? Chronic inflammation of thyroid >
Riedel's thyroiditis caused by?
fibrous tissue (Hypothyroid or euthyroid)
Riedel's thyroiditis presents as? Fixed, hard, rock-like thyroid Painless goiter Histology:
Histology? Macrophages and Eosinophils
Prolonged Riedel's thyroiditis in a
Extension of fibrosis which may extend to airway or other
younger patient can lead to? Mimics
structures. Anaplastic carcinoma
what cancer?
Grave's disease or multinodular goiter , or even a single toxic
Radioiodine uptake scan increased
adenoma
Radioiodine uptake scan decreased Adenoma or Carcinoma or thyroid
Most common thyroid cancer Great prognosis Female
Papillary carcinoma
predominance MC during 30s-50s
"Orphan Annie" eyes or "Ground Glass" appearance or
HALLMARK Papillary carcinoma
Psammoma bodies (concentric calcfications)
Exposure to childhood irradiation Increased RET and BRAF
Papillary carcinoma risk factor
mutation
Follicular carcinoma 2nd most common thyroid cancer Worse prognosis
Follicular carcinoma diagnosis Uniform cubodial cells lining the follicles Invade thyroid capsule
What differs follicular carcinoma and
Invasion of capsule occurs in carcinoma
follicular adenoma
How does Follicular carcinoma spread Hematogenously
Benign tumor of thyroid Proliferation of follicles surrounded by
Follicular adenoma of thyroid
capsule NO invasion
Medullary carcinoma of thyroid Proliferation of parafollicular "C" cells
Parafollicular "C" cells like to secrete Calcitonin
Medullary carcinoma of thyroid assc.
MEN 2A and 2B (RET mutations)
with
Anaplastic carcinoma of thyroid Anaplastic undifferentiated neoplasm Older patients
Hoarseness (Recurrent laryngeal nerve) Hypocalcemia (removal of
parathyroid glands) Transection of recurrent and superior
Thyroidectomy complications? (3)
laryngeal nerves (during ligation of inferior thyroid artery and
superior laryngeal artery)
What type of thyroid cancer matches
each of the following statements? Most
Papillary carcinoma
common type of thyroid cancer (70-
75%)
What type of thyroid cancer matches
each of the following statements?
Follicular carcinoma
Second most common type of thyroid
cancer (10%)
What type of thyroid cancer matches
each of the following statements? Papillary and Follicular carcinoma
Activation of receptor tyrosine kinases
What type of thyroid cancer matches
each of the following statements? B-cell Lymphoma
Hashimoto thyroiditis is a risk factor
What type of thyroid cancer matches
Medullary carcinoma
each of the following statements?
Cancer arising from parafollicular C
cells
What type of thyroid cancer matches
each of the following statements?
Commonly associated with either a Follicular carcinoma
RAS mutation or a PAX8-PPAR
gamma 1 rearrangement (LY)
What type of thyroid cancer matches
each of the following statements?
Commonly associated with Papillary carcinoma
rearrangements in RET oncogene or
NTRK1
What type of thyroid cancer matches
each of the following statements? Most
Papillary carcinoma
common mutation in the BRAF GENE
(serine/threonine kinase)
Cold tolerance Hypothyroidism
Enlarged thyroid cells with ground-
Papillary carcinoma
glass nuclei
Hypothyroidism will cause elevation
(Hypercholesterolemia) Due to decreased LDL receptor expressio
LDL and cholesterol. Why?
Excessive daytime sleepiness Rapid progression from walking
What are the characteristics of
state to REM sleep Cataplexy (sudden muscular weakness)
narcolepsy
Hallucinations
Daytime stimulants:(Amphetamines/Modafinil) and Nighttime
What is the treatment for narcolepsy
sodium oxybate (GHB)
What is the first-line treatment for a Phosphodiesterase inhibitors Sildenafil Vardenafil Tadalafil If
patient with erectile dysfunction hypogonadal: Testosterone replacement
What other drugs are used to treat
Infliximab and adlimumab
Crohn disease that target TNF-
With what physical exam finding must
you presume scaphoid fracture despite Anatomical snuffbox tenderness = scaphoid fracture
a normal initial x-ray
What might result in a proximal
fracture of the scaphoid if left Avascular necrosis in the proximal fragment
untreated
During what week of fetal
development will the fetus reach the Week 1
following landmarks? Implantation
During what week of fetal
development will the fetus reach the Week 3-8
following landmarks? Organogensis
During what week of fetal
development will the fetus reach the
Week 4
following landmarks? Heart begins to
beat
During what week of fetal
development will the fetus reach the
Week 10
following landmarks? can distinguish
male or female genetalia
During what week of fetal
development will the fetus reach the Week 3
following landmarks? gastrulation
During what week of fetal
development will the fetus reach the
Week 3-4
following landmarks? Formation of
primitive streak and neural plate
Overgrowth of desquamated keratin debris within middle ear
What is a cholesteatoma, and how does
space May erode ossicles, mastoid air cells > conductive hearing
it present
loss and vertigo Grayish-white pearly lesion behind TM
Premature puberty in children Premature closure of epiphyseal
What are the adverse reactions of
plate Erythrocytosis Worsening of sleep apnea Suppression of
exogenous testosterone in males
spermatogenesis Increased LDL and Decreased HDL
What is the initial medical treatment
for the arrhythmia known as torsades IV Magnesium sulfate
de pointes
What structures give rise to the blood Tight junctions between nonfenestrated capillary endothelial cells
brain barrier Basement membrane Astrocyte foot processes
Which diuretics are most appropriate K+-sparing diuretics Spironolactone, Amiloride, Triamterene,
for patients with hyperaldosteronism? Eplerenone Competitive aldosterone receptor antagonist
What are their important side effects? Hyperkalemia:SE
Diagram the pathway by which the Cochlea > Cochlear nuclei > Contralateral superior olivary nucleus
cochlea communicates a signal to the > Lateral lemniscus > Inferior colliculus > Medial geniculate body
primary auditory cortex. > Primary auditory cortex
What clinical presentation might lead
you to suspect a patient may have Weight loss, night sweats, fever, and painless lympadenopathy
lymphoma
Most common leukemia in children ALL
Most common leukemia in adults in
CLL
US
Characteristic Auer rods AML
Greater than 20% blasts in marrow Acute Leukemia
Leukemia with more mature cells and
Chronic leukemia
less than 5% blasts
PAS (+) acute leukemia ALL
Always positive for the Philadelphia
CML
chromosome (t 9;22)
Acute leukemia positive for
AML
PEROXIDASE
Solid sheets of lymphoblasts in marrow ALL
Always associated with the BCR-ABL
CML
genes
What histological change takes place in
Meteplasia (columnar to squamous)
the trachea of a smoker
Self-limited episode that must be present for atleast 2 weeks
Major Depressive Disorder
Decreased NE and Serotonin
SSRI- inhibits reuptake of serotonin SNRI- inhibits reuptake of
Common Antidepressants serotonin and NE TCA- inhibits reuptake of serotonin and NE
MAOI- monoamine oxidase inhibitor
Electroconvulsive therapy (ECT) Used for depression that isn't responding to any other treatments
Treatment for atypical depression First-line: SSRI
50-85% incidence rate Depressed affect, tearfulness, and fatigue
"Maternal (postpartum) blues" (2-3 days after delivery) Resolves in a week Support + follow-up=
treatment
What is postpartum depression Postpartum depressive symptoms that persist for more than or
equal to 2 weeks 10-15% incidence rate Depressed affect Anxiety
Poor conc.
0.1-0.2% incidence rate Mood-congruent delusions Hallucinations
Postpartum psychosis Thoughts of harming baby/self Few days to few weeks Rx.
Antipsychotics/Antidepressants
Dysthmia Persistent depressive disorder Often milder Lasts at least 2 years
Mild depression during winter months (due to decreased sunlight)
Seasonal affective disorder
Rx. Go outside or go on vacation
SAD PERSONS scale: Sex (male) Age (under 19 or over 45)
Depression Previous attempts Ethanol (alcohol/drug use) loss of
Risk factors for Suicide
Rational thinking Sickness Organized plan No social support
Stated future intent
Who tries to commit suicide more?
Women: try Men: succeed
Who succeeds?
Hypersomnia Hyperphagia Mood reactivity Hypersensitivity to
What is atypical depression
rejection Leaden paralysis Common subtype of Depression
A 28-year-old woman has symptoms of
mild depression for 6 years. What's the Dysthymia
diagnosis
A 55-year-old man who is a smoker
and heavy drinker presents with a new
cough and flu-like symptoms. Gram-
Legionairres disease
stain shows no organisms; silver stain
of sputum shows gram-negative rods.
What is the diagnosis?
What is the classic presentation of a
patient with temporalis arteritis? What Migranes (unilateral), loss of vision, pain when chewing Increased
lab findings help diagnose temporalis ESR and Biopsy (definitive)
arteritis
Used for Depression, Bulimia, general anxiety, and OCD,
SSRIs Premenstrual dysmorphic disorder Inhibit re-uptake of serotonin
Takes weeks for it have any effect Less toxic
Side effects of SSRI Sexual dysfunction and Serotonin syndrome
Drugs associated with Serotonin SSRIs SNRIs MAOIs St. Johns Wort Kava Kava Tryptophan
Syndrome? (8) Cocaine Amphetamines
Cool patient down and provide benzodiazepine (first line) Or
Treatment for Serotonin Syndrome
Cyproheptadine (5-HT2 receptor blockers)
Side effects of SNRIs Increased BP, sedation, nausea
Inhibit serotonin and NE re-uptake (Amitrptyline and
TCAs (Tricyclic Antidepressants) Clomipramine-OCD) Older drug Depression Imipramine=
bedwetting (enuresis)
Sedation Alpha blocking effects Anticholinergic effects Decrease
Side effects of TCAs
seizure threshold (desipramine and nortripyline not as much)
NaHCO3 to prevent arrhythmia (to alkalinized the urine)- excretes
Rx Treatment for TCA overdose
TCAs
Monoamine Oxidase Inhibitors Nonselective MAO inhibition which increases levels of amine
(MAOI) neurotransmitter (NE, 5-HT, and Dopamine)
What is Selegiline used for Parkinsonism-like symptoms, so it increases dopamine
MAOI side effects? What is "wash- Tachycardia and arrhythmia Must wait 2 weeks after stopping
out" period MAOI before starting new drug
Used for smoking cessation Increases NE and dopamine by
Bupropion decreasing reuptake NDRI (NE and Dopamine reuptake inhibitor)
Lowers seizure threshold NO sexual dysfunction
Alpha-2 antagonist (pre-synaptic nerve terminals of adernergic
Mirtazapine neurons) Increase NE and 5-HT Also potent 5-HT2 & 5-HT3
receptor blockers Sedation, Increased appetite, weight gain
Blocks 5-HT2 and alpha-1 receptors Good for Insomnia Side
Trazodone
effect: priapism, sedation, postural hypotension
What is the MOA of each of the
5-HT inhibition re-uptake
following medication classes? SSRI
What is the MOA of each of the
Inhibit serotonin and NE re-uptake
following medication classes? SNRI
What is the MOA of each of the
Inhibit serotonin and NE re-uptake (older)
following medication classes? TCA
What is the MOA of each of the Nonselective MAO inhibition which increases levels of amine
following medication classes? MAOI neurotransmitter (NE, 5-HT, and Dopamine)
SSRI drugs (5) Fluoxetine, Sertraline, Paroxetine, Citalopram, Fluvoxamine
Notriptyline, Imipramine, Amitriptyline, Desipramine,
TCA drugs
Clomipramine, Doxepin
MAOI drugs (3) Selegiline, Tranylcypromine, Phenelzine
NDRI drugs Bupropion
SNRI drugs (3) Venlafaxine and Duloxetine, Milnacipran
Tetracyclic drugs (2) Mirtazapine and Trazodone
Which antidepressant matches each of
Trazodone
the following statements? SE:priapism
Which antidepressant matches each of
the following statements? Lowers the Burpropion and TCAs
seizure threshold
Which Rx matches each of the
following statements? Works well with Trazadone
SSRIs and increases REM sleep
Hyperthermia Myoclonus Hyperreflexia Flushing and Diarrhea
What are the symptoms of serotonin
Autonomic instability (HTN and Increased HR) Mental status
syndrome?
change Neuromuscular abnormalities
Which antidepressant matches each of
the following statements? Appetite
Mirtazapine
stimulant that is likely to result in
weight gain
Which antidepressant matches each of
the following statements? Bedwetting TCAs (imipramine)
in children
What are the symptoms of TCA Convulsions Coma Cardiotoxicity (arrhythmias) Respiratory
overdose? Tri-Cs depression Confusion and hallucination Hyperpyrexia
Which antidepressant matches each of
the following statements? Smoking Bupropion
cessation
What happens if you ingest tyramine
Hypertensive crisis Hemorrhagic stroke Cardiac arrythmias
while on MAOIS?
What drug is associated with the
following side effects? Dry mouth, Amitriptyline (due to anticholinergic effects)
tinnitus, blurred vision, mania
Tendon xanthomas Familial hypercholesteremia
Cafe-au-lait spots NF-1 and Mccune albright
Tuft of hair on lower back Spina bifida occulta
Axillary nerve Posterior circumflex a. Supraspinatus tendon
What structures are at risk for injury
Anterior glenohumoral ligament Glenoid labrum Posterolateral
with an anterior shoulder dislocation?
humoral head
What are the layers of the epidermis Corneum Lucidum Granulosum Spinosum Basalis
Thrush found in top of mouth and tongue Can be scraped off
Candida Albicans (leave bleeding mucosa) Common in immunocompromised
(AIDS, diabetes, steroids, neonates)
Caused by Candida albicans Severe itching Edema and discomfort
Vulvovaginitis
Vaginal discharge Acidic env
Candida albicans deep infections? Common in Drug users Endocarditis Dissemiated candidiasis
Cause what? Chronic mucocutaneous candidasis (immundef)
When placed in serum: Develops germ tubes at 37 degrees
Candida albicans diagnosis
Pseudohypahe and budding yeasts at 20 degrees
Topical azloe- vaginal Nystatin, Fluconazole, or Caspofungin -
Candida albicans treatment oral/esophageal Caspofungin, Fluconazole, and amphotericin B-
systemic
Cryptococcus neoformans How does it
Heavily encapsulated Narrow based-budding- Rep. Found in soil
replicate? Where is it found? Cultured
and pigeon dropping Cultured on Sabouraud agar
on?
Cryptococcus neoformans diagnosed
India Ink (stains polysaccharide capsule) Mucicarmine (stains red)
using
Cryptococcus neoformans antigen test Latex agglutination test detects polysaccharide capsular antigen
used and is more specific
Cryptococcus neoformans lesions look
Soap Bubble in brain (Cryptococcal meningoencephalitis)
like? What is the disease assc.
Combo of Amphotericin B + Flucytosine and then followed by
Cryptococcal meningitis treatment
single-therapy Fluconazole
Pneumocystis pneumonia (PCP) Diffuse intersitisl pneumonia
Pneumocystis jirovecii What disease
Aytpical pneumonia (walking pneumonia) Asymptomatic
does it cause? Who does it affect?
normally Symptomatic for Immunosuppressed
Identified in tissue or bronchial lavage using silver stain Chest
Pneumocystis jirovecii diagnosis
imaging shows: Ground-glass opacities of interstital infiltrates
Pneumocystis jirovecii treatment TMP/SMX (Bactrim)
When do you start PCP prophylaxis on
Start when CD4+ count drops <200 cells/mm3
HIV patients? (TMP-SMX)
Can cause: Allergic reactions (ABPA) Colonized = Aspergilloma
Aspergillus fumigatus (MOLD)
(fungus ball) after Tb Invasive aspergillus (immunocompromised)
Narrow septate hyphae that branch at acute angles (less than 45
Aspergillus fumigatus diagnosis
degrees)
Aspergillus fumigatus treatment Combo ("-Azole" + Amphotericin B)
Cause mucormycosis Cause disease in severely
Mucor and Rhizopus
immunocompromised
Broad Irregular shaped, nonseptate hyphae branching at WIDE
Mucor and Rhizopus diagnosis angles (90 degrees) -Ribbon-like Penetrate cribiform plate to enter
bloodstream
Patients with diabetic ketoacidosis or Rhinocerebral frontal lobe abcess: Facial pain and headache Black
leukemia are prone to develop necrotic eschar on face Cranial nerve deficits and blindness
Surgical debriment and amphotericin B High mortality
Mucor and Rhizopus treatment
Progression rapid
Branching septate hyphae visible on KOH prep. with blue fungal
Dermatophytes (diagnosis)
stain
What are 4 molds that are considered
dermatophytes (fungal species Trichophytin Microsporum (most prevalent) Epidermophytin
commonly found invading superficial Malassezia furfur
layers of skin)
What infections are commonly caused Tina pedis- feet Tina cruis- groin Tinea corporis- body (ringworm)
by dermatophytes Tinea capitis- head (ringworm
Simple cutaneous dermatophytosis Topical Terbinafine/Azoles Extensive infection or that involve
treatment scalp and hair require oral Terbinafine/Azoles
Tinea unguium-(onchomycosis)-nails Caused by Trichophyton rubrum Thickened discolored nails
Treatment Terbinafine/Azoles (oral)
Caused by Malassezia furfur Lipophillic yeast
Tinea Vesicolor Caused by? Type of
Hypopigmented/hyperpigmented patches on body (back and
yeast? Causes? Occurrence?
shoulders) Effects trunk and proximal limbs Hot, humid weather
Tinea Vesicolor diagnosis and KOH prep: spaghetti and meatballs look" Treat with topical azole
treatment (selenium sulfide)
Systemic MYCOSES Causes? Type of Pneumonia and disseminate Dimorphic fungi NO person to person
fungi? transmission Granuloma formation (noncaseseating)
Systemic mycoses treatment? Local:
Local: Fluconazole/itraconazole Systemic: Amphotericin B
Systemic:
Macrophages filled with Histoplasma (smaller than RBC)
Histoplama capsulatum "Histo hides"
Hepatosplenomegaly
Inhaled Can disseminate to skin and bones Forms granulomatous
Blastomyces dermatitidis
nodules
Exists as a spherule (filled with endospores) Arthroconidia (barrel
Coccidioides immitis
shape) Bigger than RBC
Coccidioides immitis can cause if
Desert bumps= erythema nodosum Desert rheumatism= arthralgias
severe
Inhaled Disseminates widely Severe pneumonia, infection of
Paracoccidioides brasiliensis
mucus membranes and skin Captain Wheels formation
Sporotrichosis Dimorphic fungus Lives in soil and on vegetation
Sporothrix schenckii Can cause
Rose garderner's disease
Caused by pricks from thorns or sphagnum moss Small
Rose garderner's disease How is it
pustule/ulcers Nodules (painless) also form along draining
caused? What forms
lymphatics
Sporotrichosis treatment Itraconazole or Potassium iodide
Found in SW U.S. including West
Coccidiomyces
Texas and California
Found in Mississippi and Ohio River
Histoplasma
basins
Found in rural Latin America Paracoccidiomyces
Associated with plant thorns and
Sporothrix schenckii
cutaneous injury
Causes San Joaquin Valley fever Coccidiomyces
Found in states east of the Mississippi
Blastomyces
River (Central America-Canada)
Found in bird and bat droppings Histoplasma
Causes thrush in immunocompromised
Candida albicans
patients and vulvovaginitis in women
Mold form contains barrel-shaped Coccidiomyces
arthroconidia
Multiple budding of yeast form Paracoccidiomyces b.
Opportunistic mold with septate
Aspergillus fumigatus
hyphae that branch at a 45 angle
Yeast known for causing meningitis in
Cryptococcus meningitis
AIDS patients
Opportunistic mold with irregular
nonseptate hyphae that branch at wide Mucor and Rhizopus
angles (>90 degrees)
Associated with dust storms and
Coccidiomyces
earthquakes
An HIV (+) patient with CSF showing
75/mm3 lymphocytes suddenly dies.
Cryptococcus neoformans
Yeast is identified in the CSF. What is
the most likely diagnosis?
Broad-based budding of yeast Blastomyces
Histoplasma capsulatum Cryptococcus neoformans
What infections are associated with
Chlamydophila psittaci H5N1 Influenza (Bird Flu) West Nile
birds
Virus
Causes diaper rash Candida albicans Erythematous skin Satellite lesions
Known for causing pneumonia in
AIDS patients start Bactrim Pneumocystis jirovecii
prophylaxis when CD4 <200
Most common opportunistic infection
Pneumocystis jirovecii
in HIV patients
Prophylaxis for Cryptococcus in AIDS
Fluconazole
patients
Prophylaxis for PCP in AIDS patients TMP-SMX
Treatment for oral candidiasis Nystatin/Fluconazole
Treatment for systemic candidiasis Amphotericin B
Candida albicans Aspergillus Cryptococcus neoformans Mucor &
Opportunistic fungal infections
Rhizopus spp.
What is the classic clinical presentation Presents as anterior midline neck mass that moves with
of a thyroglossal duct cyst swallowing or protrusion of tongue (asymptomatic) <30 yrs Can
become infected (abcess)
What conditions are associated with an Most anemias Infections (osteomyletis) Polymyagia rheumatica
elevated ESR (erythrocyte Inflammation (Temporal arteritis) Cancer Pregnancy Autoimmune
sedimentation rate) disorders (Rheumatoid)
What is the clinical use for tiotropium Anti-muscuranic Respiratory COPD, asthma Relax airway
when reaction velocity is directly proportional to concentration of
1st order velocity/kinetics
substrate
is when velocity is completely independent of concentration of
Zero order velocity/kinetics
substrate
maximum velocity the reaction can achieve Proportional to the
Vmax
amount of enzymes available
substrate concentration at 1/2 of Vmax Inversely related to the
Km
affinity of the enzyme for its substrate
x-axis: 1/S y-axis: 1/V Slope = Km/Vmax X-intercept: 1/-Km Y-
Lineweaver-Burk plot
intercept: 1/Vmax
Increased y-intercept does what to
Decreases Vmax
Vmax
A shift to the right on the x-intercept
(closer to zero) does what to Km and increases Km and Lowers affinity
affinity
Competitive inhibitor does what to
it does nothing to Vmax and increases Km
Vmax
Noncompetitive inhibitor does what to
it decrease Vmax and does nothing to Km
Vmax
1) Volume of Distribution= Amt of drug in body/plasma drug conc
What 4 pharmacokinetics equations are 2)Clearance= Rate of elimin/plasma drug conc= (0.7 x Vd/half-
most important to know for Step 1 life) 3) Loading Dose= Conc at steady state X Vd 4)Maintenance
dose= Conc at steady state x Clearance
the time required to change the amount of drug in the by body
Half-life
HALF during elimination or constant infusion
94% steady state concentration can be
4 Half-lives
achieved after how many half lives
Efficacy the maximal effect a drug can produce
Potency amount of drug needed for a given effect
Shifts curve to right and lowers potency no change in efficacy
Competitive anatagonist
Increased Km & same Vmax
Noncompetitive antagonist or Shifts curve down (decrease efficacy) Decrease Vmax and same
Irreversible antagonist Km
Shifts curve down and left (decrease efficacy but increased
Partial agonist
potency) Decrease Vmax and Km
What variable can you not compare
potency, because it is an independent variable
between Agonist and Partial agonist
LD50= lethal dose to about 50% of people
Therapeutic index (TI)= _____________________________________________ ED50=
effective dose to about 50% of people
Safer drugs have a Higher therapeutic index (Increased LD50 or Decreased ED50)
Examples of drugs with low TI? (5) Digioxin Lithium Theophylline Warfarin Seziure drugs
Measure of clinical drug effectiveness for a patient Higher
Therapeutic window
therapeutic window= safer the drug
How do you decreased target
you stop infusion of drug for 1 half-life
concentration by 1/2
If clearance is decreased, but Vd is
loading Dose would remain unchanged, but Maintenance dose
unchanged, How does that effect
would decrease.
loading dose and maintenance dose?
Where would you expect to find B
B-Cells located in Follicle of lymph node
cells in a lymph node
T-Cell located in Paracortex of lymph node macrophages located
Where would you find T cell in Medullary Sinus of lymph node Plasma cells located in
Medullary Cords of lymph node
Where would you find macrophages macrophages located in Medullary Sinus of lymph node
Where would you find plasma cells plasma cells located in Medullary Cords of lymph node
The protein derived from what gene
FOXP3- Found on X chromosome Codes for Forkhead Box
serves as a transcription factor for the
Protein P3 Important for self-tolerance Absence= autoimmune
development and function of
diseases
regulatory T cells
What is the female homologue to each
Vestibular bulbs
of the following male structures?
Corpus spongiosum
What is the female homologue to each
of the following male structures? Greater vestibular glands of Bartholin
Cowper's glands (bulbourethral glands)
What is the female homologue to each
of the following male structures? Urethral and paraurethral glands of Skene
Prostate
What is the female homologue to each
of the following male structures? Glans Glans clitoris
penis
What is the female homologue to each
of the following male structures? Labia minora
Ventral penile shaft
What is the female homologue to each
of the following male structures? Labia majora
Scrotum
What important intracellular proteins
are common to both the extrinsic and Cytosolic caspases
intrinsic apoptotic pathways?
What ratio indicates fetal lung
maturity? What is the proper name for Lecithin:sphingomyelin ratio (>2:1) Surfactant main component =
the main component of pulmonary lecthins (dipalmitoylphosphatidylcholine)
surfactant?
What amino acids are necessary for
Glycine Aspartate Glutamine (GAG)
purine synthesis
What nutrient deficiency is associated
with spooning of the nails iron
(koilonychia)
What is the equation for determining a
CL= (0.7) x Vd ----------------- Half-Life
drug's clearance
Which bacteria are well known for
Rickettsia Chlamydia Coxiella
being obligate intracellular bacteria
Why can't obligate intracellular
because they rely on host ATP, can't make it on their own
bacteria replicate extracellularly
What is compartment syndrome, and Trauma/Burn/intense excercise to leg: Increased pressure within
how is it treated the fascial compartment Treatment: Fasciotomy
Separates the greater and lesser sacs Gastrohepatic ligament (right) Gastrosplenic ligament (left)
May be cut during surgery to access
Gastrohepatic ligament
the lesser sac
2 ligaments that connect the spleen to
Gastrosplenic and Splenorenal ligaments
other structures
Contains the portal triad Heaptoduodenal ligament
Connects liver to the anterior
Falciform ligament
abdominal wall
Where are neurotrasmitters made RER
infant with failure to thrive,
hepatosplenomegally, Neimann-Pick disease (genetic sphingomyelinase deficiency)
neurodegeneration
infant with hypoglycemia, failure to
Cori's disease (debranching enzyme deficiency)
thrive, and hepatomegaly
infant with microcephaly, rocker-
bottom feet, clenched hands, and Edward's syndrome (trisomy 18)
structural heart defect
infant with failure to thrive,
hepatosplenomegally, Neimann-Pick disease (genetic sphingomyelinase deficiency)
neurodegeneration
infant with hypoglycemia, failure to
Cori's disease (debranching enzyme deficiency)
thrive, and hepatomegaly
infant with microcephaly, rocker-
bottom feet, clenched hands, and Edward's syndrome (trisomy 18)
structural heart defect
Jaundice, RUQ pain, fever Charcot's triad (ascending cholangitis)
Keratin pearls on a skin biopsy squamous cell carcinoma (lung and cervix)
erythema chronicum migrans from Ixodes tick bite (Lyme
large rash with bull's eye appearance
Disease: Borrelia)
Lucid interval after traumatic brain
epidural hematoma (middle meningeal artery rupture)
injury
male child, recurrent infections, no Bruton's disease (X-linked agammaglobulinemia)
mature B cells
Mucosal bleeding and prolonged Glanzmann's thrombasthenia (defect in platelet aggregation due to
bleeding time lack of GpIIb/IIIa)
Multiple colon polyps, osteomas/soft
tissue tumors, impacted/supernumerary Gardner's syndrome (subtype of FAP)
teeth
Necrotizing vasculitis (lungs) and Wegener's (c-ANCA positive) and Goodpasture's syndromes (anti-
necrotizing glomerulonephritis basement membrane antibodies)
Neonate with arm paralysis following
Erb-Duchenne palsy (superior trunk [C5-C6] brachial plexus
difficult birth or following clavicle
injury: "waiter's tip")
fracture
no lactation postpartum, absent
Sheehan's syndrome (pituitary infarction)
menstruation, cold intolerance
Nystagmus, intention tremor, scanning
speech, bilateral internuclear Multiple Sclerosis (Charcot's triad)
ophthalmoplegia
Cheyne-Stokes respirations (central apnea in CHF or increased
Oscillating slow/fast breathing
intracranial pressure)
painful blue fingers/toes, hemolytic cold agglutinin disease (autoimmune hemolytic anemia caused by
anemia Mycoplasma pneumoniae, infectious mononucleosis)
painful, pale, cold fingers/toes Raynaud's syndrome (vasospasm in extremities)
painful, raised red lesions on palms and
Osler's node (infective endocarditis)
soles
painless erythematous lesions on palms
Janeway lesions (infective endocarditis)
and soles
painless jaundice cancer of the head of the pancreas obstructing the bile duct
palpable purpura, joint pain, abdominal Henoch-Schonlein purpura (IgA vasculitis affecting skin and
pain (child) kidneys)
pancreatic, pituitary, parathyroid
Wermer's syndrome (MEN I)
tumors
precocious puberty, cafe-au-lait spots, McCune-Albright syndrome (a form of Polyostotic fibrous
multiple unilateral bone lesions dysplasia)
pink complexion, dyspnea, "pink puffer" (emphysema: centroacinar [smoking], panacinar
hyperventilation [alpha1-antitrypsin deficiency])
polyuria, acidosis, growth failure,
Fanconi's Syndrome (proximal tubular reabsorption defect)
electrolyte imbalances
Positive anterior "drawer sign" Anterior cruciate ligament (ACL) injury
ptosis, miosis, anhidrosis Horner's Syndrome (sympathetic chain lesion)
pupil accommodates but doesn't react Argyll Robertson pupil (neurosyphilis)
Rapidly progressive leg weakness that
Guillain-Barre syndrome (autoimmune acute inflammatory
ascends (following GI/upper
demyelinating polyneuropathy)
respiratory infection)
Secondary Syphilis, Rocky Mountain Spotted Fever, Coxsackie
Rash on palms and soles
Virus
Recurrent colds, unusual eczema, high Job's syndrome (hyper-IgE syndrome: neutrophil chemotaxis
serum IgE abnormality)
Red "currant jelly" sputum in alcoholic
Klebsiella pneumoniae
or diabetic patients
Red, itchy, swollen rash of
Paget's disease of the breast (represents underlying neoplasm)
nipple/areola
red urine in the morning, fragile RBCs paroxysmal nocturnal hemoglobinuria
renal cell carcinoma,
von Hippel-Lindau disease (dominant tumor suppressor gene
hemangioblastomas, angiomatosis,
mutation)
pheochromocytoma
resting tremor, rigidity, akinesia,
Parkinson's disease (nigrostriatal dopamine depletion)
postural instability
restrictive cardiomyopathy (juvenile
form: cardiomegaly), exercise Pompe's disease (lysosomal glucosidase deficiency)
intolerance
retinal hemorrhages with pale center Roth's spots (bacterial endocarditis)
Crigler-Najjar syndrome (congenital unconjugated
severe jaundice in neonate
hyperbilirubinemia) and lots of other causes
Severe RLQ pain with rebound
McBurney's sign (appendicitis)
tenderness
short stature, increased incidence of
Fanconi's anemia (genetically inherited; often progresses to AML)
tumors/leukemia, aplastic anemia
single palm crease Simian Crease (Down Syndrome)
Situs inversus, chronic sinusitis,
Kartagener's syndrome (dynein defect affecting cilia)
bronchiectasis
Addison's disease (primary adrenocortical insufficiency of
skin hyperpigmentation
autoimmune or infectious etiology)
Slow, progressive muscle weakness in Becker's muscular dystrophy (X-linked, defective dystophin; less
boys sever than Duchenne's)
small, irregular red spots on
buccal/lingual mucosa with blue-white Koplik spots (measles)
centers
smooth, flat, moist white lesions on
condylomata lata (secondary syphilis)
genitals
splinter hemorrhages in fingernails bacterial endocarditis
Scarlet fever, Kawasaki disease, toxic shock syndrome=streak
"Strawberry tongue" ovaries, congenital heart disease, horseshoe kidney=turner
syndrome (XO, short stature, webbed neck, lymphedema)
sudden swollen/painful big toe joint,
gout/podagra (hyperuricemia)
tophi
swollen gums, mucous bleeding, poor scurvy (vitamin C deficiency: can't hydroxylate proline/lysine for
wound healing, spots on skin collagen synthesis)
osteoarthritis (osteophytes on PIP [Bouchard's nodes], DIP
swollen, hard, painful finger joints
[Heberden's nodes])
systolic ejection murmur (crescendo-
aortic valve stenosis
decrescendo)
Thyroid and Parathyroid tumors,
Sipple's Syndrome (MEN 2A)
pheochromocytoma
Toe extension/fanning upon plantar
Babinski's sign (UMN lesion)
scrape
Unilateral facial drooping involving
Bell's palsy (LMN CN VII palsy)
forehead
Urethritis, conjunctivitis, arthritis in a
reiter's syndrome (reactive arthritis associated with HLA-B27)
male
Hemangioma (benign, but associated with Sturge-Weber
Vascular birthmark (port-wine stain)
syndrome)
Vasculitis from exposure to endotoxin Shwartzman reaction (following second exposure to endotoxin)
causing glomerular thrombosis
Vomiting blood following
Mallory-Weiss syndrome (alcoholic and bulimic patients)
esophagogastric lacerations
"Waxy" casts with very low urine flow chronic end-stage renal disease
WBC casts in urine acute pyelonephritis
weight loss, diarrhea, arthritis, fever,
Whipple's disease (tropheryma whippelii)
adenopathy
"worst headache of my life" subarachnoid hemorrhage
WBCs in urine acute pyelonephritis and cystitis
Autosomal Chromosome 17 mutation
in tumor suppressor gene that regulates Neurofibromatosis type I
"ras" signaling
vitamin K dependent clotting factors II, VII, IX, and X as well as proteins C and S
anticentromere antibodies Scleroderma (CREST)
Antidesmoglein (epithelial) antibodies pemphigus vulgaris (blistering)
anti-glomerular basement membrane
Goodpasture's syndrome (glomerulonephritis and hemoptysis)
antibodies
Drug-induced SLE (hydralazine, isoniazid, phenytoin,
antihistone antibodies
procainamide)
Rheumatoid arthritis (systemic inflammation, joint pannus,
anti-IgG antibodies (IgM)
boutonniere deformity)
Primary Biliary cirrhosis (female, cholestasis, portal hypertension)
antimitochondrial antibodies (AMAs)
(also pANCA present)
vasculitis (c-ANCA: Wegener's; pANCA: microscopic
antineutrophil cytoplasmic antibodies
polyangiitis, Churg-Strauss syndrome, Pauci-immune crescentic
(ANCAs)
glomerulonephritis)
Antinuclear antibodies (ANAs: anti-
SLE (type III hypersensitivity)
Smith and anti-dsDNA)
Antiplatelet antibodies Idiopathic thrombocytopenic purpura (ITP) (bleeding diathesis)
Anti-topoisomerase antibodies diffuse systemic scleroderma (not CREST)
anti-transglutaminase antibodies Celiac disease (diarrhea, distention, weight loss)
antigliadin antibodies Celiac disease (diarrhea, distention, weight loss)
anti-endomysial antibodies celiac disease (diarrhea, distention, weight loss)
azurophilic granular needles in auer rods (acute myelogenous leukemia: especially the
leukemic blasts promyelocytic type)
"Bamboo spine" on x-ray ankylosing spondylitis (chronic inflammatory arthritis: HLA-B27)
Basophilic nuclear remnants in RBCs Howell-Jolly bodies (due to splenectomy or nonfunctional spleen)
Basophilic stippling of RBCs Lead poisoning or siderblastic anemia
Bloody tap on LP Subarachnoid hemorrhage
"Boot-shaped" heart on x-ray Tetralogy of Fallot, RVH
Branching gram-positive rods with
actinomyces israelii
sulfur granules
pancoast's tumor (can compress sympathetic ganglion and cause
bronchogenic apical lung tumor
Horner's syndrome)
Hemorrhage (hemosiderin) causes brown color of osteolytic cysts.
"Brown" tumor of bone
Due to: 1. hyperparathydoidism 2. Osteitis fibrosa cystica
Cardiomegaly with apical atrophy Chagas' disease (typanosoma cruzi)
cellular crescents in Bowman's capsule rapidly progressive crescentic glomerulonephritis
"chocolate cyst" of ovary endometriosis (frequently involves both ovaries)
circular groupoing of dark tumor cells Homor Wright rosettes (neuroblastoma, medulloblastoma, Ewing
surrounding pale neurofibrils Sarcoma)
Colonies of mucoid Pseudomonas in Cystic fibrosis (CFTR mutation in Caucasians resulting in fat-
lungs soluble vitamin deficiency and mucous plugs)
abdominal pain, ascites, hepatomegaly Budd-Chiari syndrome (posthepatic venous thrombosis- no JVD)
Achilles tendon xanthoma familial hypercholesterolemia (increased LDL leads to deposits)
Adrenal hemorrhage, hypotension, DIC Waterhouse-Friderichsen syndrome (meningococcemia)
arachnodactyly, lens discoloration,
Marfan's syndrome (fibrillin defect)
aortic dissection, hyperflexible joints
athlete with polycythemia erythropoietin injection
back pain, fever, night sweats, weight
Pott's disease (vertebral tuberculosis) with caseating granulomas
loss
Bilateral hilar adenopathy, uveitis
blue sclera Osteogenesis imperfecta (type I collagen defect)
bluish line on gingiva Burton's line (lead poisoning)
bone pain, bone enlargement, arthritis, PAGET'S DISEASE of bone (increased osteoblastic and
increased alk phos osteoclastic activity)
Bounding pulses, diastolic heart
aortic regurgitation
murmur, head bobbing
Cafe-au-lait spots, Lisch nodules (iris Neurofibromatosis type I (+pheochromocytoma, optic gliomas)
hamartoma) Neurofibromatosis type II (+bilateral acousitc neuromas)
Cafe-au-lait spots, polyostotic fibrous McCune-Albright syndrome (mosaic G-protein signaling
dysplasia, precocious puberty mutation)
Calf pseudohypertrophy muscular dystrophy (most commonly Duchenne's)
Tay-Sachs (ganglioside accumulation) or Niemann-Pick
"Cherry-red spot" on macula
(sphingomyelin accumulation), central retinal artery occlusion
Chest pain, pericardial effusion/friction Dressler's syndrome (autoimmune-mediated post-MI fibrinous
rub, persistent fever following MI pericarditis, 1-12 weeks after acute episode)
Gowers' sign (Duchenne muscular dystophy: X-linked recessive
Child uses arms to stand up from squat
deleted dystrophin gene)
Child with fever develops red rash on "Slapped cheeks" (erythema infectiosum/fifth disease: parvovirus
face that spreads to body B19)
chorea, demetnia, caudate degeneration Huntington's disease (autosomal-dominant CAG repeat expansion)
chronic exercise intolerance with
McArdle's disease (muscle phosphorylase deficiency)
myalgia, fatigue, painful cramps
Cold intolerance hypothyroidism
conjugate lateral gaze palsy, horizontal internuclear ophthalmoplegia (damage to MLF; bilateral [multiple
diplopia sclerosis], unilateral [stroke])
continuous "machinery" heart murmur PDA (close with indomethacin; open with misoprostol)
cutaneous/dermal edema due to
myxedema (hypothyroidism, Graves' disease)
connective tissue deposition
Kaposi's sarcoma (usually AIDS patients [gay men]: associated
Dark purple skin/mouth nodules
with HHV-8)
Deep, labored
Kussmaul breathing (diabetic ketoacidosis)
breathing/hyperventilation
Dermatitis, dementia, diarrhea, death pellagra (niacin [vitamin B3] deficiency)
dilated cardiomyopathy, edema, wet beriberi (thiamine [vitamin B1] deficiency) -> dry is without
polyneuropathy dilate cardiomyopathy and edema
dog or cat bite resulting in infection Pasteurella multocida (cellulitis at inoculation site)
dry eyes, dry mouth, arthritis Sjogren's syndrome (autoimmune destruction of exocrine glands)
Dysphagia (esophageal webs), Plummer-Vinson syndrome (may progress to esophageal
glossitis, iron deficiency anemia squamous cell carcinoma)
elastic skin, hypermobility of joints Ehlers-Danlos Syndrome (collagen defect, usually type III)
enlarged, hard left supraclavicular node virchow's node (abdominal metastasis)
erythroderma, lymphadenopathy, Sezary syndrome (cutaneous T-cell lymphoma) or mycosis
hepatosplenomegaly, atypical T cells fungoides
facial muscle spasm upon tapping Chvostek's sign (hypocalcemia)
fat, female, forty, and fertile Chvostek's sign (hypocalcemia)
fever, chills, headache, myalgia
Jarisch-Herxheimer reaction (rapid lysis of spirochetes results in
following antibiotic treatment for
toxin release)
syphilis
Fever, cough, conjunctivits, coryza,
measles (morbillivirus)
diffuse rash
fever, night sweats, weight loss B symptoms (lymphoma) or TB
Fibrous plaques in soft tissue of penis Peyronie's disease (connective tissue disorder)
gout, mental retardation, self-
Lesch-Nyhan syndrome (HGPRT deficiency, X-linked recessive)
mutilating behavior in a boy
Green-yellow rings around peripheral Kayser-Fleischer rings (copper accumulation from Wilson's
cornea disease)
Hamartomatous GI polyps,
Peutz-Jeghers syndrome (genetic benign polyposis can cause
hyperpigmentation of
bowel obstruction; increase cancer risk
mouth/feet/hands)
Hepatosplenomegaly, osteopososis,
Gaucher's disease (glucocerebrosidase deficiency)
neurologic symptoms
Hereditary nephritis, sensorineural
Alport's syndrome (type IV collagen mutation)
hearing loss, cataracts
hypercoagulability (leading to
Trousseau's sign (adenocarcinoma of pancreas or lung)
migrating DVTs and vasculitis)
Hyperphagia, hypersexulaity,
Kluber-Bucy syndrome (bilateral amygdala lesion)
hyperorality, hyperdocility
Hypertension, hypokalemia, metabolic
Conn's syndrome (primary hyperaldosteronism)
alkalosis
hypoxemia, polycythemia, hypercapnia "blue bloater" (chronic bronchitis: hyperplasia of mucous cells)
NONPAINFUL: chancre (primary syphilis, Treponema pallidum);
indurated, ulcerated genital lesion PAINFUL: painful, with exudate: chancroid (Haemophilus
ducreyi)
Degeneration of dorsal column nerves tabes dorsalis dorsalis (tertiary syphilis)
Depigmentation of neurons in Parkinson's disease (basal ganglia disorder: rigidity, resting
substantia nigra tremor, bradykinesia)
desquamated epithelium casts in curschmann's spirals (bronchial asthma; can result in whorled
sputum mucous plugs)
disarrayed granulosa cells in
Call-Exner bodies (granulosa-theca cell tumor of the ovary)
eosinophilic fluid
back pain, fever, night sweats, weight
Pott's disease (vertebral TB)
loss
big toe extension/fanning upon plantar
Babinski's sign (UMN lesion --> spastic paralysis)
scrape
Bilateral hilar adenopathy, uveitis sarcoidosis (noncaseating granulomas)
elevated D-dimers can be due to... thrombosis, DIC, PE, DVTs, Budd Chiari
Budd-Chiari syndrome (posthepatic venous thrombosis - looks like
abdominal pain, ascites, hepatomegaly
CHF but no JVD)
Bilateral hilar adenopathy, uveitis sarcoidosis (noncaseating granulomas)
bluish line on gingiva Burton's line (lead poisoning)
What is deficient in I- cell disease? Phosphitransferase - tags enzymes with mannose 6 phosphate
what bacterial structure is the space
between the inner and outer cell walls periplasmic space
of gram negative bacteria
what bacterial form provides resistance
(endo)spore
to dehydration, heat, and chemicals
what bacterial structure forms
(F or sex) pilus
attachment between two bacteria
during conjugation (transfer of DNA
material)
what bacterial structure is genetic
material within the bacteria that plasmid
contains genes for antibiotic resistance
what stain is required to see chlamydia giemsa stain
treponema (too thin), legionella (fatty acids), mycoplasma (no cell
which organisms are not well
wall), mycobacteria (high lipid content) [also chlamydia and
visualized with gram stain
rickettsia]
By what method are plasmids
conjugation
exchanged between bacteria
which exotoxin inhibits acetylcholine
release at the neuromuscular junction, botulinum toxin
resulting in flaccid paralysis
which exotoxin is a phospholipase that
alpha toxin of clostridium perfringens
causes gas gangrene
which exotoxin inhibits the inhibitor of
adenylyl cyclase, causing whooping pertussis toxin
cough
which exotoxin stimulates adenylyl
cyclase, resulting in chloride and water
cholera toxin, heat labile ETEC toxin
entering the intestinal lumen, causing
diarrhea
which exotoxin destroys leukocytes PV leukocidin and gamma-hemolysin of staph aureus
which exotoxin is composed of edema
factor, lethal factor, and protective anthrax toxin of bacillus anthracis
antigen
A group of disorders caused when something disrupts the
Myelodysplastic Syndrome
production of blood cells
70yo M dies in a motor vehicle
collision. Was undergoing evaluation tubular adenoma
for occult blood in the stool. Dx?
38yo M with 1-week hx of watery,
itchy eyes and a runny nose. Physical Loratidine
shows inflamed nasal mucosa. No
congestion in lower lung.
Pharmacotherapy?
24yo M with small tender blisters on
his penis 3 days after unprotected sex. Herpes simplex virus type 2
Photograph shown. Causal agent?
42yo F with 3-year hx of an
intermittent facial rash, including the
forehead, eyelids, nose, and cheeks.
Rash seems to be getting worse since
Rosacea
she moved from New York to Florida
last year. Spicy foods precipitate a
flushing reaction that seems to
exacerba
53yo M returned from Africa, has
fever, headache, and abdominal
discomfort. Received appropriate
Malaria
vaccinations prior to the trip. T 39.4C.
A wright-stained peripheral smear
shown. Dx?
68yo F with T2DM and hypertension
that has even poorly controlled despite
hydrochlorothiazide treatment. BP
150/96, Labs show serum glucose Lisinopril
concentration of 130 and proteinuria.
In addition to current Rx, which is
most appropriate pharmacotherapy?
66yo M with stage IV colon cancer
with 3-day hx of severe diarrhea after
receiving chemotherapy with
flourouracil, leucovorin, and Loperamide
irinotecan. perscribed opioid
antidiarrheal agent with no CNS
effects. Which med?
35yo M in ED with 2-hour hx of sever
fatigue and dizziness. Had profuse,
watery diarrhea for 8 hours despite a Activation of adenylyl cyclase
lack of oral intake. Recently returned
from a medical relief trip to a remove
village in Honduras. T 36.7 C, P
122/min, BP 90/50. PE shows dry s
59yo F with gradual onset of lack of
muscle control in her left arm and leg.
Sx 1 mo ago after dx with metastatic
breast cancer. PE shows ataxia of left
Cerebellum
upper and lower extremities. Muscle
strength, DTR, sensation,
proprioception normal. Metastatic
tumor
Newborn delivered at 38 weeks'
gestation weighs 1800 g. PE shows
petechial rash, microcephaly, and
hepatosplenomegaly. Serologic test for CMV
CMV: IgG + in mother, + in newborn;
IGM - in mother, + in newborn.
Explanation?
Female newborn is delivered at 38
weeks' gestation. Apgar 8 and 8 at 1/5
min. PE shows a bulging, fluod0filled
mass approximately 5 cm in diameter 15 to 40
in the midline over the lumbosacral
region. No spontaneous movements of
the lower extremities. Abnormality m
64yo M in ED 3 hours after SOB with
exertion and extreme fatigue. Has
ischemic heart disease. P 125/min, BP
105/60. ECG shows atrial fibrillation. Torsades de Pointes
Intravenous ibutilide is administered.
Ten minutes later, ECG shows normal
sinus rhythm. Risk for which drug
65yo F with 20-year hx of
osteoarthritis of the hands now has
pain radiating down the distal anterior
thigh, knee, medial leg, and food. Bony L-3 to 4
outgrowth of vertebrae compressing
one of the spinal nerves is suspected.
Nerve root in which intervertebral for
38yo M with 3-year hx of T2DM.
Taking an oral antihyperglycemic
agent, he has tried diet and exercise.
BMI 32. PE normal. Hb A1c is 10%.
Contemplation
Physician recommends initiation of
insulin injections. Responds, "I know
that insulin would help control my
blood suga
24yo M with 2-day history of an itchy
rash on his arms and legs. Returned
from a camping trip in the woods 5
T lymphocytes
days ago. PE shows edematous,
erythematous rash with linear vesicles.
Cause is activation of which cell types?
70yo M from china with poorly
differentiated monoclonal carcinoma of
the nasopharynx. DNA probes of Epstein-Barr virus
neoplastic cells are most likely to
detect genome of which virus?
24yo M with hx of IVDA could not be
aroused. Friend reports that the patient
injected himself with a drug 6 hours
ago. Labs show drug concentration of 480
0.3. Assuming first-order one-
compartment kinetics, has a half-life of
2 hours, and a volume of distribu
Compound is taken up by bacterial
cells. No energy is necessary for
uptake, and the compound is not Carrier-mediated diffusion
concentrated in the cell. Which
describes this mechanism of transport?
Newborn has male genital ducts but
female external genitalia. Cytogenetic
analysis shows a 46,XY karyotype, and
genetic testing shows a mutation of the Scrotum
gene encoding 5alpha-reductase. In
absence of this mutation, labia majora
would have been?
Study designed to evaluate the efficacy
of coenzyme Q10 in improving cardiac
output in patients with CHF. Sixty
Randomized clinical trial
patients with CHF are recruited. Each
assigned by coin toss to one of tw
groups. Design?
12yo boy immersed up to his neck in
Central blood volume DECREASED, ADH (vasopressin)
60F water for 20 minutes.
INCREASED, Atrial Natriuretic Peptide INCREASED??
Physiological changes?
Tells patient he has lung cancer. The
patient reponds, "How can this be "It must be difficult for you to accept this diagnosis when you feel
happening to m? I eat right and healthy."
exercise." Appropriate response?
18mo girl. Separation of the umbilical
cord was delayed after birth. Has had
four severe skin infections
Staphylococcus aureus; No pus Leukocyte adhesion and transmigration
formation at infection sites. Persristent
leukocytosis in absence of infection.
Mechanisms impaired?
32yo M with 6-month hx of low back
pain and stiffness, worse in morning
and improve during the day; the pain
radiates to his buttocks but not down X-rays of the sacroiliac joints
his legs. Back stiffness if he sits for
prolonged periods. Which to confirm
diagnosis?
17yo boy with 8-kg weight gain during
the past year. No medical illness. BMI
"Yes, your weight gain can be caused by genes and environment
is 32. He asks, "Do you think that my
combined."
weight gain is inherited from my
father?" Appropriate response?
52yo M neighbors have reported that
he has been confused and not taking
care of himself. 4-mo Hx of diarrhea.
Pellagra
PE shows extreme muscle wasting,
stomatitis, and a diffuse rash that is
worse in sun-exposed areas.
Diagnosis?
80yo F with suspected temporal
arteritis (TA). ESR is 100, Pretest
probability for TA is 50% in this
patient. In the evaluation of TA, ESR Additional testing to confirm the diagnosis of TA
has a sensitivity of 99% and specificity
of 60%. Based on the restults of the
ESR testing, most appropriate next ste
Sequence surrounding the first two
exons of the human beta-globin gene
shown, with exons in bold. Translation
start codon is underlined. A mutation Disruption of normal splicing by creation of a new 3' splice site??
from G-->A at position 355 is most
likely to lead to beta-thalassemmia by
which mechanism?
42yo M in ED because of a 10-day
history of progressive fever, SOB, and
nonproductive cough. 20-kg weight
loss. Immigrated to USA from the Infection with HIV-2
Ivory Coast 4 years ago. T 38C. Lungs
clear, CXr shows diffuse interstitial
infiltrate. Silver stain obtained via br
27yo F with vaginal bleeding for 3
weeks. First pregnancy ended with a
spontaneous abortion 8 months ago.
No Rx since dilatation and curettage at
Trophoblastic tissue
that time. PE shows enlarged uterus,
beta-hCG markedly increased.
Ultrasonagraphy of pelvis shows
material w
65yo M emigrated from Brazil with 8-
month history of shortness of breath
and fatigue, edema of lower ext. CXR
shows cardiomegaly. endomyocardial
Trypanosoma cruzi
biopsy specimen shows myofiber
necrosis with a mixed inflammatory
infiltrate of PMNs, T lymphos,
m.phages, and
40yo F with mole on her back that has
increased in size during past 4 months.
PE shows raised irregular lesion with
variegated black-tan pigmentation and Melanoma
ill-defiined margins. Pleomorphic,
hyperchromatic cells within clear
islands that tend to collapse. E
42yo farmer has a 7-mm red scaly
plaque on helical rim of left ear. Actinic keratosis
Photomicrograph shown. Dx?
30yo F training for a marathon,
running 20 mild/day. Fasting glucose is
60. After her glucose stores have been Kidney
depleted, which organ, in addition to
liver, releases glucose?
Male newborn at 28 weeks'. Given
ventilatory support with up to 80%
oxygen for the next 72 hours, but dies
Not sure about this one. Looking for surfactant.
of resp. failure. Cause is inadequate
secretion from which labeled cell
types?
9yo girl with poor growth during the
past year. < 3rd %ile for height and at
10th percentile for weight. PE normal.
Visual field testing shows bitemporal Diverticulum of the roof of the embryonic oral cavity
hemianoia. Labs show GH deficiency.
MRI shows calcified cystic mass in
suprasellar region. Tumor deri
28yo F G1P1 with 2-day history of a
painful mass in her right breast.
Delivered healthy female newborn 3
weeks ago, and been breast0-feeding
Staphylococcus aureus
since. T 37C, PE shows 3-cm tender
mass surrounded by an area of
erythema beneath the right areola.
Causal org?
3yo boy with bacterial colitis caused
Interleukin-8 (IL-8)
by Salmonella enterica serovar
arizonae. Which factor accounts for
recruitment of PMNS to inflammatory
site by intestinal epithelial cells?
27yo F with fever, malaise, abdominal
pain, and vaginal d/c for 4 days. LMP
5 days ago. Had ectopic 1 year ago. T
38.3 C, bilateral lower quadrant Gonorrhea
tenderness with rebound and guarding.
Pelvic exam with cervical motion
tenderness and bilateral adnexal tend
20M with 3-month hx of progressive
thirst (drinking a lot of fluids) and
urinary frequency during past 3 days.
Posterior pituitary gland
U/A shows specific gravity less than
1.006. Most likely has dysfunction of
which endocrine structure?
21yo M in ED with excruciating anal
pain for 4 hours. Exam shows 15-mm,
blue tinged rounded mass at anal Inferior rectal vein
margin. Represents thrombus in a
tributary of which blood vessel?
6-week-old girl with 6-day hx of
vomiting small amount of milk 2 to 3
Immature lower esophageal sphincter
times daily. 50 %ile for length and
weight. Cause?
37yo M with 4-day hx of diarrhea and
abdominal pain, worse in past 24h,
with watery-brown stools. Completed a
10-day course of amoxicillin for a
Pseudomemnbranes of fibrin and inflammatory debris
sinus infection 5 days ago. Stool
shows: Fecal fat negative; ova and
parasites negative; Cx for infx
negative;
28yo M in ED 30 minutes after SOB.
3-year hx of cocaine abuse. T 38.1, P
100/min, BP 150/45. PE: diminished Dissecting aneurysm
pulses in left upper extremity. Crackles
heard over all lung fields. 2/6 diastolic
murmur at left sternal border. CXr
shows a widened aortic arch.
1-week-old girl. screening showed a
possible defect in fatty acid oxidation. Measurement of serum acylcarnitine concentrations
PE normal. Next step?
79yoM 30 minutes after LOC for 30
seconds. Alert, but dizzy. No urinary or
fecal incontinence. Pulse 40/min, BP
92/56. PE shows no tongue biting.
Insertion of transvenous pacemaker
Lungs clear, Variable intensity S1.
Oriented X3. ECG shows a third-
degree atrioventricular block. Next
step?
55yo M with chronic bronchitis in ED
after being unresponsive. Found bottles
of albuterol, ampicillin, codeine, and
Naloxone
theophylline bedside. T 37.2 C, p
112/min, respirations are 6/min, BP
95/60. Acute Rx should include?
50yo man has persistent cough for 2
months. He has had a 5 kg (11 lb)
weight loss. He is a farmer and on
itraconazole 4 weeks for
Omeprazole
histoplasmosis from chicken coops.
Medications: hydrochlorothiazide,
enalapril, atenolol, omeprazole, and
metoclopramide for
20yo man with 6-hour difficulty
breathing and vomiting. 10-year
history of type 1 diabetes on insulin.
Pulse 90, respirations 30 and deep, bp
Epinephrine
90/60. Physical shows dehydration.
Labs: Na+ 130, K+ 6.5, HCO3 5,
glucose 500, pH 7.2, pCO2 25 mm Hg.
Which compo
Physician sad to inform patient of
progression of carcinoma to the "Yes, it is."
terminal phase. Physician's face makes
patient cry and ask, "It's bad news, isn't
it?" Which is best response?
75yo man 2-year history of decreased
force of urinary stream, urinate several
times throughout night. BUN 55, Cr
Increased hydrostatic pressure in Bowman space
5.0. Ultrasound shows bilateral
hydronephrosis and dilated ureter.
Mechanism of renal failure?
4yo boy two bacterial urinary tract
infections past year. Physical exam
normal. Radiologic studies show
dilation of left ureter and renal pelvis, Congenital ureteral obstruction
minimal left-sided renal function. Left
nephrectomy done. Photo: dilated renal
pelvis and ureter. Which is ca
45yo man poorly controlled type 2
diabetes 1-month low-grade fever.
Getting hemodialysis for end-stage
renal disease. T 37 C (98.6 F), pulse
Enterococcus faecalis
72, bp 144/92. Physical subclavian
catheter below right clavicle. Lungs
clear. Cardiac exam no murmurs.
Blood cult
3yo boy sickle cell disease with fever
and pain over left foot 3 weeks.
Hematocrit stable. Leukocyte count
Osteomyelitis
15,000 predominance of neutrophils.
Which is most likely explanation for
findings?
45yo woman follow-up exam after 8
weeks tamoxifen therapy for estrogen-
and progesterone-positive invasive
ductal carcinoma of breast. 50yo sister
25%
also hormone-sensitive breast cancer.
Physical exam normal. Serum
decreased concentrations of endoxifen,
act
60yo man progressive shortness of Alveolar macrophage
breath past 3 months. Worked in
shipyard. Respirations 25. Bilateral
basilar crepitant crackles. Xray chest
reticulonodular pulmonary infiltrates
consistent with interstitial fibrosis.
Picture: sputum sample of elongate s
30yo woman 20 weeks gestation,
uncomplicated pregnancy. Fundal
height is greater than expected for
Tracheoesophageal atresia
gestational age. Ultrasound shows
increased amniotic fluid. Which
abnormality is cause?
27yo woman 12-hour history of fever
and abdominal pain. History of
recurrent urinary tract infections.
Temperature is 39 C (102.2 F).
Struvite
Physical exam tenderness of right
flank. Abdominal xrays bilateral
staghorn renal calculi. Urinalysis pH 8,
many RBCs, WB
48yo nulligravid woman with
excessive uterine bleeding for 3
months. Bleeding during menses and at
irregular intervals. Menses were
Endometrial hyperplasia
regular before. BMI 27. Pelvic exam:
adnexae are nonpalpable. Endometrial
curettage shows abundant tissue.
Which is cause o
55yo woman 6-week history low
energy, irritability, crying spells,
difficulty falling asleep, wakes up
during night, cannot focus. Taking
Major depressive disorder
lorazepam for 15y for generalized
anxiety disorder. Taking estrogen
replacement therapy for
postmenopausal symptoms.
Ten years after total gastrectomy, 60yo
man difficulty walking. Diffuse B12 (cobalamin)
spasticity in arms and legs, impaired
proprioception in his feet, increased
muscle stretch reflexes in arms and
knees, absence of muscle stretch
reflexes in ankles, bilateral extensor
80yo man type 2 diabetes 2-month
history severe constipation. Laxatives
haven't relieved symptoms. Abdominal
Pelvic splanchnic
exam shows distention. Colonoscopy
shows no abnormalities. Patient has
dysfunction of which nerve?
34yo man lightheaded after running 12
miles of marathon hot day. Pulse 130 Sympathetic efferent activity increased, parasympathetic efferent
bp 80/60. Which changes to autonomic activity decreased
nervous system occurred?
6-Mercaptopurine (6-MP) used to treat
acute lymphoblastic leukemia (ALL).
6-MP acted on by enzymes to make 6-
thioguanine nucleotides (6-TGN).
They may be given normal doses of 6-MP (?)
Efficacy and toxicity of 6-MP
correlated with 6-TGN. 6-MP acted on
by xanthine oxidase (XO), thiopurine
methyltra
Ten human subjects given new oral
drug to monitor drug effect and
toxicity. Blood analyzed for human Phase I
pharmacokinetics of drug for first time.
Which trial type?
15yo girl health maintenance exam.
Mother dx squamous cell carcinoma
face, maternal grandfather died of
Macrophages
metastatic melanoma. In patient this
age, which factor most predicts
compliance with photoprotection?
44yo woman follow-up after two Pap
smears showing atypical squamous
cells of undetermined significant. Test Ubiquitin ligase
shows viral E6 protein of human
papillomavirus. This protein promotes
cell growth and malignancy by causing
cellular p53 protein degradation. This
40yo woman hx of 6 month episodic
sinusitis with 2-week intermittent
headaches, fatigue, and generalized
joint pain, worsening cough productive
Wegener granulomatosis
of blood-tinged sputum. Failed
antibiotics, decongestants and nasal
corticosteroids. Physical exam:
erythema na
25yo woman 6-month history of joint
pain poorly responsive to aspirin.
Physical exam: bilateral swelling of
proximal interphalangeal joints, Rheumatoid Arthritis
metacarpophalangeal joints, and wrists;
weakness of grasp. Small nodules
palpated beneath skin around joints of f
50yo woman 1-year hx of hot flashes
and irregular menses. Decreased bone
Inhibition of osteoclast-mediated bone resorption
mineral density. Alendronate
prescribed. Mechanism of drug?
Female newborn at 36 weeks gestation
has respiratory distress. Apgar 3 and 5
at 1 and 5 minutes. Physical shows
cyanosis. Endotracheal and NG tubes
Incomplete formation of pleuroperitoneal membrane
placed. Xray shows nasogastric tube in
left hemithorax, mediastinum
displacement to right, absence bowel
ga
65yo women progressive vulvar
itching past 2 months; miconazole for
yeast infections ineffective. Exam:
Lichen sclerosus
atrophy of labia minora and thin,
parchment-like skin over vulva and
anus. Dx?
75yo man 2-day ear ringing, nausea,
fatigue. Temp 37 C (98.6 F), pulse 100, Salicylate poisoning
respirations 24, bp 140/85. Physical:
mild epigastic tenderness. ABG pH
7.42 pCO2 30 pO2 95 HCO3 19. Dx?
15yo girl 1-day hx redness and painful
skin following sunbathing. She used
sunblock. No medications. Physical First-degree burn
exam: severe erythema of back and
extremities, no blisters. Dx?
58yo man supraventricular
tachyarrhythmia refractory to
pharmacotherapy gets ablation of
accessory excitatory pathway in atrial The junction of the superior vena cava and the right atrium
endocardium. Which area should be
avoided to leave sinoatrial (pacemaker)
node intact?
81yo woman massive pulmonary
embolism from deep venous
thrombosis. Platelet count 160,000.
Appropriate pharmacotherapy is
Potentiates the action of antithrombin III
started. One week later, platelets
55,000. Thrombocytopenia most likely
caused by a drug with which of the
following mechanism of act
22yo woman, g1p1, 2-day hx of fever,
severe vaginal bleeding. four days ago
delivered healthy male newborn. Temp
38.1 C (100.6 F). Pelvic exam: open Internal iliac
cervix, heavy vaginal bleeding. US
shows uterus with no placental tissue
or thrombi. If operation required
35yo woman abnormal Pap smear.
Cervical biopsy shows microinvasive
Neoplastic cells in sub-basement membrane connective tissue
cervical carcinoma. Which
microscopic features led to dx?
17yo girl 1-day shortness of breath,
weakness and muscle tenderness. Did
triathlon previous day. BMI 19. Myoglobin
Temperature 38 C (100.4 F),
respirations 20, bp 150/90. Physical
bilateral crackles lower lobes, muscle
tenderness. Creatinine 4. Urinalysis 3+
protei
16yo boy with no signs of puberty. Sex
development Tanner stage 2. Physical
exam: circumcised penis, soft small
tests 5 mL, prostate firm, nontender, no Luteinizing hormone
discharge or lesions. Testosterone low.
Which hormone is cause of decreased
serum testosterone and lac
28yo woman wants to lose weight. She
binges on high-carbohydrate foods 2 to
3 times a week, forcing herself to Parotid gland enlargement
vomit after. BMI 23. Which physical
finding is likely?
15yo girl emigrated from India and
with several lesions on neck for 2
weeks. Physical exam shows
hypopigmented, hypoesthetic area on
Temperature senisitivity
left side of forehead and 4-cm lesions
on neck. Biopsy shows acid-fast
bacilli. Best explanation why the
organism results
59yo man has total thyroidectomy for
4-cm follicular carcinoma of thyroid.
Twelve hours after procedure, has
Decreased parathyroid hormone, decreased calcium
paresthesias of hands and feet. Vitals
stable, carpal spasm on inflammation
of bp cuff. Lab findings?
41yo man with asthma and allergy to
grass pollen wheezes and difficulty
breathing 10 min after mowing lawn. Albuterol
Drug for immediate relief of acute
symptoms?
14yo boy daily headaches for 2
months. Headaches are bilateral aching
Inhaled glue
in temples. "Has not been himself" for
months. Confused, forgetting names,
dates, places, clumsy, frequent falls.
School performance declined. Physical
exam: broad-based ataxic gait. Sl
40yo African American woman 2-
week hx fever, malaise, dyspnea.
Temperature 36.7 C (98 F),
respirations 20. Physical exam:
1,25-Dihydroxycholecalciferol
erythema nodosum, parotid
enlargement, hepatosplenomegaly.
Calcium 16. CT chest bilateral hilar
adenopathy. Increased in which in ser
56yo woman with restrictive
cardiomyopathy, proteinuria, renal
failure. 35-year history of rheumatoid
arthritis. Renal biopsy shows
beta-pleated sheet structure
glomerular deposition of eosinophilic
hyaline material. Congo red statin:
birefringent pattern under polarized
light. Struc
63yo man 3-month hx difficulty
sleeping. Sleeps better upright. HR 90,
bp 110/60. Physical exam: increased Increased capillary hydrostatic pressure
jugular venous pressure, mild ankle
edema. Cause of edema?
14yo boy come to ER 1 hour after
colliding with teammate playing
soccer. Physical exam: edematous
tissues of left eye, mild depression of
Maxillary division of trigeminal nerve
left zygomatic bone. Skin between eye
and upper lip numb. Double vision
look upward. Nerve damaged causing
sensory lo
Maxillary division of trigeminal nerve Family history of a similar illness
18yo man Crohn disease 1-day hx
severe abdominal pain and intermittent
bloody diarrhea. Temperature 38 C T-lymphocyte function
(100.4 F), pulse 98, respirations 18.
Physical exam: draining anal fisutla.
Treatment with antibiotics and
prednisone over next 3 weeks recovers.
Mech
34yo woman with pyelonephritis
treated with bactericidal antibiotic 4
days no improvement. Antibiotic added
Gentamicin
that inhibits binding to 30S ribosome,
blocking protein synthesis
intracellularly. Antibiotic?
3yo boy and his 5yo brother with
recurrent hemarthroses. Both parents
healthy, but mother with two younger
brothers with same sx and maternal Factor VIII (antihemophilic factor)
uncle who died at 8 of mild head
trauma. Partial thromboplastin time is
prolonged. Defect?
6yo boy from Russia with unstable gait
and incoordination for 2 weeks. Pale,
bulky stools for 4 years and two
episodes of bacterial pneumonia and Vitamin E
chronic cough since age 1 year. 3%ile
for height/weight. Neuro exam shows
ataxia, no DTRs, loss of propriocep
54yo F 1 week after sudden loss of
vision in left eye, returned within 1
day. 3-month hx of progressive SOB
with exertion. Echocardiography
Myoxma
shows mass in the left atrium of the
heart. Lesion is resected,
photomicrograph of it is shown. Which
describes the
27yo primigravid woman at 34 weeks'
with nausea and vomiting, and
abdominal pain for 12 hours.
Everything's been normal. BP is now
Schistocytes
164/102, and right upper quadrant
tenderness. Labs show Hb 7.4, HCT
24%, Platelets 72k, Cr 1.2, total bili
2.3, AST 112, ALT
60yo F 3 hours after sudden onset
ankle pain. 4-year Hx of increasing
serum creatinine concentrations. Began
furosemide 1 month ago, also takes Nephrolithiasis
glipizide. P 120/min, resp 25/min, BP
150/100. Joint fluid shows negatively
birefringent crystals. Increased ri
56yo F follow-up 8 weeks after
recovering from pneumococcal
pneumonia. Chest X-rays normal. Metaplasia of mesenchymal cells to pneumocytes
Which allowed this resolution to
occur?
29yo F with 5-week hx of fatigue and
4-day hx of heart palpitations and
anxiety. Has primary hypothyroidism TSH DECREASED, Free thyroxine DECREASED, Free
Rx with triiodothyronine, but she has triiodothyronine INCREASED
doubled the dose because of fatigue.
TFT will show which?
20yo F has multiple neurofibromas.
Mom, uncle, and brothers with similar Autosomal Dominant
lesions. Mode of inheritance?
45yo M with yellow skin. Drinks eight
to ten 12-ounce cans of beers daily for
10 days. Liver is tender. Serum: total
Mallory Hyaline
bili 5.9, Alk Phos 210, AST 110, ALT
69, gamma-glutamyltransferase 25 (n =
0-30). Liver biopsy will show?
35yo M uses crack cocaine daily, with
2-hour Hx of substernal chest pain. T
Admit the patient to the hospital for possible myocardial ischemia
37C, P 110/min, BP 160/100. Most
appropriate next step?
54yo F with hypertension and bilateral
renal artery stenosis starts taking
NSAIDS for back pain. Her Cr Vasodilating prostaglandins at the afferent arteriole
concentration increases from 1.0 to 5.0.
Cause is due to inhibiting which?
83yo M brought to ED after being
Early septic shock
found at home bedridden and confused.
No meds. P 100/min, BP 85/50. BP
unchanged after 1L IV saline.
Pulmonary artery catheter shows:
Cardiac output high, PCWP low,
systemic vascular resistance low.
Cause of hypotension?
32yo M with 3-month hx of swelling
and breast tenderness. Receiving
thyroid hormone and steroid
replacements since removal of pituitary
Tissue: Testicle; Effect: estradiol production
adenoma 2 years ago. Began hCG
injections 4 months ago. Most likely
binding site of hCG causing
gynecomastia?
38yo F with 3-day hx of sore throat.
Photo shown of throat. Which nerves is
Vagus
tested by saying "ah," elevating area at
tip of the arrow?
8yo boy with disruptive behavior,
interrupts, always moving, trouble
Increased release of dopamine and norepinephrine
completing tasks. Drug with which
mechanism is appropriate?
62yo M with alcohol-induced liver
disease develops ascites. Infection
Spironolactone
ruled out. Most appropriate diuretic, in
addition to loops, is which?
17. 72yo M with weakness and fatigue.
Hemogrlobin concentration is 9.2,
GI blood loss
WBC 5400, platelets 350k. Peripheral
blood smear is shown. Cause?
65yo F with widely metastatic breast
cancer unresponsive to chemo. No
family. "close friend," at all her visits
"The two of you seem to have a very important relationship. Of
and now she is moved to inpatient
course you may stay together."
hospice after she decides she wants no
further curative therapy. Says, "We
can't bear to be apart. It would
46yo M treated with oral cyclosporine Suppressing the early response of T lymphocytes to activation
after cardiac transplant. Cyclosporine
decreases likelihood of rejected by
which actions?
30yo M in ED 15 minutes after found
unconscious. Comatose, pupils 4 mm
in diameter, not reactive to light. CT Middle meningeal artery
head shown. Cause of coma is bleeding
from which structures?
68yo M with 6-month hx of erectile
dysfunction. PE and labs normal. If
Inhibition of phophodiesterase
pharmacotherapy is indicated, drug
with which MOA?
35yo M with recurrent sinusitis and
bronchitis. Cardiac examination shows
PMI at fourth intercostal space within
the midclavicular line on the right. Dynein arms
Hepatic margin is palpable on the left.
Endoscopy shows nasal polyps. Biopsy
shows thickened, ciliated, p
30yo primi at 22 weeks' gestation with
1-day hx of fever, chills, and muscle
aches. T 39.4, P 114/min, resp 15/min,
BP 104/72. PE shows uterus consistent Listeria monocytogenes
with 22-week gestation. Fetahl heart
sounds are heard. WBC 12K, Blood
cultures grow gram-positive rod
42yo M with multiple lesions over his
body. PE shows flaccid bullous
erosions involving upper and lower
extremities and torso. Biopsy shows
Development of autoantibodies against desmosomal proteins
extensive epidermal acantholysis
resulting in the formation of
intraepidermal blister. Intact basal
layer of kerat
50yo M 3 days after his first
generalized tonic-clonic seizure. 1-
Calcium
month hx of frequent episodes of pins-
and-needles sensation around the
mouth, hands, and feet, involuntary
contraction of muscles. Neuro exam
shows mild, diffuse hyperreflexia.
Which serum
52yo F with hot flashes. Menses have
been irregular for the past 6 months. Failure of the ovaries to secrete 17beta-estradiol
Physiologic cause?
32yo F G2P1 at 7 weeks' gestation
with vaginal bleeding for 3 days and
increasingly severe left abdominal pain
for 18 hours. Direct and rebound Ectopic pregnancy
tenderness with guarding in left lower
quadrant. Cervical os is closed. serum
beta-hCG is 6000. U/S shows empty
26yo F 5 weeks after birth of first
child. Worries constantly that the
infant is ill and wakes up to make sure
he is well. Washes her hands 30 times Sertraline
per day. Worried about people braking
into her house, checks lock 3-4 times a
night. Not breast feeding. R
10yo boy has had anemia since birth.
Spleen is five times normal.
Hereditary spherocytosis
Splenectomy is indicated if anemia is
caused by which?
62yo F in ED for 2-day hx of fever,
abdominal tenderness, and painful
urination. Agitated. T 38.8C, Labs
show WBC 14k. Admitted to hospital, Delirium
nurses note she has torn up four
breakfast menus because she is
confused. Cause?
60yo F with 3-year Hx of
hyperlipidemia. Low-cholesterol diet
and exercise program ineffective after
Ezetimibe
1 year. Lovastatin initiated, but unable
to tolerate greater than 20 mg daily.
Additional drug is added that inhibits
transport of cholesterol through int
48yo M with bronzing of his skin,
weakness, and fatigue during the past 3
months. PE shows hepatomegaly, and
small testes. Serum: AST
Increased intestinal iron absorption
INCREASED, ALT INCREASED,
iron INCREASED, transferrrin sat
INCREASED, ferritin INCREASED,
testosterone DECREASED, LH DECR
45yo M with intermittent bloody
diarrhea and abd pain. Sigmoidoscopy
and rectal biopsy show IBD. Tumor necrosis factor
Monoclonal antibody is begun, which
is directed against what components?
"string of beads" sign Dx? Fibromuscular dysplasia
55yo M with sepsis. Appears anxious
and confused. Rx with vancomycin and
ceftriaxone initiated in ED. T 39.8, P
0.9% Saline
132/min, BP 85/48. PE shows warm,
flushed skin. No aedema. Administrer
which solutions?
36yo M undergoes elective liposuction
under general anesthesia. Operation is
terminated when patient develops
Decreases release of Ca from the sarcoplasmic reticulum
hyperthermia, tachycardia, and marked
muscle rigidity. MOA of drug that
should be administered?
In a survey of 100 households (average
three residents per household), 45 with 15%
asthma are detected. Prevalence?
While lifting weights, 24yo M swelling
in right inguinal region. Photograph Strangulation
shown of small intestine resected. Dx?
24yo with second-degre burn. Two
weeks after, tissue shows increased
Transorming growth factor-beta
fibroblast migration and proliferatoin,
increased collagen and fibronectin, and
decreased metalloproteinases. Caused
by production of which?
Protein found in brown adipose tissue
of mice causes leak of H ions inward
across inner mitochondrial membrane.
Increased ratio of oxygen consumption to ATP generation
Effect of this protein on oxidative
phosphorylation and energy
metabolism?
57yo M with alcoholism has distended
abdomen with shifting dullness, fluid
wave, caput medusae, palmar Gynecomastia
erythema, spider angiomata. Additional
finding?
16yo girl with 2-year hx of fainting;
increased in frequency during past 6
months. BP 110/80 supine and 60/40
standing. Neuro exam normal. Plasma
Dopamine beta-hydroxylase
shows undetectable noreipinephrine
and marked increase in dopamine
concentration when standing.
Deficiency of
HALLMARK: Peanut Farmer from
Aflatoxin
China
42yo M in ED for 5-hour hx of fever,
chills, and severe pain and swelling of
his left arm. Scratched his arm on a
nail yesterday. Appears confused, T
IL-1 and tumor necrosis factor (TNF)-alpha
40C, BP 71/38. Labs show Hb 14, HCt
42%, WBC 15K (35% PMNs, 40%
bands, 25% lymphos), Platelets 50K,
Serum
62yo M dies suddenly while playing
tennis. No cardiac risk factors, no hx of
CAD. Autopsy, cardiac valve defect Aortic stenosis
and concentric LVH. Which valve
abrnomalities is most likely involved?
67yo M has urinary urgency after
Inhibition of muscarinic receptors
placement of urinary bladder catheter
during transurethral resection of the
prostate. Most appropriate Rx has
which MOA?
Mouse embryos are produced with two
pronuclei, both of same parental origin.
When the pronuclei are maternal,
produces have poorly developed
Imprinting
extraembryonic structures. When both
pronuclei are paternal, produces have
poorly developed embryonic tissue.
Whic
18-yo F with sepsis after an abortion.
Within 24 hours she becomes
dyspneic, oliguric, and develops
Decreased plasma fibrinogen concentration
petechiae, ecchymoses, and bleeding
from venipuncture sites. Which lab
finding?
63yo M with 6-month hx of exertional
chest pain relieved by rest. smoked for
Calcified 80% stenosis
45 years. Mild HTN, no meds. Which
lesion in LAD is most likely cause?
65yo F with ovarian cancer treated
with cyclophosphamide and other
chemotherapeutic agents. DNA replication
Cyclophosphamide affects which
target?
Pharm co trying to develop a long-
acting weight-loss agent that mimics
activity of a naturally occurring peptide
originates in adipose tissue, signals Leptin
brain about stored fat, and suppresses
appetite by its action in the CNS.
Which chemical mediator?
45yo F has thyroidectomy because of
asymmetric enlargement of thyroid
noticed 6 weeks ago. Underwent Calcitonin
adrenalectomy for pheochromocytoma
3 years ago. Bilateral thyroid lesions
with spindle cells arranged in small
clusters. Amyloid deposits b/w
neoplastic
45yo F intubated, mechanically
ventilated with fungemia with Candida
albicans. Rx with caspofungin is Beta-Glucan carbohydrates in the cell wall
started. Feature of causal organism
targeted by this drug?
56yo M 4 hours after sudden onset of
uncontrollable irregular movements of
the left side of the body. PE shows
Right subthalamic
flailing movements of the proximal
appendicular muscles on the left.
Nuclei damaged?
60yo M 1-month Hx of progressive
SOB with exertion. Breath sounds: Dec
on right lung base, normal on left lung
base Percussion note dull on right lung Pleural effusion
base, nml on left lung base Tactile
fremitus decreased on right lung base,
nml on left lung base Adv
68yo M with difficulty swallowing
solids for 2 months. Hx of dilated
cardiomyopathy. X-rays of esophagus
w/ barium contrast show indentation Left atrium
and posterior displacement of the
esophagus. Enlargement of what
caused dysphagia?
25yo F with 3-year hx of irregular
menses. Menarche was at age of 14
years. BP 116/62. PE shows increased
hair growth on the face and chest.
21-hydroxylase
Pelvic exam shows clitoromegaly and a
normal-appearing uterus. Serum shows
increased 17-hydroxyprogesterone and
an
60yo M in ED for sudden onset of
Mesenteric venous thrombosis
acute abdominal pain and tenderness,
nausea, vomiting, and bloody diarrhea
2 hours ago. He has a Hx of cirrhosis
and hepatocellular carcinoma. BP
99/50. Loss of bowel sounds. Surgery
shows small intestine with dark purple-
38yo M in ED 30 min after unable to
stand upright. Lethargic, pulse
110/min, BP 90/62. PE shows dry
mucosa and poor skin turgor.
Metabolic acidosis
Midepigastric tenderness. Labs show:
Serum: Na 143, K 3.2, Cl 101, HCO3
11 ABG: pH 7.28, Pco2 23, Po2 98
Acid-base status
2mo boy given vaccine to convert T-
independent antigens to T-dependent
Haemophilus influenzae type b
forms to enhance protection in young
children. Which vaccine given?
A study conducted to assess
effectiveness of injections of lidocaine
into "trigger points" of pain symptoms
in patients with fibromyalgia. Fifty Placebo effect
patients randomly assigned - 0.9%
saline only or saline plus lidocaine.
Graph shows self-reported pain scores.
In a clinical study, a polymorphic
marker with three alleles, 1, 2, and 3, is
found to be tightly linked to the gene
for polycystic kidney disease. Pedigree 2,3
shown. If III, 1 is unaffected by this
disease, patient is most likely carrier
of?
6yo girl with 4-day hx of round shiny
bumps in areas where she has eczema.
Her mother saw similar bumps on a
playmate at pool party 3 weeks ago. Poxvirus
No other Sx. PE shows firm, smooth,
umbilicated papules 2 to 4 mm
diameter in clusters. Causal organism?
80yo F in ED for 2-day hx of "feeling
funny." "Lost my pep." Hx of poorly
controlled hypertension. Just started
Hydrochlorothiazide
medication 2 weeks ago. BP 130/85.
Pe normal. Serum potassium is 3.
Which drug?
68yo M in for a hemiorrhaphy.
Surgeon gives info of risks and
benefits. Patient says that he
understands what he has been told, and
Information, competence, voluntariness
his family will be able to discuss later.
In this patient, which combination of
components fulfill the criteria for fully
i
47yo F with psoriasis for follow-up.
Was given several topical creams, to be
used in specified sequence twice daily. "using something twice daily can be difficult. I assume you are
No improvements apparent at this like most patients who miss at least 10% of treatments."
appointment. How to begin discussion
of compliance?
Male newborn has macrocephaly with
poor skull mineralization, shortened
Collagen
extremities with misshaped long bones,
and several fx. Defect in which?
27yo F in ED 30 min after ejected
through windshield during MVC.
Unrestrained front-seat passenger. PE
Levator labii superioris
shws marked edema and tenderness of
the jaw. Panorex x-ray of mouth
shown. Which is injured?
Girl for well-child exam. Normal
development includes pincer grasp,
finger feeding, standing while holding 9
onto a table, and playing peekaboo.
Age (in months)?
18yo M with yellow nodules on
achilles tendons of his feet and Absence of functional LDL receptors in hepatocytes
extensor tendons of his hands. Exam
shows collection of foamy histiocytes
within the dermis. Serum cholesterol is
980, and lipoprotein electrophoresis
shows a selective increase in LDL.
Underl
35yo M with 4-day Hx of high-grade
fever, sever muscle aches, malaise, loss
of appetite, and a nonproductive cough.
wife and kids had similar illness. Temp Influenza virus
39.2, PE normal. CBC and CXR
normal. Causal virus replicates its
genome within the cell's nucleus.
28yo F at 18 weeks' gestation has
palpitations. Labs show increased
serum total thyroxine (T4) Thyroid antibodies
concentration. Best test to confirm
hyperthyroidism?
21yo M in ED 45 minutes after
sustaining multiple injuries in a MVC.
His BP is 90/50, PE shows diffuse
abdominal tenderness. Dx with Streptococcus pneumoniae
laceration of the spleen and undergoes
splenectomy. Predisposed to infection
with?
70yo F in longitudinal study of effects
of aging on pulmonary function tests.
Residual volume UP, Arterial Po2 DOWN, Alveolar-arterial Po2
Which represents woman now
difference UP
compared with results at age of 20
years?
45yo F farmer in ED for 2-day hx of
confusion, lethargy, fever, headache,
muscle pain, vomiting, and a rash on
her wrists and ankles. Bitten by a tick a Doxycycline
few days ago. T 38.5 C, Red-purple
papules on distal extremities progress
to trunk. Rx?
Study of breast cancer in women.
Cohort
Hundred healthy women observed for
10 years. Goal is to determine if
number of family members who
previously received dx of breast cancer
correlates with incidence of future
development of cancer. Best design?
62yo F with recurrent pulmonary
emboli comes for follow-up. PE
normal. Labs show PT of 12 seconds.
VII (proconvertin)
Warfarin begun. Which clotting factors
is first to be decreased by 50% after
initiation of Rx?
48yo F with 2-month hx of fatigue and
intermittent headaches. BP 180/110,
PE normal. Serum show a decreased
potassium concentration and increased Decreased plasma renin activity
aldosterone. CT abdomen shows tumor
on adrenal gland. Which additional
findings supports aldosterone-secretin
20yo F with 1-day hx of increasing
urinary frequency and a burning
sensation with urination. One sexual
partner, uses condoms. VSS. PE shows
Mannose-binding (type 1) fibria
mild suprapubic tenderness to deep
palpation. Urine shows rare epithelial
cells and 10 WBC/hpf. Urine grows E.
col
6yo boy with 3 systemic infections
with Neisseria meningitidis over the
Total hemolytic complement concentration
past 2 years. Healthy otherwise. Which
lab test is most likely abnormal?
13yo girl 2/6, holosystolic murmur
heard best over left fifth intercostal
space adjacent to the sternum; it Tricuspid
increases with inspiration. Abnormality
of which valves?
55yo F with left flank pain and gross
hematuria. Mass is palpable in LUQ of Renal cell adenocarcinoma
abdomen. Ultrasonography shows a
12-cm solid mass on lower pole of left
kidney. Angiograms show
hypervascular mass. Dx?
19yo M in MVC. Penetrating wound to
right cerebral cortex with paralysis of
the left lower extremity, fracture of
right mid humerus with severing of the Left Achilles tendon
radial nerve, and a fracture of right
tibia. After 10 weeks, DTR strongest in
which locations?
40yo F with 1-year hx of episodes of
crampy abdominal pain, intermittent
diarrhea, and rectal bleeding with
passage of mucus. BMI 18. Abdomal Sulfasalazine
exam: diffuse tenderness with no
rebound. Sigmoidoscopy shows diffuse
ulcers. Initial Rx?
27yo M for employment exam. No Hx
of major illness. Never been sexually
active. Minimal contact with parents
and siblings, no hobbies. Doesn't feel Schizoid
depressed. Shrugs in response to
congratulations about his new job. Flat
affect. Personality disorder?
30yo F has ptosis, ophthalmoplegia,
and diplopia. Serum shows
autoantibody with affinity for
Thymoma
acetylcholine receptor at the
postsynaptic neuromuscular junction.
Which neoplasm?
Exam scaled so scores are normally
distributed with mean of 500 and SD of
67%
100. Which % are between 400 and
600?
62yo M with pericardial friction rub 3
days after acute myocardial infarction. Fibrinous pericarditis
Cause of rub?
42yo F with 1-mo Hx of abdominal Cholesterol synthesis
pain, after eating fatty meals. BMI 31.
PE shows jaundice and tenderness of
RUQ. Increase of which liver function?
70yo M with recent loss of mental
function. Hx of weight loss. No drugs.
VSS, not dehydrates. Mild anemia.
Pulmonary neoplasm
Labs show Na 110, Cl 85, K 4.4, BUN
15, Cr 15; Plasma osmolality 250;
Urine osmolality 750. Dx?
40yo M skin extremely sensitive to
sunlight, which causes formation of
vessicles and blisters on the skin which
take weeks to heal. Diagnosed with Heme Synthesis
disorder caused by increased synthesis
of compounds in the skin that are
subject to excitation by visible li
60yo M with 6-month hx of fatigue.
Four years ago, had subtotal
gastrectomy after gunshot wound.
Drinks 6-8 beers daily. PE shows
Pariteal Cells
paresthesias of both hands. Labs show:
Hb 8, HCT 24%, MCV 115, WBC 5k,
Platelets 165k, RBC Folic acid 500 (N
= 125-600), B12
40yo M with interstitial pulmonary
fibrosis has greater maximal expiratory
Increased radial traction on airways
flow rate than predicted. Which best
explains this?
20-year-old F secretary with 8yr
history of intermittent headaches.
Flashing lights in her right visual field,
followed 20 minutes later by a Migraine
unilateral throbbing headache
accompanied by nausea and vomiting.
occur around time of menses. Dx?
70 yr old african american women
come to physician after 1 day onset of Gender
back pain. She's a part time cashier,
low income, and smoked 1/2 a pack for
50 years and drinks 3 caffeinated
beverages a day. X-ray shows vertebral
compression fracture of L3 and she
5-year-old-boy with mental
retardedation is grossly obese and has
facial features of Prader-Willi
syndrome. Karytoyping and Maternal Origin of Chromosome 15
flourescent in situ hybridization studies
do not show deletion in the usual site.
Which to confirm PWS?
83yo M from Puerto Rico with weight
loss and abdominal pain and blood in
his stool for 1 month. Possible colon
cancer. Poor english. Needs Use a Spanish-speaking interpreter to determine how much the
colonoscopy but family doesn't want to patient wishes to know about diagnosis and treatment
hear bad news and wants to make
decision for about his treatment. Next
step?
38-year-old F undergoes laparoscopic
cholecystecomy with general
anesthesia. Afetr she awakens
Ondansetron
postoperatively, she is nauseated and
vomits threee times in 20 minutes.
Treatment for N/V?
During an experimental study of
oxygen consumption in the kidney,
experiemtnal animals are ventillated
Distal convoluted tubule?
with 100% nitrogen. Cells from which
of the following areas of the kidney
first show signs of anoxic injury?
7-year-old girl 30 minutes after being
hit in the mouth with basketball.
Something stuck in her throat. Part of
one tooth is missing. Lateral x-ray of Right Lower
the neck and chest is shown; arrow
shows part of the tooth. Greatest risk
for aspiration into which lobe
Poliovirus mRNA lacks a 5' m7G cap
but is translated efficiently by cellular
ribosomes. Which of the following
additional structural features of Presence of an internal ribosome entry site
poliovirus mRNA is the most likely
cause of its ability to be translated in
the absence of a cap?
67-year-old F brother and mother have
history of colon cancer. Physician
recommends colonscopy, but patent
only wants her stool to be tested for Low specificity
blood. Most likely concerned about
which of the following regarding this
test?
69-year-old African American woman
has moderate hypertension and type 2
DM. BMI 31. On hydrohlorothiazide. Irbesartan
Labs show microalbuminuria. Most
appropriate to add which drugs?
36-year-old man with 2-hour history of
pain and swelling of his right calf. No
shortness of breath or chest pain. 18-
hour airplane trip 4 hours ago. Activation of antithrombin III
Noninvasive vascular studies show an
occlusion of right femoral vein.
Immediate therapy has which mechanis
64 yr old alcoholic man with 1 day of
confusion. Disoriented, disheveled.
Dehydrated, jaundiced. and has spider
angiomata over face and chest. Has
Killing of bacteria in the gut that generate ammonia
flapping up and down of the hands
when his arms are outstretched.
Abdominal distention and bulging
flanks. H
1 year old boy is brought in. Has white,
pale hair that hasn't changed color
since birth. His eyes are blue. During Inability to make melanin
opthalmic examination, the patient
turns away from the flashlight and
starts crying. Which of the following is
the most likely cause of the
2-year-old boy with developmental
delay. Hx of hearing loss in mother and
delayed speech in older sister.
Maternal uncle had stroke-like
Heteroplasmy
episodes at the age of 25 years.
Physical shows ophthalmoplegia and
hypotonia. Lactic acid concentration
increased. E
Male newborn at 28 weeks' is
tachypneic and hypoxemic. Which
Decreased numbers of lamellar bodies
altered structure changes in the type II
pneumocytes is the most likely cause?
16-year-old girl with cystic fibrosis
with 3-week history of generalized
weakness, numbness and tingling of
her arms and legs, and difficulty Vitamin E
walking. Not adhered to medications.
Bilateral weakness and decreased deep
tendon reflexes in the upper and lowe
48-year-old woman with 6-month hx of
irregular menstrual periods and hot
flashes. LMP 35 days ago, and had
scant blood flow. Menses had
Decreased follicle-stimulating hormone
previously occurred at regular 28-day
cycles. Mild thinning of the vaginal
tissue. Labs most likely to show which
of th
62-year-old M with unstable angina
pectoris undergoes coronary
catheterization. To visualize the
anterior interventricular (left anterior Left coronary
descending) artery, the tip of the
catheter would need to be placed into
the orifice of which arteries?
35-year-old F with intermittent sharp
Pericarditis
chest pain exacerbated by deep
breathing, and can be decreased by
leaning forward. 2-month hx of pain
and swelling in her hands and knees
accompanied by morning stiffness that
lasts 1 hour. Pulsus paradoxus less than
60-year-old M with a systolic murmur
is a heard, which is loudest at the point
indicated by the X in the diagram. Aortic valve stenosis
Which cardiac abnormality is the
cause?
31-year-oldwoman with invasive
squamous cell carcinoma of the cervix.
Biopsy shows tumor cells express
human papillomavirus, type 16 T lymphocytes
antigens. Which cell types plays a role
in recognizing and killing these virus-
infected tumor cells?
An 85-year-old woman is diagnosed
with a fracture of the right femur and
begins treatment with morphine by
patient-controlled analgesic pump.
Morphine is metabolized to active metabolites that accumulate
Three days later, her respirations are
6/min. Physical examination shows
pinpoint pupils. Her serum creatinine
co
35 y/o man with 3 yr history of
enlarging nose, coarse facies, muscle
weakness, increased hand/foot size.
Large fleshy nose and prognathism on
Adenlyl cyclase
exam. High IGF-1 in serum. MRI
shows pituitary adenoma. Morphologic
analysis of the tumor shows a densely
granul
30-year-old woman with Li-Fraumeni
syndrome found to have
adenocarcinoma of the breast. Family Impaired regulation of apoptosis
history includes osteosarcoma. Which
mechanisms underlies this condition?
56-year-old with palpable hard nodule
on prstate has increased serum
prostate-specific antigen concentration.
Fine-needle biopsy specimen shows Pelvic parasympathetic nerves
adenocarcinoma. Patient undergoes
radical prostatectomy. Which structure
is at greatest risk for injury during
56-year-old woman frequently burned
herself while cooking. Loss of pain and
temperature sensation in both upper
extremities and portion of her trunk Syrinx of the central region of the spinal cord from C-4 to T-5
from clavicles to just below the
nipples. Touch, vibratory sensation,
and proprioception normal. Findings
25-year-old man comes to the
physician 8 hours after the onset of
severe pain of his low back that
radiates down his left leg. He started a Rupture of an intervertebral disc
weight-lifting regimen earlier in the
day during which he tried to lift a bar
loaded with 91 kg (200 lb) from the g
43-year-old man with 6-week hx of
sharp, stabbing pain on the left side of
his face that occurs when he touches it.
Pain when shaving. Pain just lateral to Rotundum
the left nasal ala reproduces the pain.
Nerve supplying this area exits the
skull through which for
33-year-old man with 3-month hx of
muscle weakness and cramping,
appears shortly after he begins
exercising. Serum creatine kinase
Glycogen phosphorylase
increased. Venous blood from
antecubital vein show lactate
concentrations do not increased
compared with preexercise values.
67-year-old man with 2-month hx of
weight loss. Has type 1 DM, gallstone Cigarette smoking
removal 12 years ago, smoked 1 pack
daily for 45 years. BMI 34, Calcium
concentration of 11 mg/dL. Abdominal
CT shows a pancreatic mass, biopsy
shows pancreatic adenocarcinoma.
Stron
28-year-old woman with 1-week
history of fever and chills. 4.5-kg
weight loss, 5-year hx of chronic
sinusitis. Mildly distressed. BMI 18.
Wegener granulomatosis
Temp is 39 C (102.2 F). Has markedly
diminished nasal septa. Chest x-ray
shows multiple pulmonary nodules.
Serologic
52-year-old man with 3-week history
of increased thirst and urinary
frequency; 4.5-kg weight loss. Has
hypertension and hyperlipidemia Fasting serum glucose concentration
treated with pravastatin and
metoprolol. BMI 34. Which is most
likely to be increased?
48-year-old man with possible
hypertension. On basis of ten
measurements, the patient's average
diastolic blood pressure is 113, and
Increase in width
standard deviation is 8. If four rather
than ten measurements are made,
which is the expected impact on 95%
confidence int
66-year-old M with 6-month history of
decreased exercise tolerance and
shortness of breath with exertion.
Vitals stable. Auscultation shows
Dilated cardiomyopathy
bilateral basilar crackles. Cardiac
examination shows S3 gallop. Grade
2/6 holosystolic murmur heard best at
the ap
29-year-old woman for advice on
losing weight. Has been taking Follicular atrophy
thyroxine for several months in attempt
to lose weight. Her thyroid function is
normal. Which findings is most likely
on histopathologic eam of the thyroid
gland?
44-year-old man with 2-month history
of abdominal pain and diarrhea. Pain
relieved after eating and antacids.
EPigastric tenderness. Serum gastrin Surgical removal of the suspected tumor
concentration of 500 pg/mL (N=50 -
100) and gastric acid secretion of 80
(N=6-40). Most definitive treatment
50-year-old man with progressive
bulge in his abdomen during past 6
months. No changes in bowel habits.
Midline hernia above umbilicus that Collagen
cannot be reduced be gently pushing
on it. Operative repair initiated. Which
extracellular matrix components requir
50-year-old man with progressive
bulge in his abdomen during past 6
months. No changes in bowel habits.
Midline hernia above umbilicus that Lysosomes
cannot be reduced be gently pushing
on it. Operative repair initiated. Which
extracellular matrix components requir
53-year-old man in ED 1 hour after
right-sided weakness. Right perioral
droop. Babinski sign present on the
right. CT scan of the head shows no
Microglial cells
abnormalities. One week later, a repeat
CT scan shows a small area of
hypodensity involving the left internal
c
38-year-old woman in for pre-
employment exam. No illness. No
meds. Labs show Hb 8.2, HCT 25%, Iron deficiency anemia
MCV 69. WBC 5.9k, Retics 0.8%,
platelets 350k. Dx?
27-year-old man with acute myelocytic
leukemia receives high-dose
cyclophosphamide in preparation for
hematopoietic stem cell Mesna
transplantation. Which will decrease
toxicity from this chemotherapy
regimen?
30-year-old man develops hemoptysis,
dyspnea, weakness, and hematuria.
Diffuse pulmonary hemorrhages
bilaterally. Renal biopsy shows focal
Autoantibodies against host cell basement membranes
glomerular necrosis with crescent
formation and linear deposition of IgG
and C3 in glomerular capillary loops.
Patho
21-year-old woman with 2-week hx of
blood-tinged vomiting. 2-year hx of
self-induced vomiting after gorging on Fluoxetine
food. BMI 24. Which drugs is most
appropriate?
56-year-old man scheduled for
physical therapy 3 days following right
shoulder operation. Therapy to
Lateral (external) rotation
strengthen the infraspinatus and teres
minor muscles. Which should this
patient perform against resistance?
cASPofungin cell wall inhibitor used in invasive aspergillosis
SAFE Moms Take Really Good Care Sulfonamides,
Rx to avoid in Pregnancy Aminoglycosides, Fluoroquinolones, Erythromycin, Metro, Tetra,
Ribavirin, Griseofulvin, Chloramphenicol
22q11 synd (DiGeorges) Truncus Arteriosus, TOF
Heart defects seen in Turner's
Preductal Coarctation
Syndrome
Heart defect seen in Congential
Septal defects, PDA, Pulmonary Artery Stenosis
Rubella
Problems seen in diabetic mother's Transposition of great vessels hypoglycemia after birth clavical
children fractures and erb's palsy
Sx Hypochloremic metabolic alkalosis
w hypoKalemia, nonbilious projectile Congenital Plyloric Stenosis
vomiting
Thoracodorsal + pathology unable to wipe bottom
Suprascapular + pathology trouble initiating arm abduction
Nerve runs with lateral thoracic artery long thoracic nerve
Nerve runs with deep brachial artery radial nerve
Medial Nerve + pathology loss of forearm protonation
Hemochromatosis Gene HLA-A3
PAIR Ankylosing spondylitis gene HLA-B27
Graves' Dz Gene HLA-B8
Axillary lymph node location/drain upper limb lateral breast drains
What drains to Celiac lymph nodes stomach drains into
Sigmoid colon drains to colic --> inferior mesentary
What drains to the internal iliac rectum above pectinate
What drains to the superficial inguinal anal canal scrotu thigh
What drains to superficial/deep plexus
testes
-> paraaortic lymph node
What drains to thoracic duct drains to L subclavian & internal jugular
What drains to right lymphatics drains to brachiocephalic vein
IL-12, IFN b, IFN a --> enhances NK cells
T cells receptors TCR CD28 CD3
CD 21= EBV receptor on B cellls
Helper T cell receptors CD4 CD40L
B cell receptors CD19,20,21 CD40 MHCII B7
MHC II B7 CD40 CD14***(endotoxin receptor) receptor for Fc &
Macrophage receptors
C3b
NK cell receptors MHC I CD16 (binds IgG Fc) CD 56***
Protection from Complement CD55 CD59
IL1--> stimulates endothelium adhesion molecues
IL6--> fever acute phase proteins
IL8--> major neutrophil chemotaxis
Neutrophil chemotaxis--> IL8 Leukotriene B4 C5a
IL12--> T cells into Th1 cells activate NK cell secreted by B cells
Secreted by Macrophage--> IL 1,6,8,12 TNF a
TNF-alpha causes septic shock vascular leak, activate endothelium acute phase pro
Secreted by Th1 cells IL2 IFN y
Secreted by Th2 cells IL4 IL5 IL10
IL5--> stimulates eosinophils
Interferons a & b causes induce ribonucleases block viral pro synthesis
IFN y causes increase in MHC I and II
IL3 causes all T cells to secrete
Bacteria with Ag variation Salmonella (2 flagellar) Borrelia N gonorrhea (pilus)
IFN a & b does what? released by virally infected cells
IFN y does what? inhibits production of Th2 cells
Terminal deoxynucleotidyl transferase-
adds DNA during recombination of Ab diversity (B cells)
->
C3b & IgG primary opsinins
Preformed Ab= passive immunity
To Be Healed Rapid Tetanus, Botulinum, HBV, Rabies
conditions
fungal (histo, blasto) syphilis (gummas) leprosy cat scratch fever
(Bartonella henseliae) sarcoid crohn's berylliosis listeria foreign
Name all granulomatous conditions
bodies wegeners (necrotizing granulomas) Chronic Granulomatous
Dz
MOA of hypersensitivity type 1 IgE and histamine 15 min
Ab mediated hypersensitivity IgM, IgG direct and indirect
MOA of hypersensitivity type 2
Coombs test
MOA of hypersensitivity type 3 Arthus reaction Ag-complement-IgG complex 5-12 hr
MOA of hypersensitivity type 4 T cell mediated 24-48 hr
B cell Conditions Bruton's CVID Hyper IgM Ig deficencies
DiGeorges Job's synd (FATED) IL-12 r def chronic
T cell Conditions
mucocutaneous candidiasis
B & T cell Conditions SCIDS Ataxia-Telangiectasia Wiskott-Aldrich
Chediak & Job Chronicallly Lack phagocytes (CGD and
Phagocyte Conditions
Leukocyte adhesion def)
DiGeorges Child HALLMARK young child w tetany from hypoCa++ and candidiasis
CGD Child HALLMARK young child with recurrent lung infxn and granulomatous lesions
2yo child multi viral and fungal infxn, hypoPTH, what germ layer
Ectoderm missing in DiGeorges
gives rise to missing organ?
Jobs Syndrome Symptoms repeated Staph abcesses, neutrophils don't respond to stimuli
DiGeorge Symptoms heart defects and repeat viral infxn, low T cells
MOA in Bruton's no tyrosine kinase gene, low Ig of all classes
MOA in Hyper IgM defective CD40L severe pyogenic infxn high IgM, very low IgG
IgA Deficency Symptoms milk allergy repeated sinus infxn
CVID Symptoms defect in B cell maturation lymphomas low plasma cells
IL-12 receptor Deficency Symptoms disseminated mycobacterial infxn low IFN y
IL-2 r, ADA def, MHC II def= 3 types of SCID
SCID labs high adenine low IL-2r
Ataxia Telangiectasia Symptoms DNA repair enzyme defect IgA def
X-recessive Thrombocytopenic purpura Infxn Eczema Recurrent
Wiskott-Aldrich Symptoms
pyogenic infxn
Leukocyte Adhesion Deicency
defect in LFA-1 integrin delayed umbilicus separation
Symptoms
Chediak Higashi Symptoms partial albinism pyogenic infxn neuropathy
Deficency of MAC C5-9 Symptoms recurrent Neisseria infxn
BATS Drink Blood Beta= eye open Alpha= awake, eyes closed
EEG waveforms Theta= light sleep Sigma= sleep spindles Delta= low freq, high
amplitude REM= Beta= highest freq, low amplitude
SEM * 2 +_ mean how do you determine 95% confidence interval?
3rd gen cephalosporins uses serious gram - (Ceftriaxone= meningitis & gonorrhea)
4th gen cephalosporin uses pseudomonas & gram +
Aztrenonam USE binds PBP3 no penicillin allergy
How are aztreonam & aminoglycosides
serious gram - infxn (aztreonam= aminoglycoside pretender)
similar?
Imipenem & meropenem enterococci gram +/- anareobes (very broad)
Next step in TX of otitis if resistant to
Augmentin
amoxicillin
What increases nephrotoxicity of
Cephalosporins
aminoglycosides?
Symptoms of Haemophilus influenza MOPE Meningitis, Otitis media, Pneumonia, Epiglossitis
PSEUDO Pneumonia, Sepsis, External otitis, Uti, Drug use,
Symptoms of Pseudomonas
diabetic Osteomyelitis
gut KEES PSS Klebsiella, E coli, Serratia, Proteus, Salmonella,
Enterobacteriacae
Shigella
Most common cause of gram - sepsis Ecoli + Klebsiella
Symptoms of Protease carries urease cause staghorn calculi in renal
4 A's Aspiration pneumonia, Abscess in lungs, Alcholics,
Symptoms of Klebsiella
diAbetics nosocomial UTI's
Urease + Bugs H. pylori Proteus
Cat scratch organism Bartonella Henselae transmission
Dog/cat bite organism Pasturella Multocida transmission
Cat feces organism Toxoplasmosis transmission
Puppy feces organism Yersinia enterocolitica transmission
Animal urine organism Leptospira transmission
Rat bites organism Spirillum Minus transmission
Spirochetes organism BLT Borrelia, Leptospira, Treponema
Reiter's Syndrome bug shigella flexneri C trachomonas D-K
Motile organism cause UTI Proteus
PUS Pneumonia (atypical- mycoplasma, chlamydia, legionella),
Uses for macrolides
UTI, STDs
Most common UTI bugs PEcK+ S. Saphrophyticus #1= Ecoli
Drugs for anaerobic infections Metro, clindamycin, imipenem
Conditions associated with Use of GET GAP on the metro Giardia, Entamoeba, Trichomonas,
Metronidazole Gardnerella, Anaerobes, h Pylori
MOA of Metronidazole from free rads in bacteria- damage DNA
H pylori Triple Therapy PPI, clathromycin, amoxicillin/metro
TCP Cefapime Aztreonam Fluoroquinolones Aminoglycosides
Effective against Pseudomonas
Polymixins
What type of hallucinations causes ViAO = De Sc Ep tion Visual= Delirium Auditory= Schizo
what type of conditions Olfactory= Epilepsy
Side Effects of TCA Tri=C's Convulsions, Coma, Cardiotox
Side Effects of High Potency haloperidol, trifluoperazine, fluphenazine NMS & tardive
Neuroleptics dyskinesia
Side Effects of Low Potency thioridazine, chlorpromazine anticholinergic, antihistamine, a
Neuroleptics block
Patient on diphenhydramine &
trazadone or high potency antipsychotics (b/c less anti-Ach SE)
dementia wants sleep meds, which Rx
MOA of benzos and barbituates increase in GABA (cl- channel vs duration)
SNRIs venlafaxine, duloxetine, nefanizone
the MAOI PITS Phenelzine, Isocarboxazid, Tranylcypromine,
MAOIs
Selegiline
NDRI Buproprion
Mirtazapine= use for Depression w insomnia Trazadone= use for
Tetracyclics
insomnia
Prevent relapse in alcoholics 1 AA 2 disulfram 3 naltrexone 4 topiramate 5 acamprosate
Unique properties of this Rickettsial Coxiella Burnetti neg Weil Felix from tick feces and cattle
organism placenta - spores aerosilized sx w no rash
Big Bad Bed Bugs From Your Pet (Ella) Bartonella spp, Borrelia
Zoonotic Bacteria burgdorferi, Borrelia recurrentis, Brucella spp, Francisella
tularensis, Yersinia pestis/enterolytica, Pasturella multocida
50% subclinical type D-K: urethritis, PID, ectopic preg, neonatal
Symptoms of Chlamydia Trachomatis pneumonia type L1,2,3: lymphogranuloma venereum,
lympadenitis, ulcers
Symptoms of C. Pneumoniae &
atypical pneumonia (aerosol)
Psittaci
Antibiotics contraindicated in hepatic MCMCRT Metro, Chloramphenicol, Macrolides, Clindamycin,
insufficiency Rifampine, Tetracycline
Ghon focus Calcified Scar
Ghon complex Ghon focus + hilar nodes
Symptoms of Mycobacterium kensaii cause pulm TB-like sx in COPD pt
Symptoms of Mycobacterium kensaii
cause cervical lymphadenitis in children
in child
Impenem SE of Ethambutol red-green color blindness
Rifampin uses TB & leprosy meningococcal prophylaxis Hib
Macrolide drugs azithromycin class
Fetal Erythropoiesis Young Liver Syn Blood Yolk Sac-> Liver->Spleen->BM
Truncus Arteriosus ascend. Aorta/Pulm trunk embryol structure
Bulbis Cordis R ventr and smooth parts of L&R ventri embrol structure
Trabeculated parts of L&R ventricle L ventricle embryo structure
acute pancreatitis dissecting aortic anuersym diaphragm pain
Pain radiating to back
cholecystitis
urine discharge from umbilicus vesicourachal diverticulum
Patent urachus
(=lesser)
meconium discharge from umbilicus meckel's diverticulum
Vitelline duct fails to close
(=partial closure)
Primitive atria becomes trabeculated L&R atria embrological structure
A gardener presents with SOB,
salivation, miosis, and diarrhea. What Organophosphate poison, anticholinesterase
is the cause/ MOA?
Atropine is not effective in reversal of
organophosphate poisoning. Why? No effect on cholinesterase, use Pralidoxime
What helps?
What muscarnic agonist / antagonist is
Ipratropium (an antagonist)
used in asthma / COPD?
30 YO has urinary rentention due to
Cholinergic Agonist (problem is anti cholinergic s/e's
neuroleptic, what do you treat with?
In Dark both pupils dialate. In light one Anticholinergic (atropine)
pupil is miotic while another, given
drug X, is mydratic. What is X?
What drug is most apropriate in a pt
with shock in order to maintain renal Dopamine (although clinically doesn't really work)
blood flow
60 YO male. Has a hard time driving at
night due to worsening vision and
Cataracts
halos appearing around headlights.
What is causing this?
A gymnast sustains an anterior
shoulder dislocation. What nerve is Axillary n.
injured?
A kid falls while skateboarding and
injures his elbow. He can't feel the
Ulnar N. Ulnar Claw (can't extend 4/5 digits)
medial part of his palm. What nerve is
injured? What "sign"?
A highschool athlete falls on his arm.
Radiograph shows midshaft break of
Radial n. Deep Brachial Art.
humerous. Which nerve / artery are at
risk?
What patients are suseptible to
Immunocompromised, Neonates, Pregnant Women
Listeria?
What organisms are implicatd in S. Veridians, Staph Epi, Enterococci. Staph Aure = Acute not
subacute endocarditis? subacute
A woman is breast feeding develops
swelling and redness over her right
Acute Mastitis -> Staph Aures
breast. Exam reveals a warm, fluctuant
mass. What is this?
Most common aerobic skin flora? Staph Epidermis
6 month old child is given HONEY for
a cough and cold and becomes flaccid. C. Botulinum (Gm + Rod) inhibits Ach release
What causes this? MOA?
One hour after eating a potato salad at
a picnic. Whole family vomits. 10hrs Staph Aureus. Preformed toxin ingested (no infection)
later they are better. Whats the cause?
Which complement is responsible for C5a (also leukotriene B4, IL8)
neutrophil chemotaxis?
Child presents with TETANY from
hypocalcemia and CANDIDASIS due
T cells, No Thymus = DiGeorges (22q11, "CATCH-22")
to immune suppression. What is
deficient? What is the condition?
A young child has recurrent LUNG
infection and granulomatous lesions. Lack of NADPH oxidase = no respiratory Burst
What defect in neutrophils causes this?
Mother brings 2 y/o child w/ Hx of
multiple viral, fungal infections and the
child is HYPOCALCEMIC. Which No Thymus, DiGeorge Syndrome, Endoderm (from tongue)
Germ layer gives rise to the missing
structure? (Endo, Ecto, Meso)
A child with immune diorder w/
repeated Staph Abcesses. Neutrophils
Hyper IgE aka Job Syndrome
do not respond to chemotactic stimuli.
What is the diag?
A patient suffers recurrent Neisseria
Infections. What part of complement is C5-C9, (LatE)
defective?
How does mechanism of Type 2 Type 2 = Ab against SELF antigens. Type 3 = Ab's against REAL
Hypersensitivity differ from Type 3? antigens. Complexs get stuck places and cause problems.
45 y/o female, MALAR RASH and
ARTHRITIS. Which Ab is specific for Anti dsDNA, Anti Smith. ANA is nonspecfic
the condition?
After bone marrow transplant a patient
suffers dermatitis, enteritis, and Graft vs Host Dz.
hepatitis? What is the condition?
A physican is looking for a risk factor
for Pancreatitis. He interviews 100 w/
Case Control
and 100 w/o pancreatitis. What kind of
study is this?
New glucose test arrives. You test it
with a solution of 90mg of glucose. High Precision; low accuracy (value stays in the 50s while not
The test gives you the following accurate which 50s value)
readings: 54, 56, 55, 54, 53, 56, 55, 54.
What is its presions and accuracy?
A group of ppl who smoke and do not
smoke are followed over 10 years.
Every two years they check who Cohort
develops cancer. What kind of study is
this?
A certain screening test has a 1% false
99%
negative rate. What is the sensitivity?
Prevalence of Varicella in Pop A is 2x
that of Pop B. It has the same incidence
Dz in Pop A has longer duration
in both populations. Why is the
prevalence different?
State the diagnosis: Gm (-), OXIDASE
Neisseria
(+), DIPLO COCCI
22 y/o medical student. Burning
feeling AFTER MEALS. EGD shows
H. Pylori
gm (-) RODS in gastric mucosa, what
are they?
50 y/o male smoker with new cough
and flu like symptoms. Gm stains
Legionella (atypical pneumonia)
shows nothing. SILVER STAINS
shows rods. What is the diagnosis?
40 y/o female. Acute unilateral knee
pain and bilateral BELLS PALSY. Lyme Disease via Burreli Burgdorferi via Tick
What organism? How is it transmitted?
21 y/o male. 5 day hx for fever chills
and enlarged painful knee. What Gonorrhea -Ceftriaxone or Azithyromyocin if allergy
organism? And what treatment?
After taking a course of Amoxicillin,
and adult pts develops toxic
C. Diff
MEGACOLON and DIARRHEA.
What caused this?
25 y/o with mycoplasma atypical
pneumonia, exhibits anemia due to
IgM
cryoagglutinins. What type of Ig is
responsible for anemia?
Homeless alcoholic pt vomited while
intoxicated. Develops FOUL smelling Klebsiella or anerobe
sputum. What organism?
65 y/o asks husband to stay in hospital
overnight b/c she is afraid of being
Regression
alone. What defense mechanism is
this?
Which defense mechanism underlies
Repression
all others?
60 y/o man admitted for chest pain,
jumps out of bed and does 50 push ups
Denial
to show he has not had a heart attack.
What is the defense mechanism?
4 y/o girl complains of painful
genitalia. On exam discharge with
Sexual Abuse
smear showing N. gonorrhoeae. What
happened?
72 y/o patient is unable to recall 3
objects during mini mental status
exam. When asked what he would do if
he smelled smoke he says "yell fire". Dementia
When asked what a table can chair
have in common he says both are made
of wood. Family reports he needs const
72 y/o brought to the clinic by family.
Strange behaviors in last week. Very
agitated, NAPS frequently during the
Delerium
day, URINATES on self, poor appetite.
Unable to focus during exam.
Diagnosis?
You are on call and receive a call from
a nurse asking to give sleep medication
No diphenhydramine, no Benzos, use Trazadone or Haliperidol
(diphenhramine) to an ELDERLY pt.
with DEMENTIA. What do you do?
A patient tries to commit suicide by
What did she do after cutting her wrists? Call someone? Lie in a
cutting wrists. After beening cared for
bathtub?
in the ER, what question would you
ask to determine her level of
commitment?
28 y/o female with mild depression for
Dysthamia
2 yrs. What diagnosis?
2 months after losing her spouse a 42
YO female is having trouble eating, This is still with in normal. But you can tx the insomnia and help
concentrating, and sleeping/ What do with trazadone or something else
you do?
A patient on whom you want an MRI
tells you they are claustophobic. What Give two Benzos prior to MRI
can you do?
A young woman is anxious about her
1st pap smear and is told to realx and
Systamatic Desensitization
to imagine what the steps are. What is
the process and example of?
A woman has flashbacks about her boy
friends death one monnth ago in a hit
Normal Greif
and run accident. She often cries and
wishes for justice. Diagnosis?
Nurse has hypoglycemia with no
Malingering or Facistious depenidng on 2nd gain
elevation of C-protein. Diagnosis?
40 YO female tells you she is in love
with you. You refer her (which you
should never do during USMLE) and Splitting (Borderline )
she attempts suicide. What is this
personality disorder?
30 Yo woman tells you that you are the
best doctor and the nurses are very bad.
On subsequent visit she threatens to
change doctors because you do not feel Splitting (Borderline )
a specific lab test is justified. You also
notice several "scartches" on her left
arm. What persona
55 YO female wearing all black with a
black feather boa and excess lipstick. Histrionic
What type of personality disorder?
A pt. demands only the best most
famous doctor in town. What Narcissistic
personality disorder?
A patient returns from a trip to New
Mexico, now has pneumonitis. What is Coccidioidomycocces
fungal cause?
A 30 Yo female has "cauliflower" skin
lesion. Tissue biopsy shows broad
Blastomycoisis
based budding yeast. What is this
organism?
An HIV (+) pt with CSF showing
75/mm3 lymphocytes suddenly dies.
Cryptococcus
Yeast is identified in the CSF. What is
the diag?
A pt presents with a "rose garden
scenario" (thorn prick with ulcers
Sporothrix
along lymphatic drainage). What is
infection?
A Pt who visited Mexico presents with
Cryptosporidium (usually filtered from city water supply....) more
Bloody Diarrhea. What infection could
severe in AIDS
be found in the stool?
32 YO male went camping in N.
California 2 wks ago. Pt had a 2 day
stint of diarrhea and how has liver Entamoeba Histolytica (not Giardia b/c Jaundice is present)
damage and Jaundice. What is the
diagnosis?
Pt returns after 2 wk vacation in
Africa. Typical malaria presentation
Malaria cycle in RBS's causing lysis every 48-72 hrs
and recurrent fever. What is the
mechanism for the cyclic?
Which Fetal Vessel has the highest 02
Umbilical Vein (1 verin, 2 arteries)
concentration?
45 YO male with BP 160/90 on right
arm and 170/92 on the left arm. No
Coarctation of the Aorta (adult type)
pulse in feet or ankle. What is the
diagnosis?
Describe blood flow through a PDA? Left to right shunt. (during pregnancy not called "patent" so
Just for fun what keeps it open, what incorrect to say R-> L then becomes L-> R). Hear a continuous
closes it? machine murmer. Open = PGE, close = indomethacin
Monozygotic twins are delivered. One
This is twin transfusion, mostly like due to monochorionic, mono
is pale and has a hematocrit of 15% the
amniotic pregnancy. Twin with lower hct will do better due to
other is flushed with hct of 55%. What
"sludging" in the one with high hct.
caused this? who will do better?
Fusion of the maxillry process with the Medial Nasal Process.
A child presents with Cleft lip. What
Cleft lip is mostly a aesthetic defect were cleft pallate has
process failed?
functional defiect as well
23 YO male presents with one testicle.
Germ Cell tumor of teste
what is he at risk for?
24 YO male develops testicular cancer.
via Inguinal canal to para-aortic LN's
Mets spread where?
16 YO female with amenorhea. Pt
lacks uterus and uterine tubes. Has two
round structures in midline just Androgen Insensitive (46XY)
superior to labia majora. What is the
diagnosis?
While on an ACE-i a pts develops a No Ang II = build up of bradykinn = cough. Use AngioTensin II
cough. Why? what is a replacement? Receptor Blocker (ARB)
40 yo male on lipid lower meds.
Develops rash, puritis and diarrhea. Niacin, not allergic due to prostaglandins (take asprin b4 hand to
What drug? Is this a allergic rxn? what decrease) also dec with long term therapy)
mediates this response?
What is mechanism of action of Blocks the Na/K atpase pump. Na leaves cell via Na/Ca counter
Cardiac Glycosides (Digoxin)? transporter and increases intracellular Ca lvls = better contractility
An abdominal Aortic aneurysm is most
Atheroscerois
likley due to?
A pt with poorly controlled HTN has
actue sharp substernal pain raidiating
Dissecting Aorta
to the back. Death occurs within a few
hours. Diagnosis?
During a high school football game a
young athlete collapses and dies Hypertrophic cardiomyopathy
immediately. What is the condition?
What murmers are heard best in the
Mitral Stenosis/ Regurg. And Left sided S3 / S4
Left Lateral Decubitous Position?
80 YO male. systolic cresendo
decresendo murmur. What is the Aortic Stenosis (probabley due to calcified aorta)
condition?
IV drug user presents with Chest pain,
Bacterial Endocardidits -> PE (remeber drug users get it on the
dyspnea, tachycardia, tachypnea. What
right)
is the condition?
Pt brought to ER after MVA presents
Tension Pneumothorax most likely, could be Cardiac Tamponade
with chest pain, dyspnea, tachycardia,
too
tachypnea. What is the condition?
Post-op pt presents with Chest pain,
dyspnea, tachycardia, tachypnea. What PE
is the condition?
A young girl with a congeital valve dz
is given penicillin prophylactically. In IV Vancomyocin and possible echo to check valves (might need
the ER bacterial endocarditis is diag. replacement)
What is next?
An adult pt with hx of HTN presents
with sudden sharp, tearing pain,
Widening of the mediastinum (Dissecting Aorta)
radiating to his back. What do you see
on CXR?
On auscultation of a patient you hear a
pansystolic murmur at the apex with Mitral Regurgitation (Aortic Stenosis is not PANsystolic)
radiation to the axilla. Cause?
A 25 YO pregnant woman in her 3rd
trimester has normal BP when standing
Compression of IVC, dont lie on your back.
and sitting but drops to 90/50 when she
lies supine. What is the condition?
45 YO male with squamous cell
carcinoma of the penis. He had HPV - 16 / 18
exposure to what Virus?
20 YO college student presents with
LAD, fever, and hepatosplenmegaly. B Cells - EBV mononucleosis (+) monospot. Cause of (-)
His serum agglutinates sheep RBC's. monospot mononucleosis? CMV
What cell is infected?
How does rabies travel through the
CNS to cause fatal encephalitis and Retrograde along neurons
seizures?
What is the characteristic shape of
Bullet Shaped
rabies?
An adolescent presents with cough and
rust colored sputum. What does gm Strep. Pneumo -> Gm (+) diplococci
stain show?
HIV (+) pt with a CD4 count of 250
presents with signs of meningitis. CSF
Cryptococcus Neoformans
shows a heavily encapsulated
organism. What is it?
An older patient has blood in his urine
Proteus
and renal stones. What organism?
A 50 YO pt is recovering from Abd
surgery from 2 days ago. He has had an
internal catheter in place since then. He E-Coli (UTI)
now has a fever of 100F. Most likely
organism?
Hemidesmisomes, cadherin, integrin,
ICAM-1. Which joins only cells of teh
Cadherin
same type and does not attach to the
basement membrane?
Where does new bone formation take
Epipseal Plate
place in growing long bones?
A football player was kick in the legs
and suffered a damaged medial
ACL, MCL
meniscus. What else is likely to have
been damaged?
A man presents with pain and swelling
of the knees, subcutaneous nodules
around the joints and achilles tendon,
equisite pain in the Most likely Gout
metatarsophalangeal joint of his right
big toe. Biopsy reveals needle like
crystals.
Treatment of Acute Gout
NSAID, Colchicine
exacerbation?
A patient has difficulty swallowing,
distal cyanosis in cold temp, anti-
CREST: Calcinosis, Sclerodactyly, Telangiectasia
centromere antibodies. What other S/S
will you see?
A patient presents with
photosensitivity, arthritis, renal disease
and recurrent oral ulcers. She is taking Renal Fxn (SLE)
Primaquine and NSAIDS. What should
be checked 2/yr?
30 yo woman presents with low grade
fever, rash across her nose and gets
worse in the sun and widespread ANA - SLE
edema. What blood test would you use
to screen?
A CT scan of the chest shows bilateral
Sarcoidosis
hilar LAD. What is the diagnosis?
A 75 yo male presents with acute knee
pain and swelling. X ray reveals
erosion of the joint space and calcium PsuedoGout -> Calcium pyrophosphate
deposits in the menisci. What is Diag?
What would be found on FNA of joint?
A 50 YO female complains of double
vision, amenorrhea and headaches. Prolactinoma
What is likely diagnosis?
A patients MRI shows replacment of
tissue in the sella tursica with CSF. Most likley asymptomatic or defieciency in Pituitary Hormones
What is the presentation?
What hormones come from the anterior
FSH, LH, ACTH, GH, prolactin, TSH
pituitary?
Which hormones share a common
FSH, TSH, LH, bHCG
alpha unit?
A young woman is found to have short
Albrights osteodystrophy (Pseudohypoparathydroiism) aka body
stature and shortened 4th and 5th
not responsive to PTH
metacarpals. What endocrine disorder
is this?
35 yo female presents with diffuse
goiter and hyperthyroidism. What is Low TSH High T3,T4
TSH / T3/ T4?
48 YO female presents with
progressive lethargy, and extreme cold Hypothyroid / Hasimotos most likely (High TSH low T3/4)
sensitivity. What is Diag? Lab values?
An adult male with elevated serum
cortisol and signs of cushing syndrome
undergoes dexamethasone suppresion. ACTH secreting pituitary adenoma
1mg does not decrease cortisol, 8mg
does. What is diag?
A very tan child with pale mother
comes in and is found to be Addisons
hypotensive. What is the condition?
28 YO male with normal well managed
IDDM comes in with DKA hae had
Infection
recently been taking OTC cold
medicine. What caused his DKA?
How is hemoglobin glycosylated in
Non enzymatically (slowly do to glucose build up)
DM to make HA1c?
What are the sources of Carbon for
Co2, Glycine, tetrahydrofolate / CO2 and aspartate
Purine formation? For pyrimidine?
How does UV light damage DNA? Causes Thymine Dimers to form
What ammino acid frequently has more
coding sequences in mRNAt the AUG - methione
represented in the peptide?
What happens to nRNA before it
Spliced (remove introns), Poly A tail, and 5' Cap
leaves the nucleus?
Two pts have the same mutation on
chromosome 15. but they have
different phenotypic expressions. One
Genetic Imprinting
has a mutation from the father the other
from the mother. What is this an
example of?
An obese woman presents with Polycystic Ovarian Syndrome (stein-leventhaul)
amenorrhea and increased serum
testosterone. Diag?
What type of cancer are patients with
polycystic ovarian syndrome at risk Endometrial
for?
Why is progesterone used in combo
To protect Uterus / endometrium from unregulated hyperplasia /
with estrogen in hormone replacement
cancer
therapy?
What circumstance would cause an Polycystic Ovarian Syndrome (stein-leventhaul), physiological LH
elevated LH? surge, low estrogen lvls, Turner Syndrome
A pregnant woman with previous C
section is at increased risk for what Placenta Accreta, Previa
pregnancy complications?
A pregnant women at 16 weeks
gestation presents with large abd and Hydatifrom Mole, increased beta HCG
HTN. Diag? Lab values?
What substance is elevated in
beta HCG (Really high in complete, slightly elevated in partial)
hydatifrom moles?
15 YO pt who normally comes in with
her parents presents alone. She states Check for delayed puberty, talk about how you can get pregant on
she is sexually active but knows she is your first time even before you menstrated, STDs and maybe
not pregnant because she has never contraceptives
menstrated. What should you tell her?
23 YO female is on rifampin for TB Rifampin increase Cyp450 metabolism of OCP, decreasing their
and OCP. She gets pregnant. Why? effectiveness
What is the best option of birth control
Medroxyprogesterone (injection q3 months)
of mental retarted pts?
A 58 YO post menopausal women is
on Tamoxifen. What is she at risk of Endometrial Carcinoma
acquiring?
What cells are responsible for
maintaing a high testosterone Leydig - Secrete; Sertoli - release of ABG = holds testosterone in
concentration in the seminiferous place
tubules?
A 55 y/o man undergoing tx for BPH Finasteride - 5 alpha reductase inhibitor (also used to treat renal
has increased testosterone and stones)
decreased DHT as well as
gynecomastia and edema. What
medication is he on?
Where does testicular cancer first
Para Aorotic Lymph Nodes
metastasize?
What protein is involved in
transporting an endocytosed vesicle
Clatherin
from the plasma membrane to the
endosome?
A patient with a corticol lesion is
UNAWARE of his neurologic Located on non dominate parietal Lobe (usually right)
deficiency. Where is the lesion?
Ipsilateral UMN lesion below the lesion | Ipsilateral loss of tactile,
What are the findings of Brown vibration and proprioception below lesion | Contralateral pain and
Sequard Syndrome? temp loss below lesion | ipsilateral all sension for a few levels
above lesion | LMN at level of lesion
A man in his 40s begins to develop
early dementia and uncontrolable
movements of his upper extremities. Caudate -> Huntingtons
where in the brain do you expect to see
atrophy?
A male presents with involuntary
flailing of one arm. Where is the This is hemibalismus. Contral lateral thalamus
lesion?
28 y/o chemist presents with MPTP
exposure. What neurotransmitter is Dopamine
depleted?
A patient cannot abduct her left eye on
lateral gaze but convergence is normal.
CN IV (MLF tract) + CN VII. Both are at level of Pontine
She also has difficulty smiling. Where
in the CNS is the lesion?
28 y/o woman in a MVA. Initally feels
fine then loses consiousness. CT shows
intracranial hemorrhage that does not MMA, and temporal bone
cross suture lines. What bone and
vessel were damaged?
85 y/o man with alzheimers falls at
home and presents 3 days later with
Subdural hematoma bridging veins
severe headache and vomiting. What is
the diagnosis? What is damaged?
A woman involved in a accident
cannot turn her head to the left and has
CN XI - accessory
a right shoulder droop. What is
damaged?
A 19 y/o pt presents with a furuncle on
his philtrum and the cavernous sinous
defects in CN 3, 4, 6
becomes infected. What might you
see?
A pt has a leftward deviation of the
tongue on protusion and has a right Left Medulla + corticospinal tract (happens before the tract
sided spastic paralysis. Where is the decusates)
lesion?

Вам также может понравиться